NCLEX-questions Electrolytes

You might also like

Download as pdf or txt
Download as pdf or txt
You are on page 1of 96

A nurse is assessing an electrocardiogram rhythm strip.

The P waves and QRS


complexes are regular. The PR interval is 0.16 second, and QRS complexes measure 0.06
second. The overall heart rate is 64 beats per minute. The nurse assesses the cardiac
rhythm as:
- Normal Sinus Rhythm

Correct Answer: A. Normal sinus rhythm


● Measurements are normal, measuring 0.12 to 0.20 second and 0.4 to 0.10 second,
respectively. Sinus rhythms originate in the SA node. The SA node is located in
the right atrium and is the heart’s natural pacemaker. The normal rate of the SA
node is between 60 and 100. On ECG, sinus rhythm is represented by
monomorphic P waves before each QRS complex and is regular.

● Option B: Sinus bradycardia is a cardiac rhythm with appropriate cardiac muscle


depolarization initiating from the sinus node generating less than 60 beats per minute
(bpm). The diagnosis of sinus bradycardia requires visualization of an electrocardiogram
showing a normal sinus rhythm at a rate lower than 60 bpm.
● Option C: Sick sinus syndrome, also known as sinus node dysfunction (SND), is a
disorder of the sinoatrial (SA) node caused by impaired pacemaker function and impulse
transmission producing a constellation of abnormal rhythms. These include atrial
bradyarrhythmias, atrial tachyarrhythmias and, sometimes, bradycardia alternates with
tachycardia often referred to as “tachy-brady syndrome.”
● Option D: First-degree atrioventricular (AV) block is a condition of abnormally slow
conduction through the AV node. It is defined by ECG changes that include a PR interval
of greater than 0.20 without disruption of atrial to ventricular conduction.

A nurse notices frequent artifacts on the ECG monitor for a client whose leads are
connected by cable to a console at the bedside. The nurse examines the client to
determine the cause. Which of the following items is unlikely to be responsible for the
artifact?
● A. Frequent movement of the client.
● B. Tightly secured cable connections.
● C. Leads applied over hairy areas.
● D. Leads applied to the limbs.

Correct Answer: B. Tightly secured cable connections.


Motion artifact, or “noise,” can be caused by frequent client movement, electrode
placement on limbs, and insufficient adhesion to the skin, such as placing electrodes
over hairy areas of the skin. Electrode placement over bony prominences also should be
avoided. Signal interference can also occur with electrode removal and cable
disconnection. The artifacts produced by alternating current cause a “darkened
reinforcement” in the ECG baseline, often making an analysis of rhythm difficult. This is
due to lack of filters for alternating current systems or a poor operation of the device.

● Option A: In Parkinson’s disease and parkinsonian syndromes, continuous muscle


twitching can be mistaken for atrial flutter (pseudo?atrial flutter) due to gross and
constant irregularities with a 300 bpm rate. In such cases, electrodes should be placed
at the upper part of the arms and legs. To eliminate limb tremors due to Parkinson’s
disease, the electrodes should be placed at the roots of the limbs, which attenuates or
abolishes the tremors and the myopotentials.
● Option C: Incorrect connections of electrodes during ECG recordings may resemble
rhythm or conduction alterations, myocardial ischemia or infarction. They also cause
shifts in P waves and the QRS axis, and they may mimic ectopic atrial rhythms,
fascicular block or dextrocardia; the latter occurring with left arm?right arm reversal.
● Option D: A frequent mistake when performing an ECG is a positioning of the right
precordial leads (V1, V2, V3); too high or too low, i.e., above the 4th intercostal space
observed in 50% of tracings or inferior and left shift in 30%–50% of the cases in leads V4
through V6, which indicates that these lateral precordial leads are commonly placed
outside their respective anatomical sites.

A nurse is watching the cardiac monitor and notices that the rhythm suddenly changes.
There are no P waves, the QRS complexes are wide, and the ventricular rate is regular
but over 100. The nurse determines that the client is experiencing:
● A. Premature ventricular contractions
● B. Ventricular tachycardia
● C. Ventricular fibrillation
● D. Sinus tachycardia

Correct Answer: B. Ventricular tachycardia


Ventricular tachycardia is characterized by the absence of P waves, wide QRS complexes
(usually greater than 0.14 second), and a rate between 100 and 250 impulses per minute.
The rhythm is usually regular. Ventricular tachycardia is characterized as a wide complex
(QRS duration greater than 120 milliseconds) tachyarrhythmia at a heart rate greater than
100 beats per minute. It is classified by duration as non-sustained or sustained.
Non-sustained ventricular tachycardia is defined as more than 3 beats of ventricular
origin at a rate greater than 100 beats per minute that lasts less than 30 seconds in
duration.
● Option A: During a premature ventricular contraction (PVC), the heartbeat is initiated by
the Purkinje fibers rather than the SA node. Given that a PVC occurs before a regular
heartbeat, there is a pause before the next regular heartbeat. In patients with PVCs, the
ECG may reveal other findings that include: electrolyte abnormalities (peaked T waves,
QT prolongation); left ventricular hypertrophy; with an old MI, one may see Q waves,
loss of R waves, and/or a bundle branch block; and acute ischemia may present with
ST-segment elevation/depression and/or T wave inversion.
● Option C: VF is a WCT caused by irregular electrical activity and characterized by a
ventricular rate of usually greater than 300 with discrete QRS complexes on the
electrocardiogram (ECG). QRS morphology in VF varies in shape, amplitude, and
duration with a prominent irregular rhythm.
● Option D: Sinus tachycardia is a regular cardiac rhythm in which the heart beats faster
than normal and results in an increase in cardiac output. While it is common to have
sinus tachycardia as a compensatory response to exercise or stress, it becomes
concerning when it occurs at rest.

A nurse is viewing the cardiac monitor in a client’s room and notes that the client has
just gone into ventricular tachycardia. The client is awake and alert and has good skin
color. The nurse would prepare to do which of the following?

● A. Immediately defibrillate.
● B. Prepare for pacemaker insertion.
● C. Administer amiodarone (Cordarone) intravenously.
● D. Administer epinephrine (Adrenaline) intravenously.
Correct Answer: C. Administer amiodarone (Cordarone) intravenously.
First-line treatment of ventricular tachycardia in a client who is hemodynamically stable
is the use of antidysrhythmics such as amiodarone (Cordarone), lidocaine (Xylocaine),
and procainamide (Pronestyl). Cardioversion also may be needed to correct the rhythm
(cardioversion is recommended for stable ventricular tachycardia). Procainamide will
terminate between 50% and 80% of ventricular tachycardias, and it will slow the
conduction of those that it does not terminate. Amiodarone will convert about 30% of
patients to sinus rhythm but is very effective in reducing the reversion rate of refractory
SMVT.

● Option A: Defibrillation is used with pulseless ventricular tachycardia. Pulseless VT


requires immediate electrical cardioversion with a high-energy defibrillator (150-200 J on
biphasic and 360 J on monophasic). Delaying defibrillation for 2 minutes or more
decreases survival rate compared with patients receiving immediate defibrillation (39,3%
vs. 22,2%). Defibrillation requires fewer joules if it is done early. After every shock, chest
compressions should be performed, along with oxygen delivery and intravenous injection
of vasopressors and antiarrhythmic drugs.
● Option B: The most common indications for permanent pacemaker implantation are
sinus node dysfunction (SND) and high-grade atrioventricular (AV) block. Pacemakers
are electronic devices that stimulate the heart with electrical impulses to maintain or
restore a functional heartbeat. Pacemakers were initially external and involved the
placement of subcutaneous electrodes for patients with inappropriate intrinsic cardiac
pacemaker activity and/or abnormal conducting tissue.
● Option D: Epinephrine would stimulate an already excitable ventricle and is
contraindicated. There are no absolute contraindications against using epinephrine.
Some relative contraindications include hypersensitivity to sympathomimetic drugs,
closed-angle glaucoma, anesthesia with halothane. Another unique contraindication to
be aware of is catecholaminergic polymorphic ventricular tachycardia. As is the case
with prescribing any medication, all practitioners should use clinical judgment and
evaluate the benefits versus risks with epinephrine.

A nurse is caring for a client with unstable ventricular tachycardia. The nurse instructs
the client to do which of the following, if prescribed, during an episode of ventricular
tachycardia?
● A. Breathe deeply, regularly, and easily.
● B. Inhale deeply and cough forcefully every 1 to 3 seconds.
● C. Lie down flat in bed.
● D. Remove any metal jewelry.

Correct Answer: B. Inhale deeply and cough forcefully every 1 to 3 seconds.


Cough Cardiopulmonary Resuscitation (CPR) sometimes is used in the client with
unstable ventricular tachycardia. The nurse tells the client to use cough CPR, if
prescribed, by inhaling deeply and coughing forcefully every 1 to 3 seconds. Cough CPR
may terminate the dysrhythmia or sustain the cerebral and coronary circulation for a
short time until other measures can be implemented. A nurse or physician can instruct
and coach the patients to cough forcefully every one to three seconds during the initial
seconds of a sudden arrhythmia. But because it’s not effective in all patients, it shouldn’t
delay definitive treatment.

● Option A: Asymptomatic patients with non-sustained ventricular tachycardia (VT) and no


underlying cardiac comorbidities require no additional therapy. Patients that are
symptomatic and without cardiac comorbidities should be started on a beta-blocker due
to favorable efficacy and safety profile.
● Option C: If these patients continue to have episodes of non-sustained VT despite
beta-blocker therapy, or cannot tolerate beta-blocker therapy, a calcium channel with
atrioventricular nodal action such as verapamil or diltiazem can be used.
● Option D: Patients with sustained monomorphic ventricular tachycardia (SMVT) that are
unstable should be managed following advanced cardiac life support (ACLS) guidelines.
Hemodynamically stable patients should be pharmacologically cardioverted using an
antiarrhythmic medication. Intravenous amiodarone or procainamide can be used for this
purpose.

The client is having frequent premature ventricular contractions. A nurse would place a
priority on the assessment of which of the following items?
● A. Blood pressure and peripheral perfusion.
● B. Sensation of palpitations.
● C. Causative factors such as caffeine.
● D. Precipitating factors such as infection.

Correct Answer: A. Blood pressure and peripheral perfusion.


Premature ventricular contractions can cause hemodynamic compromise. The shortened
ventricular filling time with the ectopic beats leads to decreased stroke volume and, if
frequent enough, to decreased cardiac output. Physical examination findings would
reveal an irregular heart rhythm upon auscultation if the patient is experiencing PVCs
during the examination. In some patients, cannon A waves may cause chest or neck
discomfort. Otherwise, there would not be any direct physical examination findings. A
prolonged run of PVCs can result in hypotension.
● Option B: The client may be asymptomatic or may feel palpitations. A thorough history
should include any associated symptoms with the palpitations, the patient’s medical
history, medication, and supplement usage as well as a detailed social history. It is
crucial to inquire about any illicit drug use in those who frequently experience PVCs.
● Option C: Common known etiologies include excess caffeine consumption, excess
catecholamines, high levels of anxiety, and electrolyte abnormalities. Specific electrolyte
changes found in those who experience PVCs are low blood potassium, low blood
magnesium, and high blood calcium. Alcohol, tobacco, and illicit drugs are also
associated with PVCs as are stimulant-based medications. Patients suffering from sleep
deprivation also experience PVCs.
● Option D: PVCs can be caused by cardiac disorders or by any number of physiological
stressors, such as infection, illness, surgery, or trauma, and by the intake of caffeine,
alcohol, or nicotine. There are numerous cardiac and non-cardiac pathologies that are
causative of PVCs. Examples include cardiomyopathy, mitral valve prolapse, and
myocardial infarction. Any structural heart disease that alters conduction pathways due
to tissue alterations can cause PVCs. Non-cardiac examples are hyperthyroidism,
anemia, and even hypertension.

A client has developed atrial fibrillation, which has a ventricular rate of 150 beats per
minute. A nurse assesses the client for:
● A. Hypotension and dizziness
● B. Nausea and vomiting
● C. Hypertension and headache
● D. Flat neck veins

A. Hypotension and dizziness


The client with uncontrolled atrial fibrillation with a ventricular rate more than 150 beats a
minute is at risk for low cardiac output because of loss of atrial kick. The nurse assesses
the client for palpitations, chest pain or discomfort, hypotension, pulse deficit, fatigue,
weakness, dizziness, syncope, shortness of breath, and distended neck veins. A physical
exam should always begin with the assessment of airway breathing and circulation as it
is going to affect the decision making regarding management. On general physical
examination, patients may have tachycardia with an irregularly irregular pulse.
● Option B: The presentation of AF can range from asymptomatic to devastating
complications such as cardiogenic shock and ischemic stroke. A complete history should
focus on symptoms such as palpitations, chest pain, shortness of breath, increased
lower extremity swelling, dyspnea on exertion, and dizziness.
● Option C: In addition, history is imperative in identifying risk factors such as
hypertension, history of valvular, structural, or ischemic heart disease, obstructive sleep
apnea, obesity hypoventilation syndrome, smoking, alcohol intake, illicit drug use, history
of rheumatic fever/heart disease, history of pericarditis, and hyperlipidemia.
● Option D: The physical exam should focus on identifying the cause of AF. For instance,
examining the neck of the patient may give some clues regarding carotid artery disease
or thyroid problems. The pulmonary examination may reveal signs of heart failure in the
form of rales, and the presence of wheeze may indicate antecedent pulmonary diseases
such as asthma and chronic obstructive pulmonary disease (COPD).

A nurse is watching the cardiac monitor, and a client’s rhythm suddenly changes. There
are no P waves; instead, there are wavy lines. The QRS complexes measure 0.08 second,
but they are irregular, with a rate of 120 beats a minute. The nurse interprets this rhythm
as:
● A. Sinus tachycardia
● B. Atrial fibrillation
● C. Ventricular tachycardia
● D. Ventricular fibrillation

Correct Answer: B. Atrial fibrillation


Atrial fibrillation is characterized by a loss of P waves; an undulating, wavy baseline;
QRS duration that is often within normal limits; and an irregular ventricular rate, which
can range from 60 to 100 beats per minute (when controlled with medications) to 100 to
160 beats per minute (when uncontrolled). Atrial fibrillation is the most common type of
cardiac arrhythmia. It is the leading cardiac cause of stroke. Risk factors for atrial
fibrillation include advanced age, high blood pressure, underlying heart and lung
disease, congenital heart disease, and increased alcohol consumption.

● Option A: Sinus tachycardia is a regular cardiac rhythm in which the heart beats faster
than normal and results in an increase in cardiac output. While it is common to have
sinus tachycardia as a compensatory response to exercise or stress, it becomes
concerning when it occurs at rest.
● Option C: Ventricular tachycardia is characterized by the absence of P waves, wide QRS
complexes (usually greater than 0.14 second), and a rate between 100 and 250
impulses per minute. The rhythm is usually regular. Ventricular tachycardia is
characterized as a wide complex (QRS duration greater than 120 milliseconds)
tachyarrhythmia at a heart rate greater than 100 beats per minute. It is classified by
duration as non-sustained or sustained. Non-sustained ventricular tachycardia is defined
as more than 3 beats of ventricular origin at a rate greater than 100 beats per minute
that lasts less than 30 seconds in duration.
● Option D: VF is a WCT caused by irregular electrical activity and characterized by a
ventricular rate of usually greater than 300 with discrete QRS complexes on the
electrocardiogram (ECG). QRS morphology in VF varies in shape, amplitude, and
duration with a prominent irregular rhythm.

A client with rapid rate atrial fibrillation asks a nurse why the physician is going to
perform carotid massage. The nurse responds that this procedure may stimulate the:
● A. Vagus nerve to slow the heart rate.
● B. Vagus nerve to increase the heart rate; overdriving the rhythm.
● C. Diaphragmatic nerve to slow the heart rate.
● D. Diaphragmatic nerve to overdrive the rhythm.

Correct Answer: A. Vagus nerve to slow the heart rate.


Carotid sinus massage is one of the maneuvers used for vagal stimulation to decrease a
rapid heart rate and possibly terminate a tachydysrhythmias. The others include inducing
the gag reflex and asking the client to strain or bear down. Medication therapy often is
needed as an adjunct to keep the rate down or maintain the normal rhythm. Vagal
maneuvers are techniques used to increase vagal parasympathetic tone in an attempt to
diagnose and treat various arrhythmias. They are often utilized first in an effort to abort
episodes of stable supraventricular tachycardia (SVT) or differentiate SVT from
ventricular tachycardias (VT).

● Option B: Vagal maneuvers, including Carotid Sinus Massage and Valsalva Maneuver,
transiently increase the arterial pressure in the carotid sinuses and aortic arch. This
action triggers the baroreceptor reflex, which results in increased parasympathetic output
to the heart via the vagus nerve (cranial nerve X).
● Option C: The location of the carotid sinus is at the bifurcation of the internal and
external carotid artery from the common carotid artery. The sinus contains baroreceptors
that sense changes in blood pressure. Afferent signals are then transmitted via the
glossopharyngeal nerve (cranial nerve IX) to the nucleus tractus solitarius (NTS) within
the medulla.
● Option D: Within the heart, the right vagus nerve serves to stimulate the sinoatrial (SA)
node, the pacemaker of the healthy heart, in the right atrium; this causes slowed
electrical activity within the SA node. The left vagus nerve mostly innervates
atrioventricular (AV) node, which slows conduction between the atria and the ventricles.
The end product of vagal stimulation is a decrease in the speed and frequency of
electrical impulses in the heart, which could ultimately slow or terminate a
tachydysrhythmia.
A nurse notes that a client with sinus rhythm has a premature ventricular contraction that
falls on the T wave of the preceding beat. The client’s rhythm suddenly changes to one
with no P waves or definable QRS complexes. Instead, there are coarse wavy lines of
varying amplitude. The nurse assesses this rhythm to be:
● A. Ventricular tachycardia
● B. Ventricular fibrillation
● C. Atrial fibrillation
● D. Asystole

Correct Answer: B. Ventricular fibrillation


Ventricular fibrillation is characterized by irregular, chaotic undulations of varying
amplitudes. Ventricular fibrillation has no measurable rate and no visible P waves or QRS
complexes and results from electrical chaos in the ventricles. VF is a WCT caused by
irregular electrical activity and characterized by a ventricular rate of usually greater than
300 with discrete QRS complexes on the electrocardiogram (ECG). QRS morphology in
VF varies in shape, amplitude, and duration with a prominent irregular rhythm.
● Option A: Ventricular tachycardia is characterized by the absence of P waves, wide QRS
complexes (usually greater than 0.14 second), and a rate between 100 and 250
impulses per minute. The rhythm is usually regular. Ventricular tachycardia is
characterized as a wide complex (QRS duration greater than 120 milliseconds)
tachyarrhythmia at a heart rate greater than 100 beats per minute. It is classified by
duration as non-sustained or sustained. Non-sustained ventricular tachycardia is defined
as more than 3 beats of ventricular origin at a rate greater than 100 beats per minute
that lasts less than 30 seconds in duration.
● Option C: Atrial fibrillation is characterized by a loss of P waves; an undulating, wavy
baseline; QRS duration that is often within normal limits; and an irregular ventricular rate,
which can range from 60 to 100 beats per minute (when controlled with medications) to
100 to 160 beats per minute (when uncontrolled). Atrial fibrillation is the most common
type of cardiac arrhythmia. It is the leading cardiac cause of stroke. Risk factors for atrial
fibrillation include advanced age, high blood pressure, underlying heart and lung
disease, congenital heart disease, and increased alcohol consumption.
● Option D: Asystole, colloquially referred to as flatline, represents the cessation of
electrical and mechanical activity of the heart. Asystole typically occurs as a deterioration
of the initial non-perfusing ventricular rhythms: ventricular fibrillation (V-fib) or pulseless
ventricular tachycardia (V-tach). Additionally, pulseless electrical activity (PEA) can
cease and become asystole.

While caring for a client who has sustained an MI, the nurse notes eight PVCs in one
minute on the cardiac monitor. The client is receiving an IV infusion of D5W and oxygen
at 2 L/minute. The nurse’s first course of action should be to:
● A. Increase the IV infusion rate.
● B. Notify the physician promptly.
● C. Increase the oxygen concentration.
● D. Administer a prescribed analgesic.

Correct Answer: B. Notify the physician promptly.


PVCs are often a precursor of life-threatening dysrhythmias, including ventricular
tachycardia and ventricular fibrillation. An occasional PVC is not considered dangerous,
but if PVCs occur at a rate greater than 5 or 6 per minute in the post-MI client, the
physician should be notified immediately. More than 6 PVCs per minute is considered
serious and usually calls for decreasing ventricular irritability by administering
medications such as lidocaine.

● Option A: Increasing the IV infusion rate would not decrease the number of PVCs. Those
experiencing frequent PVCs or symptomatic PVCs should be evaluated to identify the
etiology. In many cases, excess intake of stimulants and/or lower levels of potassium
and magnesium is the cause of the PVCs. These patients can be easily managed via
minimization of stimulants and/or repletion of electrolytes.
● Option C: Increasing the oxygen concentration should not be the nurse’s first course of
action; rather, the nurse should notify the physician promptly. In the emergency room,
hypoxic patients need to be provided with oxygen, the electrolyte imbalance should be
corrected and drug toxicity should be ruled out. At the same time, an acute MI must be
ruled out.
● Option D: Administering a prescribed analgesic would not decrease ventricular irritability.
The medication classes used to treat frequent and/or symptomatic PVCs include
antiarrhythmics, beta-blockers, and calcium channel blockers. Commonly used
antiarrhythmics include amiodarone and flecainide.

The adaptations of a client with complete heart block would most likely include:
● A. Nausea and vertigo
● B. Flushing and slurred speech
● C. Cephalalgia and blurred vision
● D. Syncope and slow ventricular rate

Correct Answer: D. Syncope and slow ventricular rate


In complete atrioventricular block, the ventricles take over the pacemaker function in the
heart but at a much slower rate than that of the SA node. As a result, there is decreased
cerebral circulation, causing syncope. Patients with third-degree blocks can have varying
clinical presentations. Rarely, patients are asymptomatic. Usually, they may present with
generalized fatigue, tiredness, chest pain, shortness of breath, presyncope, or syncope.
They may have significant hemodynamic instability and can be obtunded.
● Option A: The physical exam is usually remarkable for bradycardia. JVP examination
often demonstrates cannon A-waves owing to the simultaneous contraction of the atria
and ventricles. Thus a very large pressure wave is felt up against the vein.
● Option B: Especially with heart rates below 40/min, patients might also present with
features consistent with decompensated heart failure, respiratory distress, and
hypoperfusion such as diaphoresis, tachypnea, altered mental status, retraction, cool
skin, and decreased capillary refill.
● Option C: Patients with complete AV-block accompanying an acute myocardial infarction
often have ischemic symptoms of chest pain or dyspnea. The past medical history will
often include the presence of cardiovascular disease and/or its risk factors, including
diabetes mellitus, hypertension, dyslipidemia, and smoking, etc.

A client with a bundle branch block is on a cardiac monitor. The nurse should expect to
observe:
● A. Sagging ST segments.
● B. Absence of P wave configurations.
● C. Inverted T waves following each QRS complex.
● D. Widening of QRS complexes to 0.12 second or greater.

Correct Answer: D. Widening of QRS complexes to 0.12 second or greater.


Bundle branch block interferes with the conduction of impulses from the AV node to the
ventricle supplied by the affected bundle. Conduction through the ventricles is delayed,
as evidenced by a widened QRS complex. Rhythm must be of supraventricular origin
(EG: ventricular activation coming from atrial or AV nodal activation). Lead V1 should
have either a QS or a small r wave with large S wave. Lead V6 should have a notched R
wave and no Q wave.

● Option A: ST depression occurs when the J point is displaced below baseline. Just like
ST elevation, not all ST depression represents myocardial ischemia or an emergent
condition. There are multiple conditions associated with ST depression. Some of these
include hypokalemia, cardiac ischemia, and medications such as digitalis.
● Option B: Absence of P waves suggests either no normal atrial depolarization, e.g., atrial
fibrillation, atrial standstill; or the P waves are hidden within the QRS complexes, e.g.,
ventricular tachycardia, junctional tachycardia.
● Option C: Inverted T waves are associated with myocardial ischemia. The inversion of a
T wave is not specific for ischemia, and the inversion itself does not correlate with a
specific prognosis. However, if the clinical history is suggestive of ischemia in the setting
of inverted T waves, this is correlative.
When ventricular fibrillation occurs in a CCU, the first person reaching the client should:
● A. Administer oxygen.
● B. Defibrillate the client.
● C. Initiate CPR.
● D. Administer sodium bicarbonate intravenously.

Correct Answer: B. Defibrillate the client


Ventricular fibrillation is a death-producing dysrhythmia and, once identified, must be
terminated immediately by precordial shock (defibrillation). This is usually a standing
physician’s order in a CCU. Pulseless VT and VF are both shockable rhythms, and once
the staff identifies the rhythm as VF, patients should be shocked immediately with 120 to
200 joules on a biphasic defibrillator or 360 joules using a monophasic.

● Option A: Professionals should undertake cause-specific measures such as securing the


airway, correcting electrolytes, administering fluids, decompressing pneumothorax,
draining tamponade while resuscitating the patient. Once the patient attains return of
spontaneous circulation (ROSC), physicians should begin a definitive evaluation for
coronary artery disease.
● Option C: Due to the high mortality rate and extreme acuity of the condition, VF patients
warrant immediate attention. Healthcare professionals should immediately initiate
guideline-directed management as per Advanced Cardiac Life Support (ACLS) protocol.
There is a lower likelihood of survival if the healthcare professional deviates from the
ACLS guidelines. All patients with cardiac arrest should have an initial assessment while
receiving quality CPR.
● Option D: Administer epinephrine and amiodarone as per ACLS protocol in patients
sustaining VF rhythm regardless of receiving 3 shocks. Amiodarone significantly
improves survival to hospital admission without affecting survival to hospital discharge.

What criteria should the nurse use to determine normal sinus rhythm for a client on a
cardiac monitor? Select all that apply.
● A. The RR intervals are relatively consistent.
● B. One P wave precedes each QRS complex.
● C. Four to eight complexes occur in a 6-second strip.
● D. The ST segment is higher than the PR interval.
● E. The QRS complex ranges from 0.12 to 0.20 second.
Correct Answers: A, B.
The consistency of the RR interval indicates regular rhythm. A normal P wave before
each complex indicates the impulse originated in the SA node. Sinus arrhythmia is most
typically present in young, healthy individuals. Studies have attempted to establish an
increased prevalence in patients with underlying hypertension, obesity, and diabetes.
● Option A: Sinus rhythm (a.k.a. normal sinus rhythm) refers to the normal heart beat
originating from the sinoatrial node. This is manifested as an upright P wave in lead II of
the ECG. Sinus arrhythmia is a common rhythm variation. It is seen more often in
children and young adults. Respirations lead to vagal stimuli resulting in R-R interval
variations.
● Option B: Sinus arrhythmia is a commonly encountered variation of normal sinus rhythm.
Sinus arrhythmia characteristically presents with an irregular rate in which the variation
in the R-R interval is greater than 0.12 seconds. Additionally, P waves are typically
monoform and in a pattern consistent with atrial activation originating from the sinus
node.
● Option C: The number of complexes in a 6-second strip is multiplied by 10 to
approximate the heart rate; normal sinus rhythm is 60 to 100. On the EKG, sinus rhythm
appears as a beat-to-beat variation in the P-P interval. Typically, this variation is greater
than 120 msec: the P-P interval increases and decreases with inspiration and exhalation.
● Option D: Elevation of the ST segment is a sign of cardiac ischemia and is unrelated to
the rhythm. The ST Segment represents the interval between ventricular depolarization
and repolarization. The most important cause of ST segment abnormality (elevation or
depression) is myocardial ischaemia or infarction.
● Option E: The QRS duration should be less than 0.12 second; the PR interval should be
0.12 to 0.20 second. The normal duration (interval) of the QRS complex is between 0.08
and 0.10 seconds — that is, 80 and 100 milliseconds. When the duration is between
0.10 and 0.12 seconds, it is intermediate or slightly prolonged. A QRS duration of greater
than 0.12 seconds is considered abnormal.

When auscultating the apical pulse of a client who has atrial fibrillation, the nurse would
expect to hear a rhythm that is characterized by:
● A. The presence of occasional coupled beats.
● B. Long pauses in an otherwise regular rhythm.
● C. A continuous and totally unpredictable irregularity.
● D. Slow but strong and regular beats.

Correct Answer: C. A continuous and totally unpredictable irregularity.


In atrial fibrillation, multiple ectopic foci stimulate the atria to contract. The AV node is
unable to transmit all of these impulses to the ventricles, resulting in a pattern of highly
irregular ventricular contractions. Due to its rhythm irregularity, blood flow through the
heart becomes turbulent and has a high chance of forming a thrombus (blood clot),
which can ultimately dislodge and cause a stroke. Atrial fibrillation is the leading cardiac
cause of stroke.

● Option A: The most common sensation associated with PVCs is that of a skipped
heartbeat followed by a fluttering sensation. Patients commonly present complaining of
heart palpitations. The vast majority of patients are entirely asymptomatic as there are
no associated symptoms with the palpitations.
● Option B: In the presence of a pause, one should exclude premature complexes with
compensatory pause. If the ectopic beat failed to reset the sinus node, the premature
complex would be followed by a compensatory pause, i.e. the R-R interval after the
premature complex is longer than the R-R interval between normal sinus beats.
● Option D: Having bradycardia means that the heart beats very slowly. For most people, a
heart rate of 60 to 100 beats a minute while at rest is considered normal. If the heart
beats less than 60 times a minute, it is slower than normal. For some people, a slow
heart rate does not cause any problems. It can be a sign of being very fit. Healthy young
adults and athletes often have heart rates of less than 60 beats a minute.

Atherosclerosis impedes coronary blood flow by which of the following mechanisms?


● A. Plaques obstruct the vein.
● B. Plaques obstruct the artery.
● C. Blood clots form outside the vessel wall.
● D. Hardened vessels dilate to allow blood to flow through.

Correct Answer: B. Plaques obstruct the artery


Atherosclerosis is a chronic inflammatory disease in which there is a build up of plaques
inside arteries. These plaques are principally composed of lipids that induce an
inflammatory reaction causing turbulent flow with atherosclerotic cardiovascular disease
(ASCVD) as a result.
● Option A: Arteries, not veins, supply the coronary arteries with oxygen and other
nutrients. Atherosclerosis mainly develops through the continuous process of arterial
wall lesions due to lipid retention by trapping in the intima by a matrix such as
proteoglycans resulting in a modification which, in turn, aggravates chronic inflammation
at vulnerable sites in the arteries and plays an important role at all phases of the
atherogenic progression.
● Option C: Atherosclerosis is a direct result of plaque formation in the artery. This process
begins from the nascent fatty streaks in the arterial intima which evolve into fibrous
plaques and emerge into complex atherosclerotic lesions that are susceptible to rupture.
Also, stenosis from the inward expansion of the atheroma can result in occlusion of
vessels such as the coronaries.
● Option D: Hardened vessels can’t dilate properly and, therefore, constrict blood flow.
Many studies indicate the role of the endoplasmic reticulum stress pathway or unfolded
protein response as the chief mechanisms of macrophage cell death in plaques causing
accumulation of dead macrophages. Also in comparison to early plaques, microvessel
density is increased in advanced plaques by dysregulated neovascularization. In normal
and atherosclerotic arteries microvessels are thin-walled and characterized with poor
structural integrity and endothelial junctions.
A paradoxical pulse occurs in a client who had a coronary artery bypass graft (CABG)
surgery two (2) days ago. Which of the following surgical complications should the nurse
suspect?
● A. Left-sided heart failure
● B. Aortic regurgitation
● C. Complete heart block
● D. Pericardial tamponade

Correct Answer: D. Pericardial tamponade


A paradoxical pulse (a palpable decrease in pulse amplitude on quiet inspiration) signals
pericardial tamponade, a complication of CABG surgery. Cardiac tamponade is a medical
or traumatic emergency that happens when enough fluid accumulates in the pericardial
sac compressing the heart and leading to a decrease in cardiac output and shock.
● Option A: Left-sided heart failure can cause pulsus alternans (pulse amplitude
alternation from beat to beat, with a regular rhythm). Right ventricular alternans occur as
a result of right ventricular strain, often precipitated by a pulmonary embolism or
pulmonary hypertension. Other potential etiologies of right ventricular alternans include
reactive airway disease, mitral stenosis, or left-sided heart failure.
● Option B: Aortic regurgitation may cause bisferious pulse (an increased arterial pulse
with a double systolic peak). The most common causes of pulsus bisferiens are mixed
aortic valve disease (infective endocarditis, rheumatic heart disease, Marfan syndrome,
bicuspid aortic valve) and hypertrophic cardiomyopathy with obstruction (HOCM). Pulsus
bisferiens a single central pulse wave with two peaks separated by a distinct mid-systolic
dip. An early component percussion wave results from rapid left ventricular ejection. The
late component tidal wave represents a reflected wave from the periphery due to an
artery’s recoil effect.
● Option C: Complete heart block may cause a bounding pulse (a strong pulse with
increased pulse pressure). The physical exam is usually remarkable for bradycardia.
JVP examination often demonstrates cannon A-waves owing to the simultaneous
contraction of the atria and ventricles. Thus a very large pressure wave is felt up against
the vein.

After cardiac surgery, a client’s blood pressure measures 126/80. The nurse determines
that the mean arterial pressure (MAP) is which of the following?
● A. 46 mm Hg
● B. 80 mm Hg
● C. 95 mm Hg
● D. 90 mm Hg

Correct Answer: C. 95 mm Hg
The definition of mean arterial pressure (MAP) is the average arterial pressure
throughout one cardiac cycle, systole, and diastole. MAP is influenced by cardiac output
and systemic vascular resistance, each of which is under the influence of several
variables.
MAP = (2 times the diastolic + systolic)/3 or (2d + s)/3 where d is diastolic and s is
systolic
= 2 (80) + 126 / 3
= 160 + 126 / 3 = 286 / 3 = 95.33 or 95 mm HG

● Option A: Cardiac output is calculated as the product of heart rate and stroke volume.
The determination of stroke volume is by ventricular inotropy and preload. Preload is
affected by blood volume and the compliance of veins. Increasing the blood volume
increases the preload, increasing the stroke volume, and therefore increasing cardiac
output. Afterload also affects the stroke volume in that an increase in afterload will
decrease stroke volume. Heart rate is affected by the chronotropy, dromotropy, and
lusitropy of the myocardium.
● Option B: Systemic vascular resistance is determined primarily by the radius of the blood
vessels. Decreasing the radius of the vessels increases vascular resistance. Increasing
the radius of the vessels would have the opposite effect. Blood viscosity can also affect
systemic vascular resistance. An increase in hematocrit will increase blood viscosity and
increase systemic vascular resistance. Viscosity, however, is considered only to play a
minor role in systemic vascular resistance.
● Option D: A common method used to estimate the MAP is the following formula:
MAP = DP + 1/3(SP – DP) or MAP = DP + 1/3(PP)
where DP is the diastolic blood pressure, SP is the systolic blood pressure, and PP is
the pulse pressure. This method is often more conducive to measuring MAP in most
clinical settings as it offers a quick means of calculation if the blood pressure is known.

A woman with severe mitral stenosis and mitral regurgitation has a pulmonary artery
catheter inserted. The physician orders pulmonary artery pressure monitoring, including
pulmonary capillary wedge pressures. The purpose of this is to help assess the:
● A. Degree of coronary artery stenosis.
● B. Peripheral arterial pressure.
● C. Pressure from fluid within the left ventricle.
● D. Oxygen and carbon dioxide concentration is the blood.

Correct Answer: C. Pressure from fluid within the left ventricle.


The pulmonary artery pressures are used to assess the heart’s ability to receive and
pump blood. The pulmonary capillary wedge pressure reflects the left ventricle
end-diastolic pressure and guides the physician in determining fluid management for the
client. Pulmonary capillary wedge pressure (PCWP) is frequently used to assess left
ventricular filling, represent left atrial pressure, and assess mitral valve function. It is
measured by inserting a balloon-tipped, multi-lumen catheter (Swan-Ganz catheter) into a
central vein, and advancing the catheter into a branch of the pulmonary artery.

● Option A: The degree of coronary artery stenosis is assessed during a cardiac


catheterization. Although it is used for cardiac hemodynamics and assessment of
valvular lesions, its main diagnostic role is the assessment of coronary artery disease. In
the contemporary era, left heart catheterization, especially selective coronary angiogram,
is considered the gold standard test for coronary artery disease diagnosis.
● Option B: The peripheral arterial pressure is assessed with an arterial line. Arterial
pressure directly corresponds to cardiac output, arterial elasticity, and peripheral
vascular resistance. Blood pressure is remarkably easy to alter and can be affected by
many activities. In response to acute changes in blood pressure, the body responds
through the baroreceptors located within blood vessels. Baroreceptors are a form of
mechanoreceptor that becomes activated by the stretching of the vessel. This sensory
information is conveyed to the central nervous system and used to influence peripheral
vascular resistance and cardiac output.
● Option D: An arterial blood gases (ABG) test measures the acidity (pH) and the levels of
oxygen and carbon dioxide in the blood from an artery. This test is used to find out how
well the lungs are able to move oxygen into the blood and remove carbon dioxide from
the blood. As blood passes through the lungs, oxygen moves into the blood while carbon
dioxide moves out of the blood into the lungs. An ABG test uses blood drawn from an
artery, where the oxygen and carbon dioxide levels can be measured before they enter
body tissues.

FLUIDS AND ELECTROLYTES


Patient X is diagnosed with constipation. As a knowledgeable nurse, which nursing
intervention is appropriate for maintaining normal bowel function?
A. Assessing dietary intake
B. Decreasing fluid intake
C. Providing limited physical activity
D. Turning, coughing, and deep breathing

Correct Answer: A. Assessing dietary intake


Assessing dietary intake provides a foundation for the client’s usual practices and may
help determine if the client is prone to constipation or diarrhea. Check out usual dietary
habits, eating habits, eating schedule, and liquid intake. Irregular mealtime, type of food,
and interruption of the usual schedule can lead to constipation. Assist the patient to take
at least 20 g of dietary fiber (e.g., raw fruits, fresh vegetables, whole grains) per day.

Option B: Fluid intake should be increased to aid bowel elimination. Encourage


the patient to take in fluid 2000 to 3000 mL/day, if not contraindicated medically.
Sufficient fluid is needed to keep the fecal mass soft. But take note of some
patients or older patients having cardiovascular limitations requiring less fluid
intake.
Option C: Limited physical activity may contribute to constipation due to
decreased peristalsis. Assess the patient’s activity level. Sedentary lifestyles
such as sitting all day, lack of exercise, prolonged bed rest, and inactivity
contribute to constipation.
Option D: Turning, coughing and deep breathing help promote gas exchange.
Urge the patient for some physical activity and exercise. Consider isometric
abdominal and gluteal exercises. Movement promotes peristalsis. Abdominal
exercises strengthen abdominal muscles that facilitate defecation.

A 12-year-old boy was admitted to the hospital two days ago due to hyperthermia. His
attending nurse, Dennis, is quite unsure about his plan of care. Which of the following
nursing interventions should be included in the care plan for the client?
A. Room temperature reduction
B. Fluid restriction of 2,000 ml/day
C. Axillary temperature measurements every 4 hours
D. Antiemetic agent administration

Correct Answer: A. Room temperature reduction


For the patient with hyperthermia, reducing the room temperature may help
decrease the body temperature. Tepid baths, cool compresses, and cooling
blankets may also be necessary. Adjust and monitor environmental factors like
room temperature and bed linens as indicated. Room temperature may be
accustomed to near normal body temperature and blankets and linens may be
adjusted as indicated to regulate the temperature of the patient.
Option B: Fluids should be encouraged, not restricted to compensate for
insensible losses. Monitor fluid intake and urine output. If the patient is
unconscious, central venous pressure or pulmonary artery pressure should be
measured to monitor fluid status. Fluid resuscitation may be required to correct
dehydration. The patient who is significantly dehydrated is no longer able to
sweat, which is necessary for evaporative cooling.
Option C: Tympanic or rectal temperature measurements are generally accepted
and are more accurate than axillary measurements. Monitor the patient’s HR, BP,
and especially the tympanic or rectal temperature. HR and BP increase as
hyperthermia progresses. Tympanic or rectal temperature gives a more accurate
indication of core temperature.
Option D: Antipyretics, and not antiemetics, are indicated to reduce fever. Give
antipyretic medications as prescribed. Antipyretic medications lower body
temperature by blocking the synthesis of prostaglandins that act in the
hypothalamus.

Tom is ready to be discharged from the medical-surgical unit after 5 days of


hospitalization. Which client statement indicates to the nurse that Tom understands the
discharge teaching about cellular injury?
A. "I do not have to see my doctor unless I have problems."
B. "I can stop taking my antibiotics once I am feeling better."
C. "If I have redness, drainage, or fever, I should call my healthcare
provider."
D. "I can return to my normal activities as soon as I go home."

Correct Answer:
C. “If I have redness, drainage, or fever, I should call my healthcare provider.”
The knowledge that redness, drainage, or fever — signs of infection associated with
cellular injury — require reporting indicates that the client has understood the nurse’s
discharge teaching. If a cell is unable to adapt to increased stress, injury results. Cell
injury is reversible until a certain threshold where it progresses to cell death. Historically,
cell death has been designated into two classes: necrosis and apoptosis. Necrosis is
often coined as accidental death as it is generally seen as not controlled by the cell.
Apoptosis, on the other hand, is typically viewed as programmed cell death, regulated
and controlled.
Option A: Follow-up checkups should be encouraged. Cell growth, division, and
death are all important parts of this regulation, and each is highly regulated. Loss
of this balance is seen in tumor cells where mechanisms of cell death are
avoided, resulting in uncontrolled cell growth. Conversely, conditions where
extensive cell death is seen also result in loss of homeostasis, such as in the
case of neuronal loss in Alzheimer’s disease.
Option B: The nurse should place an emphasis on antibiotic compliance even if
the client feels better. The understanding of cell death and the players involved is
a subject of constant research. The better one understands the mechanism of
cell death, the more likely it is that knowledge can be integrated into clinical
medicine.
Option D: There are usually activity limitations after cellular injury. Chemotherapy
treatments with radiation can manipulate these pathways more directly by
causing DNA damage that drives the cell to apoptosis. Understanding the basics
of cell death allows for a better understanding of how tumor cells may evade
death and counter-evade clinically.

Nurse Katee is caring for Adam, a 22-year-old client, in a long-term facility. Which nursing
intervention would be appropriate when identifying nursing interventions aimed at
promoting and preventing contractures? Select all that apply.
A. Clustering activities to allow uninterrupted periods of rest.
B. Maintaining correct body alignment at all times.
C. Monitoring intake and output, using a urometer if necessary.
D. Using a footboard or pillows to keep feet in the correct position.
E. Performing active and passive range-of-motion exercises.
F. Weighing the client daily at the same time and in the same clothes.

Correct Answers: B, D, & E


Correct body alignment, preventing foot drop, and range-of-motion exercises will help
prevent contractures. Clustering activities will help promote adequate rest. Monitoring
intake and output and weighing the client will help maintain fluid and electrolyte balance.

Option A: Provide the patient with rest periods in between activities. Consider
energy-saving techniques. Rest periods are essential to conserve energy. The
patient must learn and accept his/her limitations.
Option B: Help the patient develop sitting balance and standing balance. This
helps out in retraining neural pathways, promoting proprioception and motor
response. Keep limbs in functional alignment with one or more of the following:
pillows, sandbags, wedges, or prefabricated splints.
Option C: Assess input and output record and nutritional pattern. Pressure ulcers
build up more rapidly in patients with nutritional insufficiency. Encourage a diet
high in fiber and liquid intake of 2000 to 3000 ml per day unless contraindicated.
Option D: Use a footboard or pillows to keep the feet in the correct position. This
avoids foot drop and too much plantar flexion or tightness. Maintain feet in a
dorsiflexed position.
Option E: Assess the strength to perform ROM to all joints. This assessment
provides data on the extent of any physical problems and guides therapy. Testing
by a physical therapist may be needed. Execute passive or active assistive ROM
exercises to all extremities.
A 36-year-old male client is about to be discharged from the hospital after 5 days due to
surgery. Which intervention should be included in the home health care nurse’s
instructions about measures to prevent constipation?
A. Discouraging the client from eating large amounts of roughage-containing
foods in the diet.
B. Encouraging the client to use laxatives routinely to ensure adequate bowel
elimination.
C. Instructing the client to establish a bowel evacuation schedule that changes
every day.
D. Instructing the client to fill a 2-L bottle with water every night and drink
it the next day.

Correct Answer: D. Instructing the client to fill a 2-L bottle with water every night and
drink it the next day.
Adequate fluids and fiber in the diet are key to preventing constipation. Having the client
fill a 2-L bottle with water every night and drink it the next day is one method for ensuring
the client receives at least 2,000 ml of water daily. The client also should be instructed to
drink any other fluids throughout the day.

Option A: High fiber or roughage foods are encouraged. Assist the patient to take
at least 20 g of dietary fiber (e.g., raw fruits, fresh vegetables, whole grains) per
day. Fiber adds bulk to the stool and makes defecation easier because it passes
through the intestine essentially unchanged.
Option B: Laxatives should not be used routinely for bowel elimination. They
should be used only as a last resort, because clients may become dependent on
them. The use of laxatives or enemas is indicated for the short-term management
of constipation.
Option C: A regular bowel evacuation schedule should be established.
Encourage a regular period for elimination. Most people defecate following the
first daily meal or coffee, as a result of the gastrocolic reflex.

Mr. McPartlin suffered abrasions and lacerations after a vehicular accident. He was
hospitalized and was treated for a couple of weeks. When planning care for a client with
cellular injury, the nurse should consider which scientific rationale?
A. Nutritional needs remain unchanged for the well-nourished adult.
B. Age is an insignificant factor in cellular repair.
C. The presence of infection may slow the healing process.
D. Tissue with inadequate blood supply may heal faster.

Correct Answer: C. The presence of infection may slow the healing process.
Infection impairs wound healing. Adequate blood supply is essential for healing. If
inadequate, healing is slowed. Simplistically, cell injury disrupts cellular homeostasis.
Cells are injured by numerous and diverse causes (etiologic agents) from intrinsic and
extrinsic sources; however, all of these causes and they number in the thousands,
activate one or more of four final common biochemical mechanisms leading to cell
injury.
Option A: Nutritional needs, including protein and caloric needs, increase for all
clients undergoing cellular repair because adequate protein and caloric intake is
essential to optimal cellular repair. Nutritional deficiencies, excesses, and
imbalances all predispose the cell to injury.
Option B: Elderly clients may have decreased blood flow to the skin, organ
atrophy and diminished function, and altered immunity. These conditions slow
cellular repair and increase the risk of infection. Cells and tissues age because of
accumulated damage to their proteins, lipids, and nucleic acids. Much of the
damage of aging is attributed to ROS, DNA mutations, and cellular senescence
Option D: Anything that decreases the supply of oxygen and other nutrients to
the cell or that damages mitochondria directly halts oxidative phosphorylation,
leading to rapid depletion of ATP, even in those cells that can switch to anaerobic
glycolysis. The ATP depletion results in additional cell damage by causing failure
of energy-dependent enzymes, in particular, the cell membrane adenosine
triphosphatase ion pumps that control cell volume and electrolyte balance.

A 22-year-old lady is displaying facial grimaces during her treatment in the hospital due
to burn trauma. Which nursing intervention should be included for reducing pain due to
cellular injury?
A. Administering anti-inflammatory agents as prescribed.
B. Elevating the injured area to decrease venous return to the heart.
C. Keeping the skin clean and dry.
D. Applying warm packs initially to reduce edema.

Correct Answer: A. Administering anti-inflammatory agents as prescribed


Anti-inflammatory agents help reduce edema and relieve pressure on nerve endings,
subsequently reducing pain. The burned patient may require around-the-clock
medication and dose titration. IV method is often used initially to maximize drug effect.
Option B: Elevating the injured area increases venous return to the heart.
Elevation may be required initially to reduce edema formation; thereafter,
changes in position and elevation reduce discomfort and risk of joint
contractures.
Option C: Maintaining clean, dry skin aids in preventing skin breakdown. Cover
wounds as soon as possible unless an open-air exposure burn care method is
required. Temperature changes and air movement can cause great pain to
exposed nerve endings.
Option D: Cool packs, not warm packs, should be used initially to cause
vasoconstriction and reduce edema. Altered tissue perfusion and edema
formation impair drug absorption. Injections into potential donor sites may render
them unusable because of hematoma formation.
Lisa, a client with altered urinary function, is under the care of nurse Tine. Which
intervention is appropriate to include when developing a plan of care for Lisa who is
experiencing urinary dribbling?
A. Inserting an indwelling Foley catheter.
B. Having the client perform Kegel exercises.
C. Keeping the skin clean and dry.
D. Using pads or diapers on the client.

Correct Answer: B. Having the client perform Kegel exercises.


Kegel exercises, which help strengthen the muscles in the perineal area, are used to
maintain urinary continence. To perform these exercises, the client tightens pelvic floor
muscles for 4 seconds 10 times at least 20 times each day, stopping and starting the
urinary flow.
Option A: Inserting an indwelling Foley catheter increases the risk for infection
and should be avoided. Begin bladder retraining per protocol when appropriate
(fluids between certain hours, digital stimulation of trigger area, contraction of
abdominal muscles, Credé’s maneuver).
Option C: Proper perineal hygiene decreases the risk of skin irritation or
breakdown and the development of ascending infection. The nurse should
encourage the client to develop a toileting schedule based on normal urinary
habits. However, suggesting bathroom use every 8 hours may be too long an
interval to wait.
Option D: Pads or diapers should be used only as a resort. Refer to the urinary
continence specialist as indicated. Collaboration with specialists is helpful for
developing an individual plan of care to meet a patient’s specific needs using the
latest techniques, continence products.

Jeron is admitted to the hospital due to bacterial pneumonia. He is febrile, diaphoretic,


and has shortness of breath and asthma. Which goal is the most important for the client?
A. Prevention of fluid volume excess
B. Maintenance of adequate oxygenation
C. Education about infection prevention
D. Pain reduction

Correct Answer: B. Maintenance of adequate oxygenation


For the client with asthma and infection, oxygenation is the priority. Maintaining
adequate oxygenation reduces the risk of physiologic injury from cellular hypoxia, which
is the leading cause of cell death. The purpose of oxygen therapy is to maintain PaO2
above 60 mmHg. Oxygen is administered by the method that provides appropriate
delivery within the patient’s tolerance.
Option A: A fluid volume deficit resulting from fever and diaphoresis, not excess,
is more likely for this client. Assess respirations: note quality, rate, rhythm, depth,
use of accessory muscles, ease, and position assumed for easy breathing.
Option C: Teaching about infection control is not appropriate at this time but
would be appropriate before discharge. Observe the color of skin, mucous
membranes, and nail beds, noting the presence of peripheral cyanosis (nail
beds) or central cyanosis (circumoral).
Option D: No information regarding pain is provided in this scenario. Monitor
body temperature, as indicated. Assist with comfort measures to reduce fever
and chills: addition or removal of bedcovers, comfortable room temperature, tepid
or cool water sponge bath.

Rogelio, a 32-year-old patient, is about to be discharged from the acute care setting.
Which nursing intervention is the most important to include in the plan of care?
A. Stress-reduction techniques
B. Home environment evaluation
C. Skin-care measures
D. Participation in activities of daily living

Correct Answer: B. Home environment evaluation


After discharge, the client is responsible for his own care and health maintenance
management. Discharge includes assessing the home environment for determining the
client’s ability to maintain his health at home. All instructions for care at home, including
medications, diet, therapy, and follow-up appointments, must be explained in detail to all
patients and then presented in written form to take home upon discharge.
Option A: The discharge plan begins at the admission time and includes the
patients and their families’ needs prediction, and a plan to fulfill their
requirements after discharge from the hospital. A practical discharge plan helps
to provide continuous care with the least amount of stress for patients.
Option C: Nurses, as key members of the treatment team, play a critical role in
training and taking care of the patients. One of the most basic nursing
responsibilities is to provide continuous care. In this regard, the inclusion of a
discharge plan for all admitted patients could be a symbol of such care.
Option D: Basically, any significant change or poor performance requires
physical, social, and psychological adjustment. Patients are concerned about
their discharge and are preoccupied with their ability in performing their own
duties and the way to handle themselves as well as joining the family. Therefore,
self-care training is of utmost importance for the patients and their families.

Mrs. dela Riva is in her first trimester of pregnancy. She has been lying all day because
her OB-GYN requested her to have a complete bed rest. Which nursing intervention is
appropriate when addressing the client’s need to maintain skin integrity?
A. Monitoring intake and output accurately.
B. Instructing the client to cough and deep breathe every 2 hours.
C. Keeping the linens dry and wrinkle-free.
D. Using a footboard to maintain correct anatomic position.

Correct Answer: C. Keeping the linens dry and wrinkle-free.


Keeping the linens dry and wrinkle-free aids in preventing moisture and pressure from
interfering with adequate blood supply to the tissues, helping to maintain skin integrity.
Encourage the implementation of a turning schedule, restricting time in one position to 2
hours or less, if the patient is restricted to bed.
Option A: Monitoring intake and output aids in assessing and maintaining bladder
function. Assess patient’s nutritional status, including weight, weight loss, and
serum albumin levels. An albumin level less than 2.5 g/dL is a grave sign,
indicating severe protein depletion and at high risk of skin breakdown.
Option B: Coughing and deep breathing help promote gas exchange. Reinforce
the importance of turning, mobility, and ambulation. These will enhance their
sense of efficacy and can improve compliance with the prescribed interventions.
Option D: Using a footboard is appropriate for maintaining a normal body function
position. Encourage the patient to change position every 15 minutes and change
chair-bound positions every hour. Use pillows or foam wedges to keep bony
prominences from direct contact with each other. Keep pillows under the heels to
raise off the bed.

Maya, who is admitted to a hospital, is scheduled to have her general checkup and
physical assessment. Nurse Timothy observed a reddened area over her left hip. Which
should the nurse do first?
A. Massage the reddened area for a few minutes.
B. Notify the physician immediately.
C. Arrange for a pressure-relieving device.
D. Turn the client to the right side for 2 hours.

Correct Answer: D. Turn the client to the right side for 2 hours
Turning the client to the right side relieves the pressure and promotes adequate blood
supply to the left hip. Encourage the patient to change position every 15 minutes and
change chair-bound positions every hour. During sitting, the pressure over the sacrum
may exceed 100 mm Hg. The pressure needed to close capillaries is around 32 mm Hg;
any pressure above 32 mm Hg leads to ischemia.
Option A: A reddened area is never massaged, because this may increase the
damage to the already reddened, damaged area. Massage only around the
affected area. This is to increase tissue perfusion. Massaging the actual
reddened area may damage the skin further.
Option B: The health care provider does not need to be notified immediately.
However, the health care provider should be informed of this finding the next time
he is on the unit. Educate the patient and caregiver about the causes of pressure.
This information can assist the patient or caregiver in finding methods to prevent
skin breakdown.
Option C: Arranging for a pressure-relieving device is appropriate, but this is
done after the client has been turned. Use pillows or foam wedges to keep bony
prominences from direct contact with each other. Keep pillows under the heels to
raise off bed. These measures reduce shearing forces on the skin.

Pierro was noted to be displaying facial grimaces after nurse Kara assessed his
complaints of pain rated as 8 on a scale of 1 (no pain) 10 10 (worst pain). Which
intervention should the nurse do?
A. Administering the client's ordered pain medication immediately.
B. Using guided imagery instead of administering pain medication.
C. Using therapeutic conversation to try to discourage pain medication.
D. Attempting to rule out complications before administering pain
medication.

Correct Answer: D. Attempting to rule out complications before administering pain


medication.
When intervening with a client complaining of pain, the nurse must always determine if
the pain is expected pain or a complication that requires immediate nursing intervention.
This must be done before administering the medication. Perform a comprehensive
assessment of pain. Determine via assessment the location, characteristics, onset,
duration, frequency, quality, and severity of pain.
Option A: Perform a history assessment of pain. Additionally, the nurse should
ask the following questions during pain assessment to determine its history: (1)
effectiveness of previous pain treatment or management; (2) what medications
were taken and when; (3) other medications being taken; (4) allergies or known
side effects to medications.
Option B: Guided imagery should be used along with, not instead of,
administration of pain medication. Guided imagery involves the use of mental
pictures or guiding the patient to imagine an event to distract from the pain.
Option C: The nurse should medicate the client and not discourage medication.
Nurses have the duty to ask their clients about their pain and believe their reports
of pain. Challenging or undermining their pain reports results in an unhealthy
therapeutic relationship that may hinder pain management and deteriorate
rapport.

Nurse Martha is teaching her students about bacterial control. Which intervention is the
most important factor in preventing the spread of microorganisms?
A. Maintenance of asepsis with indwelling catheter insertion.
B. Use of masks, gowns, and gloves when caring for clients with infection.
C. Correct handwashing technique.
D. Cleanup of blood spills with sodium hydrochloride.

Correct Answer: C. Correct handwashing technique.


Handwashing remains the most effective procedure for controlling microorganisms and
the incidence of nosocomial infections. According to the Centers for Disease Control and
Prevention (CDC), hand hygiene is the single most important practice in the reduction of
the transmission of infection in the healthcare setting. According to the CDC, hand
hygiene encompasses the cleansing of your hands with soap and water, antiseptic hand
washes, antiseptic hand rubs such as alcohol-based hand sanitizers, foams or gels, or
surgical hand antisepsis.
Option A: Aseptic technique is essential with invasive procedures, including
indwelling catheters. The purpose of creating a sterile field is to reduce the
number of microbes present to as few as possible. The sterile field is used in
many situations outside the operating room as well as inside the operating room
when performing surgical cases.
Option B: Masks, gowns, and gloves are necessary only when the likelihood of
exposure to blood or body fluids is high. Personal protective equipment serves as
a barrier to protect the skin, mucous membranes, airway, and clothing. It includes
gowns, gloves, masks, and face shields or goggles.
Option D: Spills of blood from clients with acquired immunodeficiency syndrome
should be cleaned with sodium hydrochloride. Standard precautions apply to the
care of all patients, irrespective of their disease state. These precautions apply
when there is a risk of potential exposure to blood; all body fluids, secretions, and
excretions, except sweat, regardless of whether or not they contain visible blood;
non-intact skin, and mucous membranes.

A patient with tented skin turgor, dry mucous membranes, and decreased urinary output
is under nurse Mark’s care. Which nursing intervention should be included in the care
plan of Mark for his patient?
A. Administering I.V. and oral fluids.
B. Clustering necessary activities throughout the day.
C. Assessing color, odor, and amount of sputum.
D. Monitoring serum albumin and total protein levels.

Correct Answer: A. Administering I.V. and oral fluids


The client’s assessment findings would lead the nurse to suspect that the client is
dehydrated. Administering I.V. fluids is appropriate. Administer parenteral fluids as
prescribed. Consider the need for an IV fluid challenge with immediate infusion of fluids
for patients with abnormal vital signs. Fluids are necessary to maintain hydration status.
Determination of the type and amount of fluid to be replaced and infusion rates will vary
depending on clinical status.
Option B: Clustering activities help with energy conservation and promote rest.
Aid the patient if he or she is unable to eat without assistance, and encourage the
family or SO to assist with feedings, as necessary. Dehydrated patients may be
weak and unable to meet prescribed intake independently.
Option C: Assessing sputum would be appropriate for a client with problems
associated with impaired gas exchange or ineffective airway clearance. Assess
skin turgor and oral mucous membranes for signs of dehydration. Signs of
dehydration are also detected through the skin. Skin of elderly patients loses
elasticity, hence skin turgor should be assessed over the sternum or on the inner
thighs. Longitudinal furrows may be noted along the tongue.
Option D: Monitoring albumin and protein levels are appropriate for clients
experiencing inadequate nutrition. Monitor and document hemodynamic status
including CVP, pulmonary artery pressure (PAP), and pulmonary capillary wedge
pressure (PCWP) if available in a hospital setting. These direct measurements
serve as an optimal guide for therapy.

Khaleesi is admitted to the hospital due to having a lower than normal potassium level in
her bloodstream. Her medical history reveals vomiting and diarrhea prior to
hospitalization. Which foods should the nurse instruct the client to increase?
A. Whole grains and nuts
B. Milk products and green, leafy vegetables
C. Pork products and canned vegetables
D. Orange juice and bananas

Correct Answer: D. Orange juice and bananas


The client with hypokalemia needs to increase the intake of foods high in potassium.
Orange juice and bananas are high in potassium, along with raisins, apricots, avocados,
beans, and potatoes. Encourage high potassium diet such as oranges, bananas,
tomatoes, coffee, red meat, and dried fruits. Discuss the use of potassium chloride salt
substitutes for a client receiving long-term diuretics. Potassium may be replaced and
level maintained through the diet when the client is allowed oral food and fluids.
Option A: Whole grains and nuts would be encouraged for the client with
hypomagnesemia. Encourage intake of dairy products, meat, fish, green leafy
vegetables, and whole grains. Provides an oral replacement for mild magnesium
deficits; may prevent a recurrence.
Option B: Milk products and green, leafy vegetables are good sources of
calcium for the client with hypocalcemia. Encourage the client to eat foods high in
calcium such as dark leafy greens, cheese, low-fat milk, yogurt, eggs, oranges,
green beans, and sardines. Avoid intake of phosphorus-rich foods such as bran,
chocolates, nuts, whole wheat, and barley.
Option C: Pork products and canned vegetables are high in sodium and are
encouraged for the client with hyponatremia. Encourage fluids and foods high in
sodium such as meat, milk, beets, celery, eggs, and carrots. Use fruit juices and
bouillon instead of water. Unless sodium deficit causes serious symptoms
requiring immediate IV replacement, the client may benefit from slower
replacement by oral method or removal of previous salt restriction.
Mary Jean, a first year nursing student, was rushed to the clinic department due to
hyperventilation. Which nursing intervention is the most appropriate for the client who is
subsequently developing respiratory alkalosis?
A. Administering sodium chloride I.V.
B. Encouraging slow, deep breaths.
C. Preparing to administer sodium bicarbonate.
D. Administer low-flow oxygen therapy.

Correct Answer: B. Encouraging slow, deep breaths.


The client who is hyperventilating and subsequently develops respiratory alkalosis is
losing too much carbon dioxide. Measures that result in the retention of carbon dioxide
are needed. Encourage slow, deep breathing to retain carbon dioxide and reverse
respiratory alkalosis. Encourage the patient to breathe slowly and deeply. Speak in a low,
calm tone of voice. Provide a safe environment. May help reassure and calm the agitated
patient, thereby aiding the reduction of respiratory rate. Assists the patient to regain
control.
Option A: Administering sodium chloride is appropriate for metabolic alkalosis.
Demonstrate appropriate breathing patterns, if appropriate, and assist with
respiratory aids or a rebreathing mask/bag. Decreasing the rate of respiration can
halt the “blowing off” of CO2, elevating Pco2 level and normalizing pH.
Option C: Administering sodium bicarbonate is appropriate for treating metabolic
acidosis. Provide comfort measures; encourage the use of meditation and
visualization. Use a tepid sponge bath/cool cloths. Promotes relaxation and
reduces stress. Control and reduction of fever reduce the potential for seizures
and helps reduce respiration rate.
Option D: Administering low-flow oxygen therapy is appropriate for chronic
respiratory acidosis. Administer CO2, or use a rebreathing mask as indicated.
Reduce respiratory rate and tidal volume, or add additional dead space (tubing)
to a mechanical ventilator.

Nurse John Joseph is totaling the intake and output for Elena Reyes, a client diagnosed
with septicemia who is on a clear liquid diet. The client intakes 8 oz of apple juice, 850 ml
of water, 2 cups of beef broth, and 900 ml of half-normal saline solution and outputs
1,500 ml of urine during the shift. How many milliliters should the nurse document as the
client’s intake? Fill in the blank and write your answer as a whole number.
Answer: (2470) mL.

Correct answer: 2470 mL.


Rationale:
The fluid intake includes 8 oz (240 ml) of apple juice, 850 ml of water, 2 cups (480 ml) of
beef broth, and 900 ml of I.V. fluid for a total of 2,470 ml intake for the shift. Monitoring of
intake helps caregivers ensure that the patient has a proper intake of fluid and other
nutrients. Monitoring of output helps determine whether there is adequate output of urine
as well as normal defecation.

Marie Joy’s lab test revealed that her serum calcium is 2.5 mEq/L. Which assessment
data does the nurse document when a client diagnosed with hypocalcemia develops a
carpopedal spasm after the blood-pressure cuff is inflated?
A. Positive Trousseau's sign
B. Positive Chvostek's sign
C. Tetany
D. Paresthesia

Correct Answer: A. Positive Trousseau’s sign


In a client with hypocalcemia, a positive Trousseau’s sign refers to carpopedal spasm
that develops usually within 2 to 5 minutes after applying and inflating a blood pressure
cuff to about 20 mm Hg higher than systolic pressure on the upper arm. This spasm
occurs as the blood supply to the ulnar nerve is obstructed.
Option B: Chvostek’s sign refers to twitching of the facial nerve when
tapping below the earlobe. In the late 1800s, Dr. Chvostek noticed that
mechanical stimulation of the facial nerve (as with the fingertip of the examiner,
for example) could lead to twitching of the ipsilateral facial muscles. The
long-accepted explanation is that this resulted from hypocalcemia, and this
relationship became known as the Chvostek sign.
Option C: Tetany is a clinical manifestation of hypocalcemia denoted by tingling in
the tips of the fingers around the mouth and muscle spasms in the extremities
and face. Tetany is generally induced by a rapid decline in serum ionized
calcium. Tetany is usually most dangerous and most commonly seen in the
presence of respiratory alkalosis causing hypocalcemia.
Option D: Paresthesia refers to numbness or tingling. Paresthesia is an abnormal
sensation of the skin (tingling, pricking, chilling, burning, numbness) with no
apparent physical cause. Paresthesia may be transient or chronic and may have
any of dozens of possible underlying causes.

Lab tests revealed that patient Z’s [Na+] is 170 mEq/L. Which clinical manifestation would
nurse Natty expect to assess?
A. Tented skin turgor and thirst
B. Muscle twitching and tetany
C. Fruity breath and Kussmaul's respirations
D. Muscle weakness and paresthesia

Correct Answer: A. Tented skin turgor and thirst


Hypernatremia refers to elevated serum sodium levels, usually above 145 mEq/L.
Typically, the client exhibits tented skin turgor and thirst in conjunction with dry, sticky
mucous membranes, lethargy, and restlessness. Most patients present with symptoms
suggestive of fluid loss and clinical signs of dehydration. Symptoms and signs of
hypernatremia are secondary to central nervous system dysfunction and are seen when
serum sodium rises rapidly or is greater than 160 meq/L.
Option B: Muscle twitching and tetany may be seen with hypercalcemia or
hyperphosphatemia. CNS features include delirium, coma, seizures,
neuromuscular hyperexcitability, (Chvostek’s sign and Trousseau’s
phenomenon), hyperreflexia, muscle cramping (e.g., carpopedal spasm), or
tetany.
Option C: Fruity breath and Kussmaul’s respirations are associated with diabetic
ketoacidosis. Kussmaul breathing, which is labored, deep, and tachypneic, may
occur. Some providers may appreciate a fruity scent to the patient’s breath,
indicative of the presence of acetone. Patients may have signs of dehydration,
including poor capillary refill, skin turgor, and dry mucous membranes.
Option D: Muscle weakness and paresthesia are associated with hypokalemia.
Significant muscle weakness occurs at serum potassium levels below 2.5 mmol/L
but can occur at higher levels if the onset is acute. Similar to the weakness
associated with hyperkalemia, the pattern is ascending in nature affecting the
lower extremities, progressing to involve the trunk and upper extremities, and
potentially advancing to paralysis.

Mang Teban has a history of chronic obstructive pulmonary disease and has the following
arterial blood gas results: partial pressure of oxygen (PO2), 55 mm Hg, and partial pressure of
carbon dioxide (PCO2), 60 mm Hg. When attempting to improve the client’s blood gas values
through improved ventilation and oxygen therapy, which is the client’s primary stimulus for
breathing?
A. High PCO2
B. Low PO2
C. Normal pH
D. Normal bicarbonate (HCO3)

Correct Answer: B. Low PO2


A chronically elevated PCO2 level (above 50 mmHg) is associated with inadequate
response of the respiratory center to plasma carbon dioxide. The major stimulus to
breathing then becomes hypoxia (low PO2). High PCO2 and normal pH and HCO3 levels
would not be the primary stimulus for breathing in this client.
Option A: The inability to fully exhale also causes elevations in carbon dioxide
(CO2) levels. As the disease progresses, impairment of gas exchange is often
seen. The reduction in ventilation or increase in physiologic dead space leads to
CO2 retention. Pulmonary hypertension may occur due to diffuse
vasoconstriction from hypoxemia.
Option C: An acid-base disturbance arises when arterial pH lies outside that
range. If pH is less than 7.35 an acidosis is present, if pH is greater than 7.45 the
alkalosis is present. Tight control on blood pH is achieved by a combination of
blood buffers and the respiratory and renal systems which make adjustments to
return pH toward its normal levels.
Option D: Acidosis can be caused by either a rise in PaCO2 or a fall in HCO3.
Alkalosis can be caused by either a fall in PaCO2 or a rise in HCO3. When the
primary change is in CO2 we name the disturbance respiratory, and when the
primary change is in bicarbonate, we name the disturbance metabolic.
1. 22. Question
A client with a very dry mouth, skin, and mucous membranes is diagnosed with
dehydration. Which intervention should the nurse perform when caring for a client
diagnosed with fluid volume deficit?
A. Assessing urinary intake and output.
B. Obtaining the client's weight weekly at different times of the day.
C. Monitoring arterial blood gas (ABG) results.
D. Maintaining I.V. therapy at the keep-vein-open rate.
2. Incorrect
Correct Answer: A. Assessing urinary intake and output.
For the client with fluid volume deficit, assessing the client’s urine output (using a
urometer if necessary) is essential to ensure an output of at least 30 ml/hour. Assess
color and amount of urine. Report urine output less than 30 ml/hr for 2 consecutive
hours. A normal urine output is considered normal not less than 30ml/hour. Concentrated
urine denotes fluid deficit.
Option B: The client should be weighed daily, not weekly, and at the same time
each day, usually in the morning. Weigh daily with the same scale, and preferably
at the same time of day. Weight is the best assessment data for possible fluid
volume imbalance. An increase of 2 lbs a week is considered normal.
Option C: Monitoring ABGs is not necessary for this client. Rather, serum
electrolyte levels would most likely be evaluated. Monitor serum electrolytes and
urine osmolality, and report abnormal values. Elevated blood urea nitrogen
suggests fluid deficit. Urine-specific gravity is likewise increased.
Option D: The client also would have an I.V. rate of at least 75 ml/hour, if not
higher, to correct the fluid volume deficit. Administer parenteral fluids as
prescribed. Consider the need for an IV fluid challenge with an immediate
infusion of fluids for patients with abnormal vital signs.
3. 23. Question
Which client situation requires the nurse to discuss the importance of avoiding foods
high in potassium?
A. A 14-year-old who is taking diuretics.
B. A 16-year-old with ileostomy.
C. A 16-year-old with metabolic acidosis.
D. An 18-year-old who has renal disease.
4. Incorrect
Correct Answer: D. An 18-year-old who has renal disease.
Clients with renal disease are predisposed to hyperkalemia and should avoid foods high
in potassium. Clients receiving diuretics, with ileostomies, or with metabolic acidosis may
be hypokalemic and should be encouraged to eat foods high in potassium. Encourage
intake of carbohydrates and fats and low potassium food such as pineapple, plums,
strawberries, carrots, cauliflower, corn, and whole grains. Reduces exogenous sources
of potassium and prevents metabolic tissue breakdown with the release of cellular
potassium.
Option A: A client receiving diuretics may be hypokalemic. Encourage high
potassium diet such as oranges, bananas, tomatoes, coffee, red meat, and dried
fruits. Discuss the use of potassium chloride salt substitutes for a client receiving
long-term diuretics.
Option B: Patients with ileostomies may have hypokalemia. Potassium may be
replaced and level maintained through the diet when the client is allowed oral
food and fluids. Dietary replacement of 40 to 60 mEq/L/day is usually sufficient if
no abnormal losses are happening.
Option C: Patients with metabolic disease may be hypokalemic. Note for signs of
metabolic alkalosis such as tachycardia, dysrhythmias, hypoventilation, tetany,
and changes in mentation. These are usually associated with hypokalemia.
5. 24. Question
Genevieve is diagnosed with hypomagnesemia, which nursing intervention would be
appropriate?
A. Instituting seizure precaution to prevent injury.
B. Instructing the client on the importance of preventing infection.
C. Avoiding the use of tight tourniquet when drawing blood.
D. Teaching the client the importance of early ambulation.
6. Incorrect
Correct Answer: A. Instituting seizure precaution to prevent injury.
Instituting seizure precaution is an appropriate intervention because the client with
hypomagnesemia is at risk for seizures. Changes in mentation or the development of
seizure activity in severe low magnesium increase the risk of client injury. Provide a quiet
environment and subdued lighting. Reduces extraneous stimuli; promotes rest.
Option B: Hypophosphatemia may produce changes in granulocytes, which
would require the nurse to instruct the client about measures to prevent infection.
Mild hypophosphatemia will not be clinically apparent. Severe hypophosphatemia
may have the clinical presence of altered mental status, neurological instability
including seizures and focal neurologic findings such as numbness or reflexive
weakness, a cardiac manifestation of possible heart failure, muscle pain, and
muscular weakness.
Option C: Avoiding the use of a tight tourniquet when drawing blood helps
prevent pseudohyperkalemia. Assess the level of consciousness and
neuromuscular function, including sensation, strength, and movement. The client
is usually conscious and alert; however, muscular paresthesia, weakness, and
flaccid paralysis may occur.
Option D: Early ambulation is recommended to reduce calcium loss from bones
during hospitalization. Encourage frequent repositioning and range-of-motion
(ROM) and/or muscle-setting exercises with caution. Promote ambulation as
tolerated. Muscle activity may reduce calcium shifting from the bones that occur
during immobilization.
7. 25. Question
Which electrolyte would the nurse identify as the major electrolyte responsible for
determining the concentration of the extracellular fluid?
A. Potassium
B. Phosphate
C. Chloride
D. Sodium
8. Correct
Correct Answer: D. Sodium
Sodium is the electrolyte whose level is the primary determinant of the extracellular fluid
concentration. Sodium a cation (e.g., positively charged ion), is the major electrolyte in
extracellular fluid. Sodium, which is an osmotically active anion, is one of the most
important electrolytes in the extracellular fluid. It is responsible for maintaining the
extracellular fluid volume, and also for regulation of the membrane potential of cells.
Sodium is exchanged along with potassium across cell membranes as part of active
transport.
Option A: Potassium (a cation) is a major electrolyte in the intracellular fluid.
Potassium is mainly an intracellular ion. The sodium-potassium adenosine
triphosphatase pump has the primary responsibility for regulating the
homeostasis between sodium and potassium, which pumps out sodium in
exchange for potassium, which moves into the cells.
Option B: Phosphate (an anion) is a major electrolyte in the intracellular fluid.
Phosphate is an essential electrolyte in the human body as it constitutes about
1% of the total body weight. In an adult, the normal serum phosphate level
ranges between 2.5 to 4.5 mg/d L. The normal serum levels of phosphate tend to
decrease with age and its highest levels i.e., 4.5 to 8.3 mg/dL are seen in infants,
about 50% higher than adults; this is because infants and children need more
phosphate for their growth and development.
Option C: Chloride, an anion (e.g., negatively charged ion), is also present in
extracellular fluid, but to a lesser extent. Chloride is an anion found
predominantly in the extracellular fluid. The kidneys predominantly regulate
serum chloride levels. Most of the chloride, which is filtered by the glomerulus, is
reabsorbed by both proximal and distal tubules (majorly by proximal tubule) by
both active and passive transport.
9. 26. Question
Jon has a potassium level of 6.5 mEq/L, which medication would nurse Wilma
anticipate?
A. Potassium supplements
B. Kayexalate
C. Calcium gluconate
D. Sodium tablets
10. Incorrect
Correct Answer: B. Kayexalate
The client’s potassium level is elevated; therefore, Kayexalate would be ordered to help
reduce the potassium level. Kayexalate is a cation-exchange resin, which can be given
orally, by nasogastric tube, or by retention enema. Potassium is drawn from the bowel
and excreted through the feces.
Option A: Because the client’s potassium level is already elevated, potassium
supplements would not be given. Patients with neuromuscular weakness,
paralysis, or ECG changes and elevated potassium of more than 5.5 mEq/L in
patients at risk for ongoing hyperkalemia, or confirmed hyperkalemia of 6.5
mEq/L should have aggressive treatment. Exogenous sources of potassium
should be immediately discontinued.
Option C: Neither calcium gluconate nor sodium tablets would address the
client’s elevated potassium level. Calcium therapy will stabilize the cardiac
response to hyperkalemia and should be initiated first in the setting of cardiac
toxicity. Calcium does not alter the serum concentration of potassium but is a
first-line therapy in hyperkalemia-related arrhythmias and ECG changes.
Option D: Sodium bicarbonate infusion may be helpful in patients with metabolic
acidosis. Bolus dosing of sodium bicarbonate is less effective. Loop or thiazide
diuretics may be helpful in enhancing potassium excretion. They may be used in
non-oliguric, volume overloaded patients but should not be used as monotherapy
in symptomatic patients.
11. 27. Question
Which clinical manifestation would lead the nurse to suspect that a client is experiencing
hypermagnesemia?
A. Muscle pain and acute rhabdomyolysis
B. Hot flushed skin and diaphoresis
C. Soft-tissue calcification and hyperreflexia
D. Increased respiratory rate and depth
12. Incorrect
Correct Answer: B. Hot, flushed skin and diaphoresis
Hypermagnesemia is manifested by hot, flushed skin and diaphoresis. The client also
may exhibit hypotension, lethargy, drowsiness, and absent deep tendon reflexes. The
most frequent symptoms and signs may include weakness, nausea, dizziness, and
confusion (less than 7.0 mg/dL). Increasing values (7 to 12 mg/dL) induce decreased
reflexes, worsening confusional state, drowsiness, bladder paralysis, flushing, headache,
and constipation.
Option A: Muscle pain and acute rhabdomyolysis are indicative of
hypophosphatemia. Mild hypophosphatemia will not be clinically apparent.
Severe hypophosphatemia may have the clinical presence of altered mental
status, neurological instability including seizures and focal neurologic findings
such as numbness or reflexive weakness, a cardiac manifestation of possible
heart failure, muscle pain, and muscular weakness.
Option C: Soft-tissue calcification and hyperreflexia are indicative of
hyperphosphatemia. Calcifications can also be present in skin, soft tissue, and
periarticular regions. Prolonged bone demineralization can lead to bone
fractures. CNS features include delirium, coma, seizures, neuromuscular
hyperexcitability, (Chvostek’s sign and Trousseau’s phenomenon), hyperreflexia,
muscle cramping (e.g., carpopedal spasm) or tetany.
Option D: Increased respiratory rate and depth are associated with metabolic
acidosis. The physical exam reveals signs unique to each cause such as dry
mucous membranes in the patient with diabetic ketoacidosis. Hyperventilation
may also be present as a compensatory respiratory alkalosis to assist with PCO2
elimination and correction of the acidemia. Compensatory reactions do not
completely correct a disturbance to the normal pH range.
13. 28. Question
Joshua is receiving furosemide and Digoxin, which laboratory data would be the most
important to assess in planning the care for the client?
A. Sodium level
B. Magnesium level
C. Potassium level
D. Calcium level
14. Incorrect
Correct Answer: C. Potassium level
Diuretics such as furosemide may deplete serum potassium, leading to hypokalemia.
When the client is also taking digoxin, the subsequent hypokalemia may potentiate the
action of digoxin, placing the client at risk for digoxin toxicity. Most cases of hypokalemia
result from gastrointestinal (GI) or renal losses. Renal potassium losses are associated
with increased mineralocorticoid-receptor stimulation such as occurs with primary
hyperreninism and primary aldosteronism.
Option A: Diuretic therapy may lead to the loss of other electrolytes such as
sodium, but the loss of potassium in association with digoxin therapy is most
important. Increased delivery of sodium and/or non-absorbable ions (diuretic
therapy, magnesium deficiency, genetic syndromes) to the distal nephron can
also result in renal potassium wasting. GI losses are a common cause of
hypokalemia with severe or chronic diarrhea being the most common extrarenal
cause of hypokalemia.
Option B: Hypomagnesemia generally is associated with poor nutrition,
alcoholism, and excessive GI or renal losses, not diuretic therapy. Magnesium
homeostasis involves the kidney (primarily through the proximal tubule, the thick
ascending loop of Henle, and the distal tubule), small bowel (primarily through
the jejunum and ileum), and bone. Hypomagnesemia occurs when something,
whether a drug or a disease condition, alters the homeostasis of magnesium.
Option D: Hypocalcemia is usually associated with inadequate vitamin D intake
or synthesis, renal failure, or the use of drugs, such as aminoglycosides and
corticosteroids. Calcitonin on the other hand lowers levels of calcium.
Hypocalcemia is a common cause of tetany and neuromuscular irritability. An
alkaline environment lowers calcium levels and induces tetany, whereas an acidic
environment is protective.
15. 29. Question
Mr. Salcedo has the following arterial blood gas (ABG) values: pH of 7.34, partial
pressure of arterial oxygen of 80 mm Hg, partial pressure of arterial carbon dioxide of 49
mm Hg, and a bicarbonate level of 24 mEq/L. Based on these results, which intervention
should the nurse implement?
A. Instructing the client to breathe slowly into a paper bag.
B. Administering low-flow oxygen.
C. Encouraging the client to cough and deep breathe.
D. Nothing, because these ABG values are within normal limits.
16. Incorrect
Correct Answer: C. Encouraging the client to cough and deep breathe.
The ABG results indicate respiratory acidosis requiring improved ventilation and
increased oxygen to the lungs. Coughing and deep breathing can accomplish this.
Encourage and assist with deep-breathing exercises, turning, and coughing. Suction as
necessary. Provide airway adjunct as indicated. Place in semi-Fowler’s position. These
measures improve lung ventilation and reduce or prevent airway obstruction associated
with the accumulation of mucus.
Option A: Breathing into a paper bag is appropriate for a client hyperventilating
and experiencing respiratory alkalosis. Provide appropriate chest physiotherapy,
including postural drainage and breathing exercises. Aids in clearing secretions,
which improves ventilation, allowing excess CO2 to be eliminated.
Option B: The nurse would administer high oxygen levels because the client does
not have chronic obstructive pulmonary disease. Administer oxygen as indicated.
Increase respiratory rate or tidal volume of the ventilator, if used. Prevents and
corrects hypoxemia and respiratory failure.
Option D: Some action is necessary because the ABG results are not within
normal limits. Monitor and graph serial ABGs, pulse oximetry readings; Hb,
serum electrolyte levels. Evaluates therapy need and effectiveness. Note:
Bedside pulse oximetry monitoring is used to show early changes in oxygenation
before other signs or symptoms are observed.
17. 30. Question
A client is diagnosed with metabolic acidosis, which would the nurse expect the health
care provider to order?
A. Potassium
B. Sodium bicarbonate
C. Serum sodium level
D. Bronchodilator
18. Incorrect
Correct Answer: B. Sodium bicarbonate
Metabolic acidosis results from excessive absorption or retention of acid or excessive
excretion of bicarbonate. A base is needed. Sodium bicarbonate is a base and is used to
treat documented metabolic acidosis. The management of metabolic acidosis should
address the cause of the underlying acid-base derangement. For example, adequate
fluid resuscitation and correction of electrolyte abnormalities are necessary for sepsis
and diabetic ketoacidosis. Potassium, serum sodium determinations, and a
bronchodilator would be inappropriate orders for this client.
Option A: The chief indication for potassium administration is potassium
deficiency or hypokalemia, a condition in which serum potassium level falls below
a critical range. Hypokalemia can occur due to multiple reasons, mainly
inadequate intake of potassium. Metabolic alkalosis can also cause hypokalemia
by shifting potassium from the extracellular to the intracellular compartment.
Option C: Among the electrolyte disorders, hyponatremia is the most frequent.
Diagnosis is when the serum sodium level less than 135 mmol/L. Hyponatremia
has neurological manifestations. Patients may present with headache, confusion,
nausea, delirium. Hypernatremia presents when the serum sodium levels greater
than145 mmol/L. Symptoms of hypernatremia include tachypnea, sleeping
difficulty, and feeling restless. Rapid sodium corrections can have serious
consequences like cerebral edema and osmotic demyelination syndrome.
Option D: Bronchodilators are indicated for individuals that have lower than
optimal airflow through the lungs. The mainstay of treatment is beta-2 agonists
that target the smooth muscles in the bronchioles of the lung. Various respiratory
conditions may require bronchodilators, including asthma and chronic obstructive
pulmonary disease.

1. 1. Question
Lee Angela’s lab test just revealed that her chloride level is 96 mEq/L. As a nurse, you
would interpret this serum chloride level as:
○ A. High
○ B. Low
○ C. Within normal range
○ D. High normal
2. Correct
Correct Answer: C. Within normal range
Normal serum concentrations of chloride range from 95 to 108 mEq/L. Chloride is an
anion found predominantly in the extracellular fluid. The kidneys predominantly regulate
serum chloride levels. Most of the chloride, which is filtered by the glomerulus, is
reabsorbed by both proximal and distal tubules (majorly by proximal tubule) by both
active and passive transport.
○ Option A: Hyperchloremia is an electrolyte disturbance in which there is an
elevated level of the chloride ions in the blood. The normal serum range for
chloride is 95 to 108 mEq/L, therefore chloride levels at or above 110 mEq/L
usually indicate kidney dysfunction as it is a regulator of chloride concentration.
○ Option B: The most reduced levels of serum chloride (range 45 to 70 mEq/L) are
associated with pernicious forms of vomiting due to gastric outlet obstruction,
protracted vomiting in alcoholics, or self-induced vomiting. Individuals with
hypochloremia secondary to total body chloride depletion will have physical
findings that indicate ECF volume contraction (e.g., hypotension, tachycardia,
and orthostatic changes in blood pressure).
○ Option D: Conditions causing an elevation of the serum chloride concentration
and a concomitant elevation of the serum sodium concentration result primarily
from disorders associated with loss of electrolyte-free fluids (pure water loss);
hypotonic fluids (water deficit in excess of sodium and chloride deficits); or
administration of NaCl-containing fluids.
3. 2. Question
Which of the following conditions is associated with elevated serum chloride levels?
○ A. Cystitis
○ B. Diabetes
○ C. Eclampsia
○ D. Hypertension
4. Correct
Correct Answer: C. Eclampsia
Eclampsia is associated with increased levels of serum chloride. Metabolic alkalosis is
uncommon in pregnancy and is most often the result of severe vomiting. If this is present
at the time of delivery, transient metabolic derangement in the fetus can occur,
potentially requiring additional organ support.
○ Option A: Urinalysis, when indicated, is the most important laboratory test in the
diagnosis of UTI. Pyuria, which is the presence of at least 10 WBCs or
leukocytes in an unspun midstream urine specimen, is almost always present.
They detect the presence of leukocyte esterase, an enzyme produced by
leukocytes, and nitrite, which is indicative of the presence of Enterobacteriaceae.
○ Option B: Present study showed the importance of measuring serum electrolytes
in patients with type 2 diabetes mellitus. As fasting blood glucose rises,
electrolytes mainly sodium, chloride, and potassium become more deranged
significantly. Also, raised fasting blood glucose worsens renal function, as shown
by an increase in microalbumin levels in urine.
○ Option D: Among the environmental factors that affect blood pressure, dietary
sodium chloride has been studied the most, and there is general consensus that
increased sodium chloride intake increases blood pressure. The role for NaCl is
supported by insights from the pressure-natriuresis mechanism, monogenic
forms of hypertension, and dietary salt reduction studies.
5. 3. Question
In the extracellular fluid, chloride is a major:
○ A. Compound
○ B. Ion
○ C. Anion
○ D. Cation
6. Correct
Correct Answer: C. Anion
Chloride is a major anion found in the extracellular fluid. Chloride is an inorganic anionic
halogen with an atomic weight of 35.5. It is distributed exclusively within the extracellular
fluid compartment (ECF), which comprises the blood/plasma (or serum) compartment
and the interstitial fluid compartment. Chloride is the major anion associated with sodium
in the ECF.
○ Option A: A compound occurs when two ions are bound together. When two
distinct elements are chemically combined—i.e., chemical bonds form between
their atoms—the result is called a chemical compound. Most elements on Earth
bond with other elements to form chemical compounds, such as sodium (Na) and
Chloride (Cl), which combine to form table salt (NaCl).
○ Option B: Chloride is an ion, but this choice is too general. Ions are formed when
the number of protons in an atom does not equal the number of electrons. If more
protons are present, the ion is positive and is known as a cation; if more
electrons are present, the ion is negative and referred to as an anion.
○ Option D: HCO3 is a cation. Cations are positively charged ions. They are formed
when a metal loses its electrons. They lose one or more than one electron and
do not lose any protons. Therefore, they possess a net positive charge.
7. 4. Question
Nursing intervention for the patient with hyperphosphatemia includes encouraging intake
of:
○ A. Vitamin D
○ B. Fleets phospho-soda
○ C. Milk
○ D. Amphojel
8. Incorrect
Correct Answer: D. Amphogel
Administration of phosphate binders (amphojel and basagel) will reduce the serum
phosphate levels. Aluminum hydroxide (brand names: Alternagel®, Amphojel®) is an
over-the-counter oral antacid and phosphate binder, most commonly used to treat high
phosphate levels secondary to kidney dysfunction (abnormal or impaired function of the
kidneys). It can also be used to reduce stomach acid production.
○ Option A: Vitamin D intoxication can produce hyperphosphatemia as a result of
excessive gastrointestinal absorption and increased renal reabsorption. Reports
indicate that the excessive use of phosphate-containing laxatives or enemas can
also produce hyperphosphatemia.
○ Option B: This medicine is used as part of a bowel cleansing procedure before an
x-ray of the bowel, colonoscopy (looking into the bowel with an instrument), or
before a bowel operation. It works by producing bowel motions that cleanse the
bowel.
○ Option C: Avoid or limit milk and dairy products, like cheese, yogurt, pudding, and
ice cream. Try flavored ice pops or sorbet instead of ice cream. Use non-dairy
creamers, soy beverages, or rice milk to replace milk. Make sure the brands are
low in phosphorus.
9. 5. Question
Etiologies associated with hypocalcemia may include all of the following except:
○ A. Renal failure
○ B. Inadequate intake calcium
○ C. Metastatic bone lesions
○ D. Vitamin D deficiency
10. Incorrect
Correct Answer: C. Metastatic bone lesions
Metastatic bone lesions are associated with hypercalcemia due to accelerated bone
metabolism and release of calcium into the serum. Although more common in adults
than pediatric patients, the next important etiology to consider is malignancy. Renal
carcinomas, leukemias, lymphomas, and rhabdomyosarcoma can be associated with
elevated calcium levels mediated by the action of PTH-related peptides. Renal failure,
inadequate calcium intake, and vitamin D deficiency may cause hypocalcemia.
○ Option A: CKD leads to impaired phosphate excretion which drives PTH
secretion and can cause secondary hyperparathyroidism. However, due to
impaired Vitamin D metabolism and high phosphorus level, the serum calcium
remains low despite the high PTH.
○ Option B: Serum calcium is normally bound to proteins in the blood most
prominently albumin and therefore low albumin states can give a falsely low total
serum calcium level. Ionized calcium level is usually normal in these states and
thus a correction of adding 0.8 mg/dL to serum calcium level is usually
recommended for every 1gm drop in serum albumin below normal (4 gm/dL)
○ Option D: Absolute or relative Vitamin D deficiency includes lack of active
metabolite of vitamin D due to inadequate sun exposure or liver disease or
kidney disease. Also, included in this category are familial causes of vitamin D
resistance.
11. 6. Question
Which of the following findings would the nurse expect to assess in hypercalcemia?
○ A. Prolonged QRS complex
○ B. Tetany
○ C. Petechiae
○ D. Urinary calculi
12. Correct
Correct Answer: D. Urinary calculi
Urinary calculi may occur with hypercalcemia. Symptoms of hypercalcemia are usually
seen when serum calcium levels are more than 12 mg/dl. Irrespective of the etiology, the
broad signs and symptoms can be summarized as “groans, bones, stones, moans,
thrones and psychic overtones”. Tetany and petechiae are signs of hypocalcemia.
○ Option A: Shortened, not prolonged QRS complex would be seen in
hypercalcemia. Severe hypercalcemia inhibits neuromuscular and myocardial
depolarization leading to muscle weakness and arrhythmias. Cardiovascular
effects include prolonged PR interval, short QT interval, widened QRS complex,
and bradycardia.
○ Option B: Generally induced by a rapid decline in serum ionized calcium. Tetany
is usually most dangerous and most commonly seen in the presence of
respiratory alkalosis causing hypocalcemia. Hypocalcemia is a common cause of
tetany and neuromuscular irritability. An alkaline environment lowers calcium
levels and induces tetany, whereas an acidic environment is protective.
○ Option C: Parathyroid hormone enhances osteoclastic bone resorption and distal
tubular reabsorption of calcium. In addition, it mediates the absorption of calcium
from the intestine. Vitamin D is known to regulate PTH release, intestinal
absorption of calcium, and also medicare’s PTH stimulated bone reabsorption.
13. 7. Question
Which of the following is not an appropriate nursing intervention for a patient with
hypercalcemia?
○ A. Administering calcitonin
○ B. Administering calcium gluconate
○ C. Administering loop diuretics
○ D. Encouraging ambulation
14. Correct
Correct Answer: B. Administering calcium gluconate
Calcium gluconate is used for replacement in deficiency states. Calcium gluconate,
gluceptate, or chloride (IV) provides rapid treatment in acute calcium deficit, especially in
the presence of tetany or convulsions. Calcitonin and loop diuretics are used to lower
serum calcium.
○ Option A: Calcitonin can be administered subcutaneously but in most cases, the
effects are mild and limited to a few days. Promotes movement of serum calcium
into bones, temporarily reducing serum calcium levels, especially in the presence
of the increased parathyroid hormone.
○ Option C: Loop diuretics should be used with caution as even though they may
enhance renal excretion, paradoxical hypercalcemia can occur due to bone
resorption. Diuresis promotes renal excretion of calcium and reduces risks of fluid
excess from an isotonic saline infusion.
○ Option D: Hypercalcemia of immobilization can be prevented by encouraging
activity as tolerated and adequate hydration. The specific cause of hypercalcemia
needs to be identified, and treatment directed accordingly.
15. 8. Question
A patient in which of the following disorders is at high risk to develop hypermagnesemia?
○ A. Insulin shock
○ B. Hyperadrenalism
○ C. Nausea and vomiting
○ D. Renal failure
16. Incorrect
Correct Answer: D. Renal failure
Renal failure can reduce magnesium excretion, leading to hypermagnesemia.
Hypermagnesemia refers to an excess amount of magnesium in the bloodstream. It is
rare and is usually caused by renal failure or poor kidney function. Magnesium is a
mineral the body uses as an electrolyte, meaning it carries electric charges around the
body when dissolved in the blood.
○ Option A: Diabetic ketoacidosis, not insulin shock is a cause of
hypermagnesemia. In DKA body magnesium deficits through urinary losses are
routinely encountered and are the consequence of absence of insulin. However,
magnesium exit from the cells may cause hypermagnesemia, which is frequent at
presentation with DKA.
○ Option B: Hypoadrenalism, not hyperadrenalism is a cause of hypermagnesemia.
Hypothyroidism and especially cortico-adrenal insufficiency, are other recognized
causes. Hypomagnesemia is quite frequent, although the symptomatology
(cramps, muscle spasms, paresthesia, and arrhythmias) appears only when
exceeding the critical value.
○ Option C: Nausea and vomiting lead to hypomagnesemia. Very low magnesium
levels typically result when an acute problem is superimposed on chronic
depletion. For example, critical levels can occur among patients with diabetes
during correction of ketoacidosis or alcoholics who develop vomiting, diarrhea, or
pancreatitis.
17. 9. Question
Nursing interventions for a patient with hypermagnesemia include administering calcium
gluconate to:
○ A. Increase calcium levels.
○ B. Antagonize the cardiac effects of magnesium.
○ C. Lower calcium levels.
○ D. Lower magnesium levels.
18. Incorrect
Correct Answer: B. Antagonize the cardiac effects of magnesium.
In a patient with hypermagnesemia, administration of calcium gluconate will antagonize
the cardiac effects of magnesium. Calcium may moderate nerve and muscle
performance in hypermagnesemia. Calcium gluconate (Kalcinate) directly antagonizes
neuromuscular and cardiovascular effects of magnesium. Use in patients with
symptomatic hypermagnesemia that is causing cardiac effects or respiratory distress.
○ Option A: Although calcium gluconate will raise serum calcium levels, that is not
the purpose of administration. Calcium gluconate is a medication used in the
management of hypocalcemia, cardiac arrest, and cardiotoxicity due to
hyperkalemia or hypermagnesemia. It is classified as a calcium salt.
○ Option C: Calcium gluconate does not lower calcium levels. The treatment of
hypocalcemia initially focuses on symptomatic treatment rather than normalizing
serum calcium. In severe hypocalcemia with seizures, laryngospasm,
hypotension, or tetany, patients should receive emergent parenteral calcium
gluconate to replenish calcium levels until severe and life-threatening
abnormalities resolve.
○ Option D: Calcium gluconate does not lower magnesium levels. It is essential to
check magnesium levels during calcium repletion as hypomagnesemia is a
crucial cause of hypocalcemia. Hypomagnesemia causes hypocalcemia through
impairment of parathyroid hormone secretion and renal resistance to parathyroid
hormone, leading to decreased renal reabsorption of calcium.
19. 10. Question
For a patient with hypomagnesemia, which of the following medications may become
toxic?
○ A. Lasix
○ B. Digoxin
○ C. Calcium gluconate
○ D. CAPD
20. Correct
Correct Answer: B. Digoxin
In hypomagnesemia, a patient on digoxin is likely to develop digitalis toxicity. Magnesium
deficiency was the most frequently identified significant electrolyte disturbance in relation
to digoxin toxicity. In the presence of magnesium deficiency digoxin toxicity developed at
relatively low serum digoxin concentrations. Neither A nor C has toxicity as a side effect.
○ Option A: Loop diuretics (including furosemide, bumetanide, and ethacrynic acid),
produce large increases in magnesium excretion through the inhibition of the
electrical gradient necessary for magnesium reabsorption in the TAL. Long-term
thiazide diuretic therapy also may cause magnesium deficiency, through
enhanced magnesium excretion and, specifically, reduced renal expression
levels of the epithelial magnesium channel TRPM6.
○ Option C: Calcium gluconate may reverse many of the magnesium-induced
changes, including respiratory depression. Administration of IV furosemide can
increase magnesium excretion when renal function is adequate; volume status
should be maintained.
○ Option D: CAPD is not a medication. CAPD, (Continuous Ambulatory Peritoneal
Dialysis), is a way of artificially removing the waste fluid and poisons from the
body by using the abdominal membrane as a filter. The treatment involves putting
special dialysis fluid into this cavity, usually four times per day.
21. 11. Question
Which of the following is the most important physical assessment parameter the nurse
would consider when assessing fluid and electrolyte imbalance?
○ A. Skin turgor
○ B. Intake and output
○ C. Osmotic pressure
○ D. Cardiac rate and rhythm
22. Incorrect
Correct Answer: D. Cardiac rate and rhythm
Cardiac rate and rhythm are the most important physical assessment parameter to
measure. Skin turgor, intake, and output are physical assessment parameters a nurse
would consider when assessing fluid and electrolyte imbalance, but choice d is the most
important. Tachycardia and hypertension are common manifestations. Tachypnea is
usually present with or without dyspnea. Elevated CVP may be noted before dyspnea
and adventitious breath sounds occur. Hypertension may be a primary disorder or occur
secondary to other associated conditions such as heart failure.
○ Option A: Skin turgor is a sign of fluid loss (dehydration). Diarrhea or vomiting
can cause fluid loss. Infants and young children with these conditions can rapidly
lose a lot of fluid if they do not take enough water. Fever speeds up this process.
To check for skin turgor, the health care provider grasps the skin between two
fingers so that it is tented up. Commonly on the lower arm or abdomen is
checked. The skin is held for a few seconds then released.
○ Option B: These measurements are important to help evaluate a person’s fluid
and electrolyte balance, to suggest various diagnoses and allows for a prompt
intervention to correct the imbalance. Records of all intake and output must be
kept meticulously in an Intake and Output Chart (I/O Chart).
○ Option C: Osmosis is the diffusion of water across a membrane in response to
osmotic pressure caused by an imbalance of molecules on either side of the
membrane. Osmoregulation is the process of maintenance of salt and water
balance ( osmotic balance) across membranes within the body’s fluids, which are
composed of water, plus electrolytes, and non-electrolytes.
23. 12. Question
Insensible fluid losses include:
○ A. Urine
○ B. Gastric drainage
○ C. Bleeding
○ D. Perspiration
24. Incorrect
Correct Answer: D. Perspiration
Perspiration and the fluid lost via the lungs are termed insensible losses; normally,
insensible losses equal about 1000 cc/day. Insensible fluid loss is the amount of body
fluid lost daily that is not easily measured, from the respiratory system, skin, and water in
the excreted stool. The exact amount is unmeasurable but is estimated to be between
40 to 800mL/day in the average adult without comorbidities.
○ Option A: Expect a minimum of 1.5 mL/kg per hour in children and greater than 1
mL/kg per hour in adults. Special situations such as administering nephrotoxic
medications such as acyclovir warrant higher thresholds for urine output to
minimize renal toxicity.
○ Option B: An important distinction in managing fluids is differentiating between
maintenance fluids and fluid replacement. Maintenance fluids should address the
patient’s basic physiological needs, including both sensible and insensible fluid
losses. Sensible fluid losses refer to typical routes of excretion such as urination
and defecation.
○ Option C: One general principle for all patient scenarios is to replace whatever
fluid is being lost as accurately as possible. These fluid losses can differ
depending on patients’ medical conditions and differ by both volume and
composition.
25. 13. Question
Which of the following intravenous solutions would be appropriate for a patient with
severe hyponatremia secondary to syndrome of inappropriate antidiuretic hormone
(SIADH)?
○ A. Hypotonic solution
○ B. Hypertonic solution
○ C. Isotonic solution
○ D. Normotonic solution
26. Incorrect
Correct Answer: B. Hypertonic solution
When hyponatremia is severe, hypertonic solutions may be used but should be infused
with caution due to the potential for the development of CHF. This water retention dilutes
serum sodium levels, making the patient hyponatremic and necessitating the
administration of hypertonic solutions to balance sodium and water.
○ Option A: In SIADH, isotonic and hypotonic solutions are not indicated, because
urine output is minimal, so water is retained. Patients presenting with severe
symptoms such as seizures, confusion, or delirium need urgent initial correction
with hypertonic saline infusion for the first few hours rather than just water
restriction.
○ Option C: A 100 mL bolus of 3% hypertonic saline is given in the first 3 to 4
hours, and sodium levels are measured within 2 to 3 hours so that further doses
can be adjusted to avoid correcting too rapidly. A rise of 3 to 4 mEq/L within the
first few hours in such distressing conditions can be justified.
○ Option D: Normotonic solutions do not exist. If the patient’s mental status does
not improve, more boluses of 100 mL hypertonic saline can be given in the same
way as above until symptoms get better.
27. 14. Question
Aldosterone secretion in response to fluid loss will result in which one of the following
electrolyte imbalances?
○ A. Hypokalemia
○ B. Hyperkalemia
○ C. Hyponatremia
○ D. Hypernatremia
28. Correct
Correct Answer: A. Hypokalemia
Aldosterone is secreted in response to fluid loss. Aldosterone causes sodium
reabsorption and potassium elimination, further exacerbating hypokalemia. Aldosterone
causes sodium to be absorbed and potassium to be excreted into the lumen by principal
cells. In alpha intercalated cells, located in the late distal tubule and collecting duct,
hydrogen ions and potassium ions are exchanged. Hydrogen is excreted into the lumen,
and the potassium is absorbed.
○ Option B: The most common cause of hyperkalemia is pseudohyperkalemia,
which is not reflective of the true serum potassium levels. Pseudohyperkalemia is
most commonly due to hemolysis of the sample causing intracellular potassium
to be measured in the serum. Metabolic acidosis may cause intracellular
potassium to shift into the extracellular space without red cell injury. Metabolic
acidosis is most frequently caused by decreased, effective, circulating, arterial
blood volume.
○ Option C: Physiological stimuli that cause vasopressin release in adjunct with
increased fluid intake can cause hyponatremia. Hypothyroidism and adrenal
insufficiency may contribute to an increased release of vasopressin.
Physiological stimuli for vasopressin release include loss of intravascular volume
(hypovolemic hyponatremia) and the loss of effective intravascular volume
(hypervolemic hyponatremia).
○ Option D: The basic mechanisms of hypernatremia are water deficit and excess
solute. Total body water loss relative to solute loss is the most common reason
for developing hypernatremia. Hypernatremia is usually associated with
hypovolemia, which can occur in conditions that cause combined water and
solute loss, where water loss is greater than sodium loss, or free water loss.
29. 15. Question
When assessing a patient for signs of fluid overload, the nurse would expect to observe:
○ A. Bounding pulse
○ B. Flat neck veins
○ C. Poor skin turgor
○ D. Vesicular
30. Incorrect
Correct Answer: A. Bounding pulse
Bounding pulse is a sign of fluid overload as more volume in the vessels causes a
stronger sensation against the blood vessel walls. Assess for bounding peripheral pulses
and S3. These assessment findings are signs of fluid overload.
○ Option B: Flat neck veins and vesicular breath sounds are normal findings.
Check for distended neck veins and ascites. Monitor abdominal girth to follow
any ascites accurately. Distended neck veins are caused by elevated CVP.
Ascites occur when fluid accumulates in extravascular spaces.
○ Option C: Poor skin turgor is consistent with dehydration. Note for the presence
of edema by palpating over the tibia, ankles, feet, and sacrum. Edema occurs
when fluid accumulates in the extravascular spaces. Dependent areas more
readily exhibit signs of edema formation.
○ Option D: Assess for crackles in the lungs, changes in respiratory pattern,
shortness of breath, and orthopnea. These signs are caused by an accumulation
of fluid in the lungs.
31. 16. Question
The physician has ordered IV replacement of potassium for a patient with severe
hypokalemia. The nurse would administer this:
○ A. By rapid bolus
○ B. Diluted in 100 cc over 1 hour
○ C. Diluted in 10 cc over 10 minutes
○ D. IV push
32. Correct
Correct Answer: B. Diluted in 100 cc over 1 hour
Potassium must be well diluted and given slowly because rapid administration will cause
cardiac arrest. Intravenous KCl may be used as an alternative in these cases if the
patient cannot tolerate oral KCl. Severe (less than 2.5 mEq/L) or symptomatic
hypokalemia necessitates intravenous administration of KCl.
○ Option A: Injectable KCl formulations have the potential to cause injection site
complications (e.g., phlebitis, erythema, thrombosis, etc.). Also, rapid injection of
KCl can precipitate mild hyperkalemia. If the necessary infusion rate for such
cases is greater than 10 mEq/hour, the KCl should be administered through a
central line and with cardiac monitoring.
○ Option C: Patients treated with intravenous KCl may require more frequent
checking, especially if the serum potassium level addressed is below 2.5 mEq/L.
The use of continuous cardiac monitoring can aid in correlating symptoms with
telling electrocardiogram (ECG) changes (e.g., peaked T waves in hyperkalemia,
flattened T waves in hypokalemia, etc.).
○ Option D: KCl toxicity is primarily a discussion of hyperkalemia. Like
hypokalemia, the potentially fatal complication of hyperkalemia is cardiac
arrhythmia. The risk for cardiac arrhythmia is significant at serum potassium
levels greater than 6.0 to 6.5 mEq/L.
33. 17. Question
Which of the following findings would the nurse expect to assess in a patient with
hypokalemia?
○ A. Hypertension
○ B. pH below 7.35
○ C. Hypoglycemia
○ D. Hyporeflexia
34. Incorrect
Correct Answer: D. Hyporeflexia
Hyporeflexia is a symptom of hypokalemia. Significant muscle weakness occurs at
serum potassium levels below 2.5 mmol/L but can occur at higher levels if the onset is
acute. Similar to the weakness associated with hyperkalemia, the pattern is ascending in
nature affecting the lower extremities, progressing to involve the trunk and upper
extremities, and potentially advancing to paralysis.
○ Option A: Hypokalemia can result in a variety of cardiac dysrhythmias. Although
cardiac dysrhythmias or ECG changes are more likely to be associated with
moderate to severe hypokalemia, there is a high degree of individual variability
and can occur with even mild decreases in serum levels.
○ Option B: Prolonged hypokalemia can cause structural and functional changes in
the kidney that include impairing concentrating ability, increased ammonia
production, altered sodium reabsorption and increased bicarbonate absorption.
Hypokalemia can also result in glucose intolerance by reducing insulin secretion.
○ Option C: Hypomagnesemia often occurs with and may worsen hypokalemia
especially in the presence of chronic diarrhea, alcoholism, genetic disorders,
diuretic use, and chemotherapy. Both promote the development of cardiac
dysrhythmias. The combination of hypokalemia and hypomagnesemia is
associated with an increased risk of torsades de pointes, particularly in
individuals receiving QT-prolonging medications.
35. 18. Question
Vien is receiving oral potassium supplements for his condition. How should the
supplements be administered?
○ A. Undiluted
○ B. Diluted
○ C. On an empty stomach
○ D. At bedtime
36. Correct
Correct Answer: B. Diluted
Oral potassium supplements are known to irritate gastrointestinal (GI) mucosa and
should be diluted. Mild to moderate hypokalemia is typically treated with oral potassium
supplements. Providing 60 to 80 mmol/day in divided doses over days to weeks is
usually sufficient. Oral supplementation can irritate GI mucosa leading to bleeding and/or
ulceration but is associated with a lower risk of rebound hyperkalemia. It should be taken
with plenty of fluids and food.
○ Option A: Potassium chloride is the preferred formulation for replacement therapy
in most cases. Increasing dietary potassium is not usually adequate to treat
hypokalemia because most of the potassium contained in foods is coupled with
phosphate.
○ Option C: The goal of potassium replacement in the context of renal or GI losses
is to immediately raise serum potassium concentration to a safe level and then
replace the remaining deficit over days to weeks. A potassium-sparing diuretic
should also be considered when the etiology of hypokalemia involves renal
potassium wasting as potassium replacement therapy alone may not suffice.
○ Option D: IV administration is preferred in the setting of cardiac dysrhythmias,
digitalis toxicity, and recent or ongoing cardiac ischemia. Pain and phlebitis
usually occur with peripheral IV infusions when infusion rates exceed 10 mmol
per hour.
37. 19. Question
Normal venous blood pH ranges from:
○ A. 6.8 to 7.2
○ B. 7.31 to 7.41
○ C. 7.35 to 7.45
○ D. 7.0 to 8.0
38. Incorrect
Correct Answer: B. 7.31 to 7.41
Normal venous blood pH ranges from 7.31 to 7.41. Normal arterial blood pH ranges from
7.35 to 7.45. Blood gas analysis is a commonly used diagnostic tool to evaluate the
partial pressures of gas in blood and acid-base content. Understanding and use of blood
gas analysis enable providers to interpret respiratory, circulatory, and metabolic
disorders.
○ Option A: A “blood gas analysis” can be performed on blood obtained from
anywhere in the circulatory system (artery, vein, or capillary). An arterial blood
gas (ABG) tests explicitly blood taken from an artery.
○ Option C: ABG analysis assesses a patient’s partial pressure of oxygen (PaO2)
and carbon dioxide (PaCO2). PaO2 provides information on the oxygenation
status, and PaCO2 offers information on the ventilation status (chronic or acute
respiratory failure).
○ Option D: PaCO2 is affected by hyperventilation (rapid or deep breathing),
hypoventilation (slow or shallow breathing), and acid-base status. Although
oxygenation and ventilation can be assessed non-invasively via pulse oximetry
and end-tidal carbon dioxide monitoring, respectively, ABG analysis is the
standard.
39. 20. Question
Respiratory regulation of acids and bases involves:
○ A. Hydrogen
○ B. Hydroxide
○ C. Oxygen
○ D. Carbon dioxide
40. Incorrect
Correct Answer: D. Carbon dioxide
Respiratory regulation of acid-base balance involves the elimination or retention of
carbon dioxide. Arterial blood gas interpretation is best approached systematically.
Interpretation leads to an understanding of the degree or severity of abnormalities,
whether the abnormalities are acute or chronic, and if the primary disorder is metabolic
or respiratory in origin.
○ Option A: When assessing the acid-base balance, most ABG analyzers measure
the pH and PaCO2 directly. A derivative of the Hasselbach equation calculates
the serum bicarbonate (HCO3) and base deficit or excess. This calculation
frequently results in a discrepancy from the measured due to the blood CO2
unaccounted for by the equation.
○ Option B: The measured HCO3 uses a strong alkali that liberates all CO2 in
serum, including dissolved CO2, carbamino compounds, and carbonic acid. The
calculation only accounts for dissolved CO2; this measurement using a standard
chemistry analysis will likely be called a “total CO2”.
○ Option C: A “blood gas analysis” can be performed on blood obtained from
anywhere in the circulatory system (artery, vein, or capillary). An arterial blood
gas (ABG) tests explicitly blood taken from an artery.
41. 21. Question
To determine if a patient’s respiratory system is functioning, the nurse would assess
which of the following parameters:
○ A. Respiratory rate
○ B. Pulse
○ C. Arterial blood gas
○ D. Pulse oximetry
42. Correct
Correct Answer: C. Arterial blood gas
Arterial blood gases will indicate CO2 and O2 levels. This is an indication that the
respiratory system is functioning. Blood gas analysis is a commonly used diagnostic tool
to evaluate the partial pressures of gas in blood and acid-base content. Understanding
and use of blood gas analysis enable providers to interpret respiratory, circulatory, and
metabolic disorders.
○ Option A: Respiratory rate can reveal data about other systems, such as the
brain, making letter c a better choice. The respiratory rate is the number of
breaths per minute. The normal breathing rate is about 12 to 20 beats per minute
in an average adult. In the pediatric age group, it is defined by the particular age
group. Parameters important here again include its rate, depth of breathing, and
its pattern rate of breathing is a crucial parameter.
○ Option B: Pulse rate is not a measure of respiratory status. Parameters for
assessment of pulse include its rate, rhythm, volume, amplitude, and rate of
increase, besides its symmetry The rate of the pulse is significant to measure for
assessing the physiological and pathological processes affecting the body. The
normal range used in an adult is between 60 to 100 beats /minute with rates
above 100 beats/minute and rates and below 60 beats per minute.
○ Option D: Pulse oximetry yields oxygen saturation levels, which is not a measure
of acid-base balance. Pulse oximetry is a non-invasive monitor that measures the
oxygen saturation in the blood by shining light at specific wavelengths through
tissue (most commonly the fingernail bed).
43. 22. Question
Which of the following conditions is an equal decrease of extracellular fluid (ECF) solute
and water volume?
○ A. Hypotonic FVD
○ B. Isotonic FVD
○ C. Hypertonic FVD
○ D. Isotonic FVE
44. Correct
Correct Answer: B. Isotonic FVD
Isotonic FVD involves an equal decrease in solute concentration and water volume.
ISOTONIC FLUID VOLUME deficit is a proportionate loss of sodium and water.
Characterized by decreased extracellular fluid, including decreased circulating blood
volume, isotonic fluid volume deficit results in signs and symptoms of dehydration.
Common causes include vomiting, diarrhea, and polyuria.
○ Option A: Hypotonic dehydration occurs when sodium loss is greater than water
loss, resulting in a decrease in serum osmolality. This causes a shift of water
from the extracellular space into the intracellular space. The cells swell and
cerebral edema may occur. Hyponatremia can be acute or chronic.
○ Option C: Hypertonic dehydration occurs when water excretion from the body
exceeds that of sodium excretion, resulting in increased sodium concentration in
the extracellular fluid (hypernatremia). Blood osmolality is increased, causing
water to shift from the intracellular to the extracellular space.
○ Option D: The excess of both solutes and water, which is also termed isotonic
volume excess. The additional fluid is retained in the extracellular compartment
resulting in fluid accumulation in the interstitial spaces.
45. 23. Question
When monitoring the daily weight of a patient with fluid volume deficit (FVD), the nurse is
aware that fluid loss may be considered when weight loss begins to exceed:
○ A. 0.25 lb
○ B. 0.50 lb
○ C. 1 lb
○ D. 1 kg
46. Incorrect
Correct Answer: B. 0.50 lb
Weight loss of more than 0.50 lb. is considered to be a fluid loss. Weigh daily with the
same scale, and preferably at the same time of day. Weight is the best assessment data
for possible fluid volume imbalance. An increase of 2 lbs a week is considered normal.
○ Option A: Severe dehydration by clinical examination suggests a fluid deficit of
10-15% of body weight in infants and 6-9% of body weight in older children. The
daily maintenance fluid is added to the fluid deficit.
○ Option C: In general, the recommended administration is one-half of this volume
administered over 8 hours and administration of the remainder over the following
16 hours. Continued losses (eg, emesis, diarrhea) must be promptly replaced.
○ Option D: An alternative approach to the deficit therapy approach is rapid
replacement therapy. With this approach, a child with severe isonatremic
dehydration is administered 20-40 mL/kg of isotonic sodium chloride solution or
lactated Ringer solution over 15-60 minutes.
47. 24. Question
Dietary recommendations for a patient with a hypotonic fluid excess should include:
○ A. Decreased sodium intake
○ B. Increased sodium intake
○ C. Increased fluid intake
○ D. Intake of potassium-rich foods
48. Correct
Correct Answer: B. Increased sodium intake
Hypotonic fluid volume excess (FVE) involves an increase in water volume without an
increase in sodium concentration. Increased sodium intake is part of the management of
this condition. Hypotonic dehydration occurs when sodium loss is greater than water
loss, resulting in a decrease in serum osmolality. This causes a shift of water from the
extracellular space into the intracellular space. The cells swell and cerebral edema may
occur.
○ Option A: Hyponatremia can be acute or chronic. If sodium loss occurs for more
than 48 hours, it becomes chronic hyponatremia, and the body may adapt to this
state. Sodium imbalance mainly manifests as neurological symptoms ranging
from headaches, nausea, lethargy, and potentially confusion, coma, and death.
○ Option C: Adequate hydration is recommended during all activities to prevent
dehydration. Water intake is key to replacing fluid lost during exercise, in hot
weather, during hospitalization, and in elderly patients with impaired thirst
sensation.
○ Option D: Intravenous fluids can be used for fluid replacement in unconscious
patients with severe dehydration while plain water, sports drinks with electrolytes,
and oral rehydration solutions are used for the prevention and treatment of mild
dehydration. Electrolyte monitoring is mandatory for patients using diuretics for
prolonged periods.
49. 25. Question
Osmotic pressure is created through the process of:
○ A. Osmosis
○ B. Diffusion
○ C. Filtration
○ D. Capillary dynamics
50. Incorrect
Correct Answer: B. Diffusion
In diffusion, the solute moves from an area of higher concentration to one of lower
concentration, creating osmotic pressure. There is a form of passive transport called
facilitated diffusion. It occurs when molecules such as glucose or amino acids move from
high concentration to low concentration facilitated by carrier proteins or pores in the
membrane.
○ Option A: Osmotic pressure is related to the process of osmosis. Osmosis is a
form of passive transport when water molecules move from low solute
concentration(high water concentration) to high solute or low water concentration
across a membrane that is not permeable to the solute.
○ Option C: Filtration is created by hydrostatic pressure. Filtration is a process used
to separate solids from liquids or gases using a filter medium that allows the fluid
to pass through but not the solid. The term “filtration” applies whether the filter is
mechanical, biological, or physical. The fluid that passes through the filter is
called the filtrate.
○ Option D: Capillary dynamics are related to fluid exchange at the intravascular
and interstitial levels. Capillary dynamics are controlled by the four Starling
forces. Oncotic pressure is a form of osmotic pressure exerted by proteins either
in the blood plasma or interstitial fluid. … The net filtration pressure is the
balance of the four Starling forces and determines the net flow of fluid across the
capillary membrane.
51. 26. Question
A rise in arterial pressure causes the baroreceptors and stretch receptors to signal an
inhibition of the sympathetic nervous system, resulting in:
○ A. Decreased sodium reabsorption
○ B. Increased sodium reabsorption
○ C. Decreased urine output
○ D. Increased urine output
52. Incorrect
Correct Answer: D. Increased urine output
Arterial baroreceptors and stretch receptors help maintain fluid balance by increasing
urine output in response to a rise in arterial pressure. Baroreceptors are sensitive to the
rate of pressure change as well as to the steady or mean pressure. Therefore, at a given
mean arterial pressure, decreasing the pulse pressure (systolic minus diastolic pressure)
decreases the baroreceptor firing rate. This is important during conditions such as
hemorrhagic shock in which pulse pressure, as well as mean pressure, decreases. The
combination of reduced mean pressure and reduced pulse pressure amplifies the
baroreceptor response.
○ Option A: During chronic increased dietary sodium intake, arterial baroreceptors
buffer against sustained increases in arterial pressure, and renal sympatho
inhibition contributes importantly to the maintenance of sodium balance by
decreasing renal tubular sodium reabsorption and increasing urinary sodium
excretion.
○ Option B: Arterial baroreceptors function to inform the autonomic nervous system
of beat-to-beat changes in blood pressure within the arterial system. Rapid
decreases in blood pressure, such as in orthostatic hypotension, resulted in
decreased stretching of the artery wall and decreased action potential frequency,
ultimately resulting in increased cardiac output and vasoconstriction resulting in
increased blood pressure.
○ Option C: Nerve impulses from cardiopulmonary baroreceptors are also tonically
active and increase their rate of firing secondary to increased blood volume and
mean arterial pressure results in decreased sympathetic outflow to the sinoatrial
node and decreased heart rate and cardiac output. In a notable difference,
sympathetic outflow to the kidney increases, which increases renal blood flow
and urine production, thereby decreasing the fluid volume of the body.
53. 27. Question
Normal serum sodium concentration ranges from:
○ A. 120 to 125 mEq/L
○ B. 125 to 130 mEq/L
○ C. 136 to 145 mEq/L
○ D. 140 to 148 mEq/L
54. Correct
Correct Answer: C. 136 to 145 mEq/L
Normal serum sodium level ranges from 136 to 145 mEq/L. Sodium, which is an
osmotically active anion, is one of the most important electrolytes in the extracellular
fluid. It is responsible for maintaining the extracellular fluid volume, and also for
regulation of the membrane potential of cells. Sodium is exchanged along with
potassium across cell membranes as part of active transport.
○ Option A: Sodium regulation occurs in the kidneys. The proximal tubule is where
the majority of the sodium reabsorption takes place. In the distal convoluted
tubule, sodium undergoes reabsorption. Sodium transport takes place via
sodium-chloride symporters, which is by the action of the hormone aldosterone.
○ Option B: Among the electrolyte disorders, hyponatremia is the most frequent.
Diagnosis is when the serum sodium level less than 135 mmol/L. Hyponatremia
has neurological manifestations. Patients may present with headache, confusion,
nausea, delirium.
○ Option D: Hypernatremia presents when the serum sodium levels greater
than145 mmol/L. Symptoms of hypernatremia include tachypnea, sleeping
difficulty, and feeling restless. Rapid sodium corrections can have serious
consequences like cerebral edema and osmotic demyelination syndrome.
55. 28. Question
When assessing a patient for electrolyte balance, the nurse is aware that etiologies for
hyponatremia include:
○ A. Water gain
○ B. Diuretic therapy
○ C. Diaphoresis
○ D. All of the following
56. Incorrect
Correct Answer: D. All of the following
Water gain, diuretic therapy, and diaphoresis are etiologies of hyponatremia. The
etiology of hyponatremia can be classified based upon the volume status of the
extracellular fluid. Sodium is the major solute of extracellular fluid (ECF). Based upon the
volume of ECF, a patient can be classified into hypovolemic, euvolemic, or hypervolemic.
○ Option A: Physiological stimuli that cause vasopressin release in adjunct with
increased fluid intake can cause hyponatremia. Hypothyroidism and adrenal
insufficiency may contribute to an increased release of vasopressin.
Physiological stimuli for vasopressin release include loss of intravascular volume
(hypovolemic hyponatremia) and the loss of effective intravascular volume
(hypervolemic hyponatremia).
○ Option B: Many drugs cause hyponatremia and the most common include:
vasopressin analogs such as desmopressin and oxytocin; medications that
stimulate vasopressin release or potentiate the effects of vasopressin such as
selective serotonin-reuptake inhibitors and other antidepressants morphine and
other opioids; and medications that impair urinary dilution such as thiazide
diuretics.
○ Option C: Water excretion is tightly regulated by antidiuretic hormone (ADH),
synthesized in the hypothalamus, and stored in the posterior pituitary gland.
Changes in tonicity lead to either enhancement or suppression of ADH secretion.
Increased ADH secretion causes reabsorption of water in the kidney, and
suppression causes the opposite effect.
57. 29. Question
Nursing interventions for a patient with hyponatremia include:
○ A. Administering hypotonic IV fluids.
○ B. Encouraging water intake.
○ C. Restricting fluid intake.
○ D. Restricting sodium intake.
58. Incorrect
Correct Answer: C. Restricting fluid intake
Hyponatremia involves a decreased concentration of sodium in relation to fluid volume,
so restricting fluid intake is indicated. In the presence of fluid excess or SIADH, fluid
restriction is indicated while in the presence of hypovolemia, volume losses are replaced
with isotonic saline, or, on occasion, hypertonic solution when hyponatremia is
life-threatening.
○ Option A: Administer sodium chloride as indicated. Used to replace deficits in the
presence of chronic or ongoing losses. Identify the client at risk for hyponatremia
and the specific cause such as sodium loss or fluid excess. Provides clues for
early intervention. Hyponatremia is a common imbalance, especially in the
elderly, and may range from mild to severe.
○ Option B: Provide or restrict fluids, depending on fluid volume status. Encourage
fluids and foods high in sodium such as meat, milk, beets, celery, eggs, and
carrots. Use fruit juices and bouillon instead of water. Unless sodium deficit
causes serious symptoms requiring immediate IV replacement, the client may
benefit from slower replacement by oral method or removal of previous salt
restriction.
○ Option D: Monitor intake and output; Calculate fluid balance. Weigh the client
daily. Fluid balance indicators are important since either fluid excess or deficit
may occur with hyponatremia.
59. 30. Question
The nurse would analyze an arterial pH of 7.46 as indicating:
○ A. Acidosis
○ B. Alkalosis
○ C. Homeostasis
○ D. Neutrality
60. Correct
Correct Answer: B. Alkalosis
Alkalosis is indicated by a pH above 7.45. A pH below 7.35 is an acidemia, and a pH
above 7.45 is an alkalemia. Due to the importance of sustaining a pH level in the needed
narrow range, the human body contains compensatory mechanisms.
○ Option A: The human body experiences four main types of acid-based disorders:
metabolic acidosis, metabolic alkalosis, respiratory acidosis, and respiratory
alkalosis. If one of these conditions occurs, the human body should induce a
counterbalance in the form of an opposite condition.
○ Option C: To maintain homeostasis, the human body employs many physiological
adaptations. One of these is maintaining an acid-base balance. In the absence of
pathological states, the pH of the human body ranges between 7.35 to 7.45, with
the average at 7.40.
○ Option D: Arterial blood gas (ABG) sampling, is a test often performed in an
inpatient setting to assess the acid-base status of a patient. A needle is used to
draw blood from an artery, often the radial and the blood is analyzed to determine
parameters such as the pH, pC02, pO2, HCO3, oxygen saturation, and more.

1. 1. Question
The net diffusion of water from one solution of water from one solution through a
semipermeable membrane to another solution containing a lower concentration of water
is termed:
○ A. Filtration
○ B. Diffusion
○ C. Osmosis
○ D. Brownian motion
2. Incorrect
Correct Answer: C. Osmosis
Osmosis is defined as the diffusion of water through a semipermeable membrane to a
solution with a lower concentration of water. It is important to emphasize that ideal
osmosis requires only the movement of pure water across the membrane without any
movement of solute particles across the semipermeable membrane. Osmosis can still
occur with some permeability of solute particles, but the osmotic effect becomes reduced
with greater solute permeability across the semipermeable membrane.
○ Option A: Filtration is the process in which fluids are pushed through biological
membranes by unequal processes. Filtration is a process used to separate solids
from liquids or gases using a filter medium that allows the fluid to pass through
but not the solid. The term “filtration” applies whether the filter is mechanical,
biological, or physical. The fluid that passes through the filter is called the filtrate.
○ Option B: Diffusion is the random kinetic motion causing atoms and molecules to
spread out evenly. There is a form of passive transport called facilitated diffusion.
It occurs when molecules such as glucose or amino acids move from high
concentration to low concentration facilitated by carrier proteins or pores in the
membrane.
○ Option D: Brownian motion is the random motion of particles suspended in a fluid
(a liquid or a gas) resulting from their collision with the fast-moving atoms or
molecules in the gas or liquid. This transport phenomenon is named after the
botanist Robert Brown.
3. 2. Question
When assessing a patient’s total body water percentage, the nurse is aware that all of
the following factors influence this except:
○ A. Age
○ B. Fat tissue
○ C. Muscle mass
○ D. Gender
4. Incorrect
Correct Answer: D. Gender
A patient’s gender does not influence the percentage of total body water. The average
percentage of body weight that is water will remain above 50 percent for most or all of a
person’s life, though it does decline over time. About two-thirds of the body’s water is
within the cells, while the remaining third is in extracellular fluid. Minerals, including
potassium and sodium, help maintain ICF and ECF balances.
○ Option A: Water needs don’t change per se, however as people age, they
generally develop a decreased sense of thirst, putting them at increased risk of
dehydration. Several studies show that individuals older than 60 do not feel
adequately thirsty even when in physiological need. The end result is that simple
environmental stresses, such as heat, can put a person at risk if adequate
hydration is not maintained.
○ Option B: The decreasing water percentage through the years is due in large part
to having more body fat and less fat-free mass as people age. Fatty tissue
contains less water than lean tissue, so the weight and body composition affect
the percentage of water in the body.
○ Option C: One of the key functions of protein is to “hold onto” fluid in the blood.
Individuals consuming too little protein are unable to maintain adequate amounts
of water in the blood. Inadequate dietary protein can cause increased urination,
fatigue, or feeling bloated.
5. 3. Question
Orly Khan is suffering from fluid volume deficit (FVD), which of the following symptoms
would the nurse expect to assess in the patient?
○ A. Rales
○ B. Bounding pulse
○ C. Tachycardia
○ D. Bulging neck veins
6. Correct
Correct Answer: C. Tachycardia
Tachycardia, poor tissue turgor, and hypotension are symptoms of FVD. Other choices
are symptoms of FVE. A decrease in circulating blood volume can cause hypotension
and tachycardia. Alteration in HR is a compensatory mechanism to maintain cardiac
output.
○ Option A: Auscultate and document heart sounds; note rate, rhythm or other
abnormal findings. Cardiac alterations like dysrhythmias may reflect hypovolemia
and/or electrolyte imbalance, commonly hypocalcemia. Note: MI, pericarditis, and
pericardial effusion with/ without tamponade are common cardiovascular
complications.
○ Option B: Usually, the pulse is weak and may be irregular if electrolyte imbalance
also occurs. Hypotension is evident in hypovolemia. Close monitoring for
responses during therapy reduces complications associated with fluid
replacement.
○ Option D: Assess skin turgor and oral mucous membranes for signs of
dehydration. Signs of dehydration are also detected through the skin. Skin of
elderly patients loses elasticity, hence skin turgor should be assessed over the
sternum or on the inner thighs. Longitudinal furrows may be noted along the
tongue.
7. 4. Question
John Reid is admitted to the hospital and is currently receiving hypertonic fluids. Nursing
management for the client includes monitoring for all of the following potential
complications except:
○ A. Water intoxication
○ B. Fluid volume excess (FVE)
○ C. Cellular dehydration
○ D. Cell shrinkage
8. Incorrect
Correct Answer: A. Water intoxication
Water intoxication is a potential complication associated with hypotonic fluid
administration. Water intoxication provokes disturbances in electrolyte balance, resulting
in a rapid decrease in serum sodium concentration and eventual death. The
development of acute dilutional hyponatremia causes neurological symptoms because of
the movement of water into the brain cells, in response to the fall in extracellular
osmolality. Other choices are potential complications of hypertonic fluid administration.
○ Option B: Fluid Volume Excess (FVE), or hypervolemia, refers to an isotonic
expansion of the ECF due to an increase in total body sodium content and an
increase in total body water. This fluid overload usually occurs from compromised
regulatory mechanisms for sodium and water as seen commonly in heart failure
(CHF), kidney failure, and liver failure.
○ Option C: Water moves from inside the cells to the bloodstream to maintain the
needed amount of blood (blood volume) and blood pressure. If dehydration
continues, tissues of the body begin to dry out, and cells begin to shrivel and
malfunction.
○ Option D: Cell shrinkage, or the loss of cell volume, is a ubiquitous characteristic
of programmed cell death that is observed in all examples of apoptosis,
independent of the death stimulus. This decrease in cell volume occurs in
synchrony with other classical features of apoptosis.
9. 5. Question
Mr. Wenceslao is scheduled to receive an isotonic solution; which one of the following is
an example of such a solution?
○ A. D10% W
○ B. 0.45% saline
○ C. 0.9% saline
○ D. 3% normal saline W
10. Incorrect
Correct Answer: C. A solution of 0.9% saline is isotonic.
A solution of 0.9% saline is isotonic. Solutions of 0.33% and 0.45% saline and D5W are
hypotonic. An isotonic solution is one that has the same osmolarity, or solute
concentration, as another solution. If these two solutions are separated by a
semipermeable membrane, water will flow in equal parts out of each solution and into
the other.
○ Option A: D10% W is a crystalloid fluid. Crystalloid fluids are a subset of
intravenous solutions that are frequently used in the clinical setting. Crystalloid
fluids are the first choice for fluid resuscitation in the presence of hypovolemia,
hemorrhage, sepsis, and dehydration.
○ Option B: 0.45% saline is a hypotonic solution. If a cell is placed in a hypotonic
solution, there will be a net flow of water into the cell, and the cell will gain
volume. If the solute concentration outside the cell is lower than inside the cell,
and the solutes cannot cross the membrane, then that solution is hypotonic to the
cell.
○ Option D: 0.33% saline is a hypotonic solution. In the case of a plant cell,
however, a hypotonic extracellular solution is actually ideal. The plasma
membrane can only expand to the limit of the rigid cell wall, so the cell won’t
burst, or lyse.
11. 6. Question
Which of the following arterial blood gas (ABG) values indicates uncompensated
metabolic alkalosis?
○ A. pH 7.48, PaCO2 42, HCO3 30
○ B. pH 7.48, PaCO2 46, HCO3 30
○ C. pH 7.48, PaCO2 34, HCO3 20
○ D. pH 7.48, PaCO2 34, HCO3 26
12. Incorrect
Correct Answer: A. pH 7.48, PaCO2 42, HCO3 30
Uncompensated metabolic alkalosis is indicated by ABG values of pH 7.48, PaCO2 42,
and HCO3 30. Normal human physiological pH is 7.35 to 7.45. A decrease in pH below
this range is acidosis, an increase over this range is alkalosis. Metabolic alkalosis is
defined as a disease state where the body’s pH is elevated to greater than 7.45
secondary to some metabolic process.
○ Option B: These values indicate metabolic alkalosis, partially compensated.
HCO3 functions as an alkalotic substance. CO2 functions as an acidic
substance. Therefore, increases in HCO3 or decreases in CO2 will make blood
more alkalotic. The opposite is also true where decreases in HCO3 or an
increase in CO2 will make blood more acidic. CO2 levels are physiologically
regulated by the pulmonary system through respiration, whereas the HCO3
levels are regulated through the renal system with reabsorption rates.
○ Option C: These values indicate respiratory alkalosis, partially compensated.
Respiratory alkalosis is 1 of the 4 basic classifications of blood pH imbalances.
Normal human physiological pH is 7.35 to 7.45. A decrease in pH below this
range is acidosis, an increase above this range is alkalosis. Respiratory alkalosis
is by definition a disease state where the body’s pH is elevated to greater than
7.45 secondary to some respiratory or pulmonary process.
○ Option D: These values indicate respiratory alkalosis, uncompensated. If the pH
is not within or close to the normal ranges, then a partial compensation exists. If
the pH is back within normal ranges then a full compensation has occurred. A
non-compensated or uncompensated abnormality usually represents an acute
change occurring in the body.
13. 7. Question
The body’s compensation of metabolic alkalosis involves:
○ A. Increasing the respiratory rate
○ B. Decreasing the respiratory rate
○ C. Increasing urine output
○ D. Decreasing urine output
14. Correct
Correct Answer: B. Decreasing the respiratory rate
The body attempts to compensate for metabolic alkalosis by decreasing the respiratory
rate and conserving carbon dioxide (an acid). The body compensates for both alkalosis
and acidosis mainly through the lungs. The lungs change the alkalinity of the blood by
allowing more or less carbon dioxide to escape as the client breathes. The kidneys also
play a role by controlling the elimination of bicarbonate ions.
○ Option A: In metabolic acidosis, as blood pH drops (becomes more acidic), the
parts of the brain that regulate breathing are stimulated to produce faster and
deeper breathing (respiratory compensation). Breathing faster and deeper
increases the amount of carbon dioxide exhaled. The kidneys also try to
compensate by excreting more acid in the urine.
○ Option C: Urine volume does not influence acid-base balance. The kidneys
compensate for a respiratory acidosis by tubular cells reabsorbing more HCO3
from the tubular fluid, collecting duct cells secreting more H+ and generating
more HCO3, and ammoniagenesis leading to increased formation of the NH3
buffer.
○ Option D: The kidney compensates in response to respiratory alkalosis by
reducing the amount of new HCO3? generated and by excreting HCO3?. The
process of renal compensation occurs within 24 to 48 hours. The stimulus for the
renal compensatory mechanism is not pH, but rather Pco2.
15. 8. Question
When assessing a patient for metabolic alkalosis, the nurse would expect to find:
○ A. Low serum potassium
○ B. Changes in urine output
○ C. Hypotension
○ D. Increased CVP
16. Incorrect
Correct Answer: A. Low serum potassium
Decreased serum potassium is a common symptom of metabolic alkalosis. The
biological effects of metabolic alkalosis are directly resultant to associated problems
such as hypovolemia and potassium and chloride depletion. These changes lead to
decreased myocardial contractility, arrhythmias, decreased cerebral blood flow,
confusion, increased neuromuscular excitability, and impaired peripheral oxygen
unloading secondary to the shift of the oxygen dissociation curve to left.
○ Option B: Of interest here, sodium is reabsorbed through an exchange for
hydrogen in the renal collecting ducts under the influence of aldosterone.
Therefore, pathologies that increase the levels of mineralocorticoids or increase
the effect of aldosterone, such as Conn syndrome will lead to hypernatremia,
hypokalemia, and hydrogen loss in the urine.
○ Option C: In general, the causes can be narrowed down to an intracellular shift of
hydrogen ions, gastrointestinal (GI) loss of hydrogen ions, excessive renal
hydrogen ion loss, retention or addition of bicarbonate ions, or volume
contraction around a constant amount of extracellular bicarbonate known as
contraction alkalosis.
○ Option D: As long as renal function is maintained, excess bicarbonate is excreted
in the urine fairly rapidly. As a result, metabolic alkalosis will persevere if the
ability to eliminate bicarbonate is impaired due to one of the following causes:
hypovolemia, reduced effective arterial blood volume, chloride depletion,
hypokalemia, reduced glomerular filtration rate, and/or hyperaldosteronism.
17. 9. Question
Which of the following blood products should be infused rapidly?
○ A. Packed red blood cells (PRBC)
○ B. Fresh frozen plasma (FFP)
○ C. Platelets
○ D. Dextran
18. Incorrect
Correct Answer: C. Platelets
Platelets and cryoprecipitate can be infused quickly. Platelet administration sets have a
smaller priming capacity than a blood administration set. A unit of platelets is usually
administered over 30 minutes. 170 – 200 micron filter is required (either a blood or
platelet administration set may be used). Platelet concentrates should not be transfused
through administration sets which have already been used for blood PRBC and FFP
should be administered over 1 ½ to 4 hours.
○ Option A: The transfusion must be completed no more than 4 hours after the
component has been removed from temperature controlled storage. 170 – 200
micron filter is required (standard blood administration set). Either gravity or
electronic infusion pumps may be used. Electronic infusion pumps should only be
used if the manufacturer verifies them as safe for that purpose.
○ Option B: A unit of FFP is usually administered over 30 minutes. Once thawed,
FFP must not be re-frozen and should be transfused as soon as possible as
post-thaw storage will result in a decline in the content of labile coagulation
factors.
○ Option D: Dextran is not a blood product. In the operating room and intensive
care unit settings, dextran is used osmotically as a colloid to treat hypovolemia
and/or hemorrhage from trauma, burns, surgeries, or other causes if ABO
compatibility tests are not possible in time.
19. 10. Question
Which of the following statements provides the rationale for using a hypotonic solution
for a patient with FVD?
○ A. A hypotonic solution provides free water to help the kidneys eliminate the
solute.
○ B. A hypotonic solution supplies an excess of sodium and chloride ions.
○ C. Excessive volumes are recommended in the early postoperative period.
○ D. A hypotonic solution is used to treat hyponatremia.
20. Incorrect
Correct Answer: A. A hypotonic solution provides free water to help the kidneys eliminate
the solute.
Hypotonic solutions provide free water, which helps the kidneys eliminate solute. A
solution that contains fewer dissolved particles (such as salt and other electrolytes) than
is found in normal cells and blood. Hypotonic solutions are commonly used to give fluids
intravenously to hospitalized patients in order to treat or avoid dehydration.
○ Option B: If a cell is placed in a hypotonic solution, there will be a net flow of
water into the cell, and the cell will gain volume. If the solute concentration
outside the cell is lower than inside the cell, and the solutes cannot cross the
membrane, then that solution is hypotonic to the cell.
○ Option C: If a cell is placed in a hypertonic solution, there will be a net flow of
water out of the cell, and the cell will lose volume. A solution will be hypertonic to
a cell if its solute concentration is higher than that inside the cell, and the solutes
cannot cross the membrane.
○ Option D: In severely symptomatic hyponatremia, administer 3% sodium chloride;
100 mL intravenous (IV) bolus (repeat up to twice if symptoms persist). In mild to
moderately symptomatic hyponatremia, 3% Sodium chloride, slow infusion (use
sodium deficit formula to calculate the rate of infusion but recalculate rate with
frequent sodium monitoring).
21. 11. Question
Redd is receiving a blood transfusion. When monitoring the patient, the nurse would
analyze an elevated body temperature as indicating:
○ A. A normal physiologic process.
○ B. Evidence of sepsis.
○ C. A possible transfusion reaction.
○ D. An expected response to the transfusion.
22. Correct
Correct Answer: C. A possible transfusion reaction.
An increase in the body temperature indicates a possible transfusion reaction and
requires immediate discontinuation of the infusion. Transfusion reactions are defined as
adverse events associated with the transfusion of whole blood or one of its components.
These may range in severity from minor to life-threatening.
○ Option A: Reactions can occur during the transfusion (acute transfusion
reactions) or days to weeks later (delayed transfusion reactions) and may be
immunologic or non-immunologic. A reaction may be difficult to diagnose as it
can present with non-specific, often overlapping symptoms.
○ Option B: The most common signs and symptoms include fever, chills, urticaria
(hives), and itching. Some symptoms resolve with little or no treatment. However,
respiratory distress, high fever, hypotension (low blood pressure), and red urine
(hemoglobinuria) can indicate a more serious reaction.
○ Option D: Immune-mediated transfusion reactions typically occur due to
mismatch or incompatibility of the transfused product and the recipient. They
include naturally occurring antibodies in the blood recipient (such as anti-A,
anti-B which are typically responsible for acute hemolytic transfusion reactions)
as well as antibodies made in response to foreign antigens (alloantibodies).
23. 12. Question
The process of endocrine regulation of electrolytes involves:
○ A. Sodium reabsorption and chloride excretion
○ B. Chloride reabsorption and sodium excretion
○ C. Potassium reabsorption and sodium excretion
○ D. Sodium reabsorption and potassium excretion
24. Correct
Correct Answer: D. Sodium reabsorption and potassium excretion
ACTH stimulates the release of aldosterone, which in turn acts on the tubules to
reabsorb sodium. When this occurs, the cation potassium is excreted. Because “water
follows salt,” this may also lead to water retention when ADH is present. Another action
of aldosterone is to increase the secretion of potassium by the kidney resulting in its
decrease in the blood and increase in the urine.
○ Option A: Aldosterone is a hormone that regulates blood sodium levels.
Aldosterone specifically increases sodium reabsorption in the distal convoluted
tubule and collecting duct of the nephrons in the kidneys. The result of this
mechanism is to conserve sodium.
○ Option B: Aldosterone release from the adrenal cortex is triggered directly by an
increase in potassium (primarily) or a decrease in sodium in the blood reaching
the adrenal cortex. Aldosterone release is also stimulated by the activation of the
renin-angiotensin system.
○ Option C: In this mechanism, the juxtaglomerular cells of the kidneys release
renin in response to a decrease in blood volume, a reduction in blood pressure,
or stimulation by the sympathetic nervous system. Renin is an enzyme that
converts a plasma protein called angiotensinogen to angiotensin I. Angiotensin I
is in turn acted upon by angiotensin-converting enzyme (ACE) to form
Angiotensin II.
25. 13. Question
The chief anion in the intracellular fluid (ICF) is:
○ A. Phosphorus
○ B. Potassium
○ C. Sodium
○ D. Chloride
26. Incorrect
Correct Answer: A. Phosphorus
Phosphorus is the major ICF cation. Phosphorus is an extracellular fluid cation.
Eighty-five percent of the total body phosphorus is in the bones and teeth in the form of
hydroxyapatite; the soft tissues contain the remaining 15%. Phosphate plays a crucial
role in metabolic pathways.
○ Option B: Potassium is a cation. Potassium is an essential mineral constituent of
the human body and is the chief cation found within the intracellular fluid of all
cells. The chief indication for potassium administration is potassium deficiency or
hypokalemia, a condition in which serum potassium level falls below a critical
range.
○ Option C: Sodium is a cation. Sodium, which is an osmotically active anion, is
one of the most important electrolytes in the extracellular fluid. It is responsible
for maintaining the extracellular fluid volume, and also for regulation of the
membrane potential of cells. Sodium is exchanged along with potassium across
cell membranes as part of active transport.
○ Option D: Chloride is the chief anion found in the ECF. Chloride is an anion found
predominantly in the extracellular fluid. The kidneys predominantly regulate
serum chloride levels. Most of the chloride, which is filtered by the glomerulus, is
reabsorbed by both proximal and distal tubules (majorly by proximal tubule) by
both active and passive transport.
27. 14. Question
The major cation in the ICF is:
○ A. Potassium
○ B. Sodium
○ C. Phosphorus
○ D. Magnesium
28. Incorrect
Correct Answer: A. Potassium
Potassium is the major ICF cation. Potassium is mainly an intracellular ion. The
sodium-potassium adenosine triphosphatase pump has the primary responsibility for
regulating the homeostasis between sodium and potassium, which pumps out sodium in
exchange for potassium, which moves into the cells.
○ Option B: Sodium is the major ECF cation. Sodium, which is an osmotically
active anion, is one of the most important electrolytes in the extracellular fluid. It
is responsible for maintaining the extracellular fluid volume, and also for
regulation of the membrane potential of cells. Sodium is exchanged along with
potassium across cell membranes as part of active transport.
○ Option C: Phosphorus is the major ICF anion. Phosphorus is an extracellular fluid
cation. Eighty-five percent of the total body phosphorus is in the bones and teeth
in the form of hydroxyapatite; the soft tissues contain the remaining 15%.
Phosphate plays a crucial role in metabolic pathways. It is a component of many
metabolic intermediates and, most importantly of adenosine triphosphate(ATPs)
and nucleotides.
○ Option D: Magnesium is the second-most abundant cation in the ICF. Magnesium
is an intracellular cation. Magnesium is mainly involved in ATP metabolism,
contraction and relaxation of muscles, proper neurological functioning, and
neurotransmitter release. When muscle contracts, calcium re-uptake by the
calcium-activated ATPase of the sarcoplasmic reticulum is brought about by
magnesium.
29. 15. Question
Hypophosphatemia may result from which of the following diseases?
○ A. Liver cirrhosis
○ B. Renal failure
○ C. Paget’s disease
○ D. Alcoholism
30. Incorrect
Correct Answer: D. Alcoholism
Hypophosphatemia may occur secondary to alcoholism. Hypophosphatemia is typically
asymptomatic and is present in up to 5% of patients. It is much more prevalent in
alcoholism, diabetic ketoacidosis, or sepsis, with a frequency of up to 80%. The
morbidity of hypophosphatemia is highly dependent on its etiology and severity.
○ Option A: Chronic liver diseases usually progress to cirrhosis. In the developed
world, the most common causes of cirrhosis are hepatitis C virus (HCV),
alcoholic liver disease, and nonalcoholic steatohepatitis (NASH), while hepatitis B
virus (HBV) and HCV are the most common causes in the developing world.
○ Option B: Renal failure is usually associated with hyperphosphatemia. Renal
failure is the most common cause of hyperphosphatemia. A glomerular filtration
rate of less than 30 mL/min significantly reduces the filtration of inorganic
phosphate, increasing its serum level. Other less common causes include a high
intake of phosphorus or increased renal reabsorption.
○ Option C: Some literary sources suggest that the family of paramyxoviruses
solely causes Paget. However, many studies have come to determine that the
osteoclast generation of a unique cytokine found exclusively in the bone marrow
of patients diagnosed with Paget disease may be the primary insult. This cytokine
is known as IL-6.
31. 16. Question
A patient with which of the following disorders is at high risk for developing
hyperphosphatemia?
○ A. Hyperkalemia
○ B. Hyponatremia
○ C. Hypocalcemia
○ D. Hyperglycemia
32. Incorrect
Correct Answer: C. Hypocalcemia
Because calcium and phosphorus ratios are inversely proportional, when phosphorus
levels are high, calcium levels are low. Phosphate binds calcium avidly, causing acute
hypocalcemia. Acute hypocalcemia secondary to hyperphosphatemia may also result
from renal failure or excess tissue breakdown because of rhabdomyolysis or tumor lysis.
In acute hyperphosphatemia, calcium is deposited mostly in the bone but also in the
extraskeletal tissue.
○ Option A: In contrast, in chronic hyperphosphatemia, which is nearly always from
chronic renal failure, calcium efflux from the bone is inhibited and the calcium
absorption is low, because of reduced renal synthesis of 1,25-dihydroxyvitamin
D.
○ Option B: Several sodium-coupled transport proteins have been identified that
enable intracellular uptake of phosphate by taking advantage of the steep
extracellular-to-intracellular sodium gradient. Type 1 sodium phosphate
cotransporters are expressed predominantly in kidney cells on the apical
membranes of proximal tubule cells and liver.
○ Option D: Phosphorus homeostasis is normally maintained through several
mechanisms. GI absorption must be matched by renal excretion, and cellular
release is balanced by uptake in other tissues. Hyperphosphatemia occurs when
the phosphorus load (from GI absorption, exogenous administration, or cellular
release) exceeds renal excretion and tissue uptake.
33. 17. Question
Normal calcium levels must be analyzed in relation to:
○ A. Sodium
○ B. Glucose
○ C. Protein
○ D. Fats
34. Incorrect
Correct Answer: C. Protein
Some calcium is bound to protein, so abnormal calcium levels are analyzed in relation to
proteins. The ability of the protein to bind calcium acts as a buffer that alters the effect of
an acute load of calcium on the concentration of ionized calcium by about 50%. Still
another consequence of the large number of unfilled binding sites for calcium is that
competition by magnesium does not have a significant effect on ionized calcium
concentration.
○ Option A: PTH also acts to maintain the steady-state level of serum calcium by its
action on the kidney. It increases the tubular reabsorption of calcium and
magnesium and decreases the tubular reabsorption of phosphate, sodium,
bicarbonate, potassium, and amino acids. PTH activates the adenylate cyclase
system by binding with receptor sites in the renal cortex. It thus leads to an
increase in cyclic adenosine monophosphate.
○ Option B: Abnormal calcium regulation may contribute to reduced ?-cells
function, thereby promoting altered glucose homeostasis. In vitro studies have
also found that high cytosolic calcium may contribute to insulin resistance within
adipocytes and skeletal muscle.
○ Option D: The serum phosphorus level also plays a role in the maintenance of a
steady-state concentration of serum calcium. While there is no exact solubility
product for calcium and phosphorus, a rise in serum phosphate usually leads to a
fall in serum calcium. Some of this decrement may be caused by the enhanced
formation of CaHPO4 complexes in the serum.
35. 18. Question
Calcium is absorbed in the GI tract under the influence of:
○ A. Vitamin D
○ B. Glucose
○ C. HCl
○ D. Vitamin C
36. Correct
Correct Answer: A. Vitamin D
Calcium is absorbed in the GI tract under the influence of vitamin D in its biologically
active form. Vitamin D also increases the intestinal absorption of calcium, as well as
bone resorption and the tubular reabsorption of calcium. The effects on intestinal
reabsorption of calcium and bone resorption seem to be due primarily to the active
metabolite 1,25-DHCC, but other metabolites may contribute to some of the other effects
on serum calcium.
○ Option B: Abnormal calcium regulation may contribute to reduced ?-cells
function, thereby promoting altered glucose homeostasis. In vitro studies have
also found that high cytosolic calcium may contribute to insulin resistance within
adipocytes and skeletal muscle.
○ Option C: Oral calcium carbonate (0-5 g, pH 9-4) increased serum gastrin and
gastric acid output with slight but insignificant change in serum calcium. A similar
rise in serum calcium during an intravenous infusion of calcium gluconate failed
to increase serum gastrin and gastric acid output.
○ Option D: Vitamin C blocks cells that break down our bones and helps us make
more cells responsible for bone formation. It also regulates important
bone-building minerals, like calcium, in our bodies. Most people can get enough
vitamin C through diet alone.
37. 19. Question
Which of the following diagnoses is most appropriate for a patient with hypocalcemia?
○ A. Constipation, bowel
○ B. High risk for injury: bleeding
○ C. Airway clearance, ineffective
○ D. High risk for injury: confusion
38. Incorrect
Correct Answer: B. High risk for injury: bleeding
A patient with hypocalcemia may bleed, since calcium is required for normal blood
clotting. First, calcium may play a role in vascular reactivity. Hypocalcemia could
therefore lead to higher BP because of increased arterial vascular tone. A and D are
diagnoses appropriate for a patient with hypercalcemia.
○ Option A: Auscultate bowel sounds. Hypotonicity leads to constipation when the
smooth muscle tone is inadequate to produce peristalsis. Maintain bulk in diet.
Constipation may be a problem because of decreased GI tone.
○ Option C: This is not associated with fluctuating calcium levels. Monitor
respiratory rate, effort, and rhythm. Place a tracheostomy set at the bedside.
Hypocalcemia may result in laryngeal stridor leading to respiratory arrest.
○ Option D: Assess the level of consciousness and neuromuscular status, including
muscle tone, strength, and movement. Nerve and muscle activity is depressed in
hypercalcemia. Lethargy and fatigue can progress to convulsions or coma.
39. 20. Question
When serum calcium levels rise, which of the following hormones is secreted?
○ A. Aldosterone
○ B. Renin
○ C. Parathyroid hormone
○ D. Calcitonin
40. Incorrect
Correct Answer: D. Calcitonin
When calcium levels rise, calcitonin is secreted from the thyroid; this hormone moves
calcium from plasma into bone. In response to hypercalcemia, calcitonin is secreted by
the parafollicular C cells. Calcitonin lowers serum calcium by decreasing renal calcium
and phosphorus reabsorption and also by decreasing bone reabsorption. Calcitonin is
not significant in overall calcium homeostasis, but it is an important therapeutic option.
○ Option A: Aldosterone enhances renal calcium reabsorption by two types of
channels. Reabsorption by the distal nephron. The present in vitro experiments
investigated the effect of the hormone on calcium (Ca2+) transport by the luminal
membrane of the rabbit nephron, independent of any systemic influence.
○ Option B: Renin secretion is mainly dependent on cyclic AMP formation. Cyclic
AMP availability is the net effect of positive adenylyl cyclase activity and
competing degradative activity of calmodulin-activated phosphodiesterase.
Increasing intracellular calcium concentrations decrease net cyclic AMP
formation by dampening adenylate cyclase and enhancing phosphodiesterase
activities.
○ Option C: Parathyroid hormone is secreted in response to lowered calcium
levels; this hormone moves calcium from bone into plasma. The main cause of
hypercalcemia is an excess parathyroid hormone (PTH). PTH-mediated causes
include adenoma/hyperplasia of the gland, familial hypocalciuric hypercalcemia,
and multiple endocrine neoplasia syndromes (type1, 2A).
41. 21. Question
The presence of which of the following electrolytes contributes to acidosis?
○ A. Sodium
○ B. Potassium
○ C. Hydrogen
○ D. Chloride
42. Incorrect
Correct Answer: C. Hydrogen
The presence of hydrogen ions determines a solution’s acidity. Acidosis is defined as an
abnormal clinical process that causes a net gain in hydrogen ions (H+) in the
extracellular fluid. Metabolic acidosis occurs when there is an accumulation of H+ or a
loss of bicarbonate ions (HCO3?) and is reflected by a decrease in plasma HCO3? (<22
mEq/L).
○ Option A: Adrenocortical insufficiency that occurs in Addison’s disease causes
hyponatremia and renal tubular acidosis (RTA). Hyponatremia results from both
aldosterone and cortisol insufficiency. RTA is due to aldosterone insufficiency.
○ Option B: A frequently cited mechanism for these findings is that acidosis causes
potassium to move from cells to extracellular fluid (plasma) in exchange for
hydrogen ions, and alkalosis causes the reverse movement of potassium and
hydrogen ions.
○ Option D: An increased plasma chloride ion concentration relative to sodium and
potassium concentrations will produce a smaller plasma strong ion difference,
leading to an increased hydrogen ion concentration, and therefore acidosis.
43. 22. Question
The lungs participate in acid-base balance by:
○ A. Reabsorbing bicarbonate.
○ B. Splitting carbonic acid in two.
○ C. Using CO2 to regulate hydrogen ions.
○ D. Sending hydrogen ions to the renal tubules.
44. Incorrect
Correct Answer: C. Using CO2 to regulate hydrogen ions
The lungs use carbon dioxide to regulate hydrogen ion concentration. The carbon
dioxide formed during cellular respiration combines with water to create carbonic acid.
Carbonic acid then dissociates into bicarbonate and a hydrogen ion. This reaction is one
of the many buffer systems in the human body; it resists dramatic changes in pH to allow
a person to remain within the narrow physiological pH range.
○ Option A: The renal system affects pH by reabsorbing bicarbonate and excreting
fixed acids. Whether due to pathology or necessary compensation, the kidney
excretes or reabsorbs these substances which affect pH. The nephron is the
functional unit of the kidney. Blood vessels called glomeruli transport substances
found in the blood to the renal tubules so that some can be filtered out while
others are reabsorbed into the blood and recycled.
○ Option B: This reaction can and does occur without an enzyme; however,
carbonic anhydrase is an enzyme that assists with this process. It catalyzes the
first reaction above to form carbonic acid which can then freely dissociate into
bicarbonate and a hydrogen ion. Carbonic anhydrase is located in red blood
cells, renal tubules, gastric mucosa, and pancreatic cells.
○ Option D: If bicarbonate is reabsorbed and/or acid is secreted into the urine, the
pH becomes more alkaline (increases). When bicarbonate is not reabsorbed or
acid is not excreted into the urine, pH becomes more acidic (decreases). The
metabolic compensation from the renal system takes longer to occur: days rather
than minutes or hours.
45. 23. Question
The respiratory system regulates acid-base balance by:
○ A. Increasing mucus production.
○ B. Changing the rate and depth of respiration.
○ C. Forming bicarbonate.
○ D. Reabsorbing bicarbonate.
46. Incorrect
Correct Answer: B. Changing the rate and depth of respiration
Through changes in the rate and depth of respiration, the acid-base balance is achieved
via CO2 elimination and retention. The pulmonary system adjusts pH using carbon
dioxide; upon expiration, carbon dioxide is projected into the environment. C and D are
responses that refer to ways in which kidneys balance acids and bases.
○ Option A: Mucus production is not part of the pulmonary regulatory system. Due
to carbon dioxide forming carbonic acid in the body when combining with water,
the amount of carbon dioxide expired can cause pH to increase or decrease.
When the respiratory system is utilized to compensate for metabolic pH
disturbances, the effect occurs in minutes to hours.
○ Option C: If bicarbonate is reabsorbed and/or acid is secreted into the urine, the
pH becomes more alkaline (increases). When bicarbonate is not reabsorbed or
acid is not excreted into the urine, pH becomes more acidic (decreases). The
metabolic compensation from the renal system takes longer to occur: days rather
than minutes or hours.
○ Option D: The renal system affects pH by reabsorbing bicarbonate and excreting
fixed acids. Whether due to pathology or necessary compensation, the kidney
excretes or reabsorbs these substances which affect pH. The nephron is the
functional unit of the kidney.
47. 24. Question
Which of the following is a gas component of the ABG measurement?
○ A. Carbon dioxide
○ B. Bicarbonate
○ C. Hydrogen
○ D. pH
48. Correct
Correct Answer: A. Carbon dioxide
The gases measured by ABGs are oxygen and carbon dioxide. Bicarbonate and
hydrogen are ions; their ratio is measured in the pH. An arterial blood gas (ABG) tests
explicitly blood taken from an artery. ABG analysis assesses a patient’s partial pressure
of oxygen (PaO2) and carbon dioxide (PaCO2).
○ Option B: The measured HCO3 uses a strong alkali that liberates all CO2 in
serum, including dissolved CO2, carbamino compounds, and carbonic acid. The
calculation only accounts for dissolved CO2; this measurement using a standard
chemistry analysis will likely be called a “total CO2”.
○ Option C: Hydrogen is not present in blood as gas and, therefore, does not exert
partial pressure. However, pH, which measures hydrogen ion activity, is a
conventional part of every arterial blood gas determination. The normal range for
blood pH is 7.35 to 7.45.
○ Option D: The pH electrode measures the potential difference between a
measuring electrode (which contains the sample in contact with a special glass
membrane permeable only to H+ ions) and a reference electrode (which has a
known, stable pH). From the voltage across these electrodes, the sample pH is
calculated.
49. 25. Question
Chloride helps maintain acid-base balance by performing which of the following roles?
○ A. Participating in the chloride shift.
○ B. Following sodium to maintain serum osmolarity.
○ C. Maintaining the balance of cations in the ICF and ECF.
○ D. Separating carbonic acid.
50. Incorrect
Correct Answer: A. Participating in the chloride shift.
To maintain acid-base balance, chloride shifts into and out of red blood cells in exchange
for bicarbonate. In the presence of an acid-base imbalance, chloride levels change
independently of sodium. Because of its inverse relationship with bicarbonate, chloride
losses result in an increase in bicarbonate, and chloride gains result in a decrease in
bicarbonate.
○ Option B: Sodium ions are reabsorbed at the membrane, and hydrogen ions are
expelled into the filtrate. The hydrogen ions combine with bicarbonate, forming
carbonic acid, which dissociates into CO2 gas and water.
○ Option C: The kidneys help maintain the acid–base balance by excreting
hydrogen ions into the urine and reabsorbing bicarbonate from the urine.
Acid–base homeostasis concerns the proper balance between acids and bases;
it is also called body pH. The body is very sensitive to its pH level, so strong
mechanisms exist to maintain it.
○ Option D: This reaction can and does occur without an enzyme; however,
carbonic anhydrase is an enzyme that assists with this process. It catalyzes the
first reaction above to form carbonic acid which can then freely dissociate into
bicarbonate and a hydrogen ion. Carbonic anhydrase is located in red blood
cells, renal tubules, gastric mucosa, and pancreatic cells.
51. 26. Question
Which of the following hormones helps regulate chloride reabsorption?
○ A. Antidiuretic hormone
○ B. Renin
○ C. Estrogen
○ D. Aldosterone
52. Incorrect
Correct Answer: D. Aldosterone
Chloride reabsorption depends on sodium reabsorption, which is regulated by
aldosterone in the distal tubule and collecting ducts. It affects blood pressure by
regulating the amount of sodium (and the chloride that diffuses with sodium across the
membranes) by increasing or decreasing the total amount of volume in the extracellular
fluid (ECF).
○ Option A: Antidiuretic hormone stimulates water reabsorption by stimulating
insertion of “water channels” or aquaporins into the membranes of kidney
tubules. These channels transport solute-free water through tubular cells and
back into blood, leading to a decrease in plasma osmolarity and an increased
osmolarity of urine.
○ Option B: It plays an essential role in the rate-limiting step of the
renin-angiotensin-aldosterone system (RAAS), responsible for the homeostasis
of blood volume and mean arterial blood pressure. Renin also acts as a
hormone, binding to pro-renin receptors, causing an increase in the conversion of
angiotensinogen to angiotensin I.
○ Option C: Estrogen is a steroid hormone associated with the female reproductive
organs and is responsible for the development of female sexual characteristics.
Estrogen or estradiol is the most common form of estrogen hormone for
FDA-approved treatment as hormone replacement therapy (HRT) in the
management of symptoms associated with menopause.
53. 27. Question
Chloride is absorbed in the:
○ A. Stomach
○ B. Bowel
○ C. Liver
○ D. Kidney
54. Correct
Correct Answer: B. Bowel
Chloride is absorbed in the bowel, mainly the duodenum and jejunum. Sodium chloride
is absorbed from the intestinal lumen by several mechanisms, most prominently by
cotransport with glucose and amino acids, and by Na+/H+ exchange, both of which
move sodium from the lumen into the enterocyte.
○ Option A: Absorbed sodium is rapidly exported from the cell via sodium pumps –
when a lot of sodium is entering the cell, a lot of sodium is pumped out of the cell,
which establishes a high osmolarity in the small intercellular spaces between
adjacent enterocytes.
○ Option C: Levels of chlorine closely parallel levels of sodium intake and output,
since a primary source of both is sodium chloride, or common table salt. Chlorine
is stored to a limited extent in the skin, subcutaneous tissues, and skeleton and
constitutes two-thirds of the negatively charged ions (anions) in the blood.
○ Option D: The majority of the filtered chloride is reabsorbed with sodium during
transport through the first portion of the tubule, the proximal tubule. The
reabsorption of chloride in this segment occurs in two phases. Both of these
active processes raise the intraluminal chloride concentration thus producing, in
the later segments of the proximal tubule, the passive movement of chloride
along a favorable concentration and electrochemical gradient.
55. 28. Question
When chloride concentration drops below 95 mEq/L, reabsorption of which of the
following electrolytes increases proportionally?
○ A. Hydrogen
○ B. Potassium
○ C. Sodium
○ D. Bicarbonate
56. Incorrect
Correct Answer: D. Bicarbonate
When chloride concentrations drop below 95 mEq/L, bicarbonate reabsorption increases
proportionally, causing metabolic alkalosis. Other choices are cations, chloride is an
anion; a cation must always exchange for a cation in order to maintain electrical
neutrality.
○ Option A: Hydrogen is used within the kidneys as an antiporter energy gradient to
retain a multitude of other elements. Of interest here, sodium is reabsorbed
through an exchange for hydrogen in the renal collecting ducts under the
influence of aldosterone.
○ Option B: Anytime that hydrogen ions are shifted intracellularly, this imbalance in
the buffer system has a relative increase in bicarbonate. Processes that drive
hydrogen intracellularly include hypokalemia.
○ Option C: Severe vomiting may lead to the most disproportionate loss of chloride
compared to sodium since gastric chloride content is greater than 100 mEq/L and
gastric sodium content is relatively low (20 to 30 mEq/L). In individuals with
protracted vomiting or nasogastric suction, the serum sodium concentration may
be only mildly depressed (130 mEq/L), whereas the serum chloride concentration
is usually markedly lowered (80 to 90 mEq/L).
57. 29. Question
Jonas is admitted with 1,000 ml of diarrhea per day for the last 3 days. An IV of 0.45%
NaCl mixed with 5% dextrose is infusing. Which of the following nursing interventions is
the most appropriate?
○ A. Get an infusion controller from the central supply.
○ B. Mix all antibiotics in 0.45% NaCl with 5% dextrose.
○ C. Check the patient’s potassium level and contact the doctor for IV additive
orders.
○ D. Assess the patient for signs of hyperkalemia.
58. Incorrect
Correct Answer: C. Check the patient’s potassium level and contact the doctor for IV
additive orders.
Potassium is lost via the GI and renal systems. Prolonged or excessive diarrhea can
lead to hypokalemia. In the event of hypokalemia, a potassium additive would likely be
prescribed. When a client experiences diarrhea, the stomach contents which is high in
potassium get flushed out of the gastrointestinal tract into the stool and out of the body,
resulting in hypokalemia.
○ Option A: Encourage increased fluid intake of 1.5 to 2.5 liters/24 hour plus 200 ml
for each loose stool in adults unless contraindicated. Increased fluid intake
replaces fluid lost in liquid stools.
○ Option B: Administer antidiarrheal medications as prescribed. Bismuth salts,
kaolin, and pectin which are adsorbent antidiarrheals are commonly used for
treating the diarrhea of gastroenteritis. These drugs coat the intestinal wall and
absorb bacterial toxins.
○ Option D: Assess for abdominal pain, abdominal cramping, hyperactive bowel
sounds, frequency, urgency, and loose stools. These assessment findings are
commonly connected with diarrhea.
59. 30. Question
Mrs. Waltraud is receiving digoxin and Lasix daily. Today, she complains of nausea, and
her apical pulse is 130 and irregular. Which of the following nursing interventions is the
most appropriate?
○ A. Hold the digoxin and check the patient’s potassium level.
○ B. Remove the orange juice from the patient’s tray.
○ C. Identify the patient as high risk for hyperkalemia.
○ D. Assess the patient for other signs of hypernatremia.
60. Incorrect
Correct Answer: A. Hold the digoxin and check the patient’s potassium level.
The patient is experiencing hypokalemia and is at risk for digitalis toxicity. Nausea and
irregular pulse are signs of digitalis toxicity. Electrolytes must also be evaluated;
hypokalemia, hypercalcemia, and hypomagnesemia are known to worsen the effects of
toxicity. The inhibition of the sodium-potassium ATPase leads to hyperkalemia and can
be used as a marker of toxicity severity.
○ Option B: Hydration, oxygenation, and close monitoring are necessary. The ECG
has to be continuously monitored for dysrhythmias. All electrolyte disturbances
need to be corrected. Digoxin concentration does not necessarily correlate with
clinical symptoms of toxicity however digoxin concentrations may be used for
calculating the amount of antidote therapy.
○ Option C: The most common trigger of digoxin toxicity is hypokalemia, which may
occur as a result of diuretic therapy. Dosing errors are also a common cause of
toxicity in the younger population.
○ Option D: Clinical signs of toxicity include gastrointestinal, neurological and the
most concerning cardiac. Most symptoms are nonspecific findings and include a
headache, malaise, insomnia, altered mental status, abdominal pain, nausea,
and vomiting.

1. 1. Question
The type of fluid used to manipulate fluid shifts among compartments states is:
○ A. Whole blood
○ B. TPN
○ C. Albumin
○ D. Ensure
2. Incorrect
Correct Answer: C. Albumin
Albumin is a colloid that is used to manipulate fluid shifts among compartments. Albumin
is also a colloid fluid administered to patients in need of fluid resuscitation, especially in
the setting of trauma (i.e. hypovolemic shock) or in the setting of large-volume
paracentesis. Strength albumin has over crystalloids is that it leads to an increase in
intravascular oncotic pressure. There are some situations in which a patient needs
improved oncotic pressure, and this characteristic can be advantageous.
○ Option A: Whole blood is used to replace blood volume. Whole Blood is the
simplest, most common type of blood donation. It’s also the most flexible
because it can be transfused in its original form, or used to help multiple people
when separated into its specific components of red cells, plasma, and platelets.
○ Option B: TPN is used for patients who are unable to take in food or fluid. Total
parenteral nutrition (TPN) supplies all daily nutritional requirements. TPN can be
used in the hospital or at home. Because TPN solutions are concentrated and
can cause thrombosis of peripheral veins, a central venous catheter is usually
required.
○ Option D: Ensure is a high-calorie nutritional supplement; it is not used to
manipulate fluid shifts. It contains well-balanced proportions of macronutrients
that conform to guidelines for Dietary Reference Intake and the latest American
Heart Association Guidelines for healthy diets.
3. 2. Question
Mr. Miyazaki, who is diagnosed with bipolar disorder has been drinking copious amounts
of water and voiding frequently. The patient is experiencing muscle cramps, twitching,
and is reporting dizziness. the nurse checks lab work for:
○ A. Complete blood count results, particularly the platelets.
○ B. Electrolytes, particularly the serum sodium.
○ C. Urine analysis, particularly for the presence of white blood cells.
○ D. EEG results
4. Correct
Correct Answer: B. Electrolytes, particularly the serum sodium.
The patient is exhibiting behavior that could lead to a sodium and water imbalance and
is exhibiting signs of hyponatremia. The nurse would check the electrolytes with attention
to the sodium level. Monitor serum and urine electrolytes and osmolality. Evaluates
therapy needs and effectiveness.
○ Option A: The monitoring of platelet quantity and function is frequently useful in
evaluating the bleeding risk in hospitalized patients. In healthy patients, platelets
are incredibly numerous, with a range of 150 to 350 x10/L. A drop in this number
can indicate the consumption of platelets by a condition such as disseminated
intravascular coagulation, or autoimmune destruction of platelets, as in immune
thrombocytopenia.
○ Option C: When this test is positive and/or the WBC count in urine is high, it may
indicate that there is inflammation in the urinary tract or kidneys. The most
common cause for WBCs in urine (leukocyturia) is a bacterial urinary tract
infection (UTI), such as a bladder or kidney infection.
○ Option D: An electroencephalogram (EEG) is an essential tool that studies the
brain’s electrical activity. Despite the development of more advanced imaging
techniques, EEG remains the essential paraclinical tool for seizure evaluation. It
is primarily used to assess seizures and conditions that may mimic seizures.
5. 3. Question
When teaching a patient about foods high in magnesium, the nurse would include:
○ A. Green vegetables
○ B. Butter
○ C. Cheese
○ D. Tomatoes
6. Correct
Correct Answer: A. Green vegetables
Green vegetables are high in magnesium. Leafy greens are extremely healthy, and
many are loaded with magnesium. Greens with significant amounts of magnesium
include kale, spinach, collard greens, turnip greens, and mustard greens. For instance, a
1-cup serving of cooked spinach has 157 mg of magnesium or 39% of the RDI (44).
○ Option B: Butter is rich in nutrients like bone-building calcium and contains
compounds linked to lower chances of obesity. Butter can also be part of a
low-carbohydrate diet, which may help people better maintain their weight or lose
weight quicker than they would with a low-fat diet.
○ Option C: Cheese is a great source of calcium, fat, and protein. It also contains
high amounts of vitamins A and B-12, along with zinc, phosphorus, and riboflavin.
Cheese made from the milk of 100 percent grass-fed animals is the highest in
nutrients and also contains omega-3 fatty acids and vitamin K-2.
○ Option D: Tomatoes are the major dietary source of the antioxidant lycopene,
which has been linked to many health benefits, including reduced risk of heart
disease and cancer. They are also a great source of vitamin C, potassium, folate,
and vitamin K.
7. 4. Question
The balance of anions and cations as it occurs across cell membranes is known as:
○ A. Osmotic activity
○ B. Electrical neutrality
○ C. Electrical stability
○ D. Sodium-potassium pump
8. Incorrect
Correct Answer: B. Electrical neutrality
Electrical neutrality refers to a state in which the same number of positively charged ions
and negatively charged ions are present on either side of the membrane. The amount of
charge on a single proton is equal to the amount of charge possessed by a single
electron. A proton and an electron have an equal amount but an opposite type of charge.
Thus, if an atom contains equal numbers of protons and electrons, the atom is described
as being electrically neutral.
○ Option A: Osmotic activity refers to the attraction of a solute to a solvent. Osmotic
activity is a passive process in which water diffuses across a cell membrane in
response to a concentration gradient.
○ Option C: The fundamental structure of the membrane is the phospholipid bilayer,
which forms a stable barrier between two aqueous compartments. In the case of
the plasma membrane, these compartments are the inside and the outside of the
cell.
○ Option D: Sodium-potassium pump refers to the exchange of electrolytes. The
sodium and potassium move against the concentration gradients. The Na+
K+-ATPase pump maintains the gradient of a higher concentration of sodium
extracellularly and a higher level of potassium intracellularly.
9. 5. Question
Maria, an 85-year-old patient with a feeding tube, has been experiencing severe watery
stool. The patient is lethargic and has poor skin turgor, a pulse of 120, and hyperactive
reflexes. Nursing interventions would include:
○ A. Measuring and recording intake and output and daily weights.
○ B. Administering salt tablets and monitoring hypertonic parenteral solutions.
○ C. Administering sedatives.
○ D. Applying wrist restraints to avoid displacement of the feeding tube.
10. Incorrect
Correct Answer: A. Measuring and recording intake and output and daily weights.
The patient is exhibiting signs of hypernatremia and dehydration. The most appropriate
nursing intervention is to measure and record intake and output and daily weight.
Monitor intake and output and specific gravity. Assess the presence and location of
edema. Weigh the client daily. These parameters are variable, depending on the fluid
status, and are indicators of therapy needs and effectiveness.
○ Option B: Restrict sodium intake and administer diuretics as indicated. Sodium
intake restriction while promoting renal clearance decreases serum sodium levels
in the presence of extracellular fluid excess.
○ Option C: Monitor level of consciousness and muscular strength, tone, and
movement. Sodium imbalances may cause changes that vary from irritability and
confusion to seizures and coma. In the presence of a water deficit, rapid
rehydration may cause cerebral edema.
○ Option D: Provide safety and seizure precaution as indicated: bed in a low
position and use of padded side rails. Cerebral edema and sodium excess
increase the risk of convulsions.
11. 6. Question
Disease of which of the following structures is most likely to affect electrolyte
reabsorption?
○ A. Glomerulus
○ B. Renal tubules
○ C. Bladder
○ D. Renal pelvis
12. Correct
Correct Answer: B. Renal tubules
The renal tubules are the site of electrolyte reabsorption. From the PCT, the
non-reabsorbed filtrates move on to the nephron loop. The nephron loop functionally
divides into a descending and an ascending limb. In the ascending limb, Na moves
passively down its concentration gradient in the thin segment of the ascending limb, and
also sodium, potassium, and chlorides get reabsorbed together through a symporter in
the thick segment of the ascending limb.
○ Option A: The glomerulus is the site of electrolyte filtration. Glomerular filtration is
the initial process in urine production. It is a passive process in which hydrostatic
pressure pushes fluid and solutes through a membrane with no energy
requirement.
○ Option C: The bladder is where the urine is stored. The bladder is essentially a
muscular sac with three layers. Its three layers are similar to the ureter except
that the muscular layer has muscle fibers organized in inner and outer
longitudinal layers and a middle circular layer.
○ Option D: The renal pelvis is where urine travels as it moves from the collecting
ducts to the ureter. Once the production of urine is complete, it travels through a
structure called ureter for urine storage in the bladder. There are two ureters in a
human body; one on each side; left and right.
13. 7. Question
Christoff is diagnosed with hypermagnesemia. Symptoms of her condition may include:
○ A. Hypertension
○ B. Tachycardia
○ C. Hyperactive deep-tendon reflex
○ D. Cardiac arrhythmias
14. Incorrect
Correct Answer: D. Cardiac arrhythmias
Cardiac arrhythmias are associated with hypermagnesemia. For higher values (over
12.0 mg/dL) muscle paralysis, paralytic ileus, decreased breathing rate, low blood
pressure, electrocardiogram (ECG) changes including an increase in PR and QRS
interval with sinus bradycardia, and atrioventricular block, coma and cardiac arrest
(exceeding 15.0 mg/dL) may occur. Hypertension, tachycardia, and hyperactive reflexes
are signs of hypomagnesemia.
○ Option A: A slight reduction in blood pressure and blurred vision caused by
diminished accommodation and convergence can manifest. Patients with
symptomatic hypermagnesemia can present different clinical manifestations
depending on the level and the time in which the electrolytic disturbance has
occurred.
○ Option B: In severe hypermagnesemia (greater than 12 mmol/dL), there is
muscle flaccid paralysis, decreased breathing rate, more evident hypotension
and bradycardia, prolongation of the P-R interval, atrioventricular block, and
lethargy are common. Coma and cardiorespiratory arrest can occur for higher
values (over 15 mg/dL).
○ Option C: The most frequent symptoms and signs may include weakness,
nausea, dizziness, and confusion (less than 7.0 mg/dL). Increasing values (7 to
12 mg/dL) induce decreased reflexes, worsening confusional state, drowsiness,
bladder paralysis, flushing, headache, and constipation.
15. 8. Question
Daniel who is a marathon runner is at high risk for fluid volume deficit. Which one of the
following is a related factor?
○ A. Decreased diuresis
○ B. Disease-related process
○ C. Decreased breathing and perspiration
○ D. Increased breathing and perspiration
16. Incorrect
Correct Answer: D. Increased breathing and perspiration
Excessive fluid can be lost if breathing and perspiration are at an increased rate for a
prolonged period. Identify the possible cause of the fluid disturbance or imbalance.
Establishing a database of history aids accurate and individualized care for each patient.
○ Option A: Assess color and amount of urine. Report urine output less than 30
ml/hr for 2 consecutive hours. A normal urine output is considered normal not
less than 30ml/hour. Concentrated urine denotes fluid deficit.
○ Option B: Monitor for the existence of factors causing deficient fluid volume (e.g.,
gastrointestinal losses, difficulty maintaining oral intake, fever, uncontrolled type II
diabetes mellitus, diuretic therapy). Early detection of risk factors and early
intervention can decrease the occurrence and severity of complications from
deficient fluid volume. The gastrointestinal system is a common site of abnormal
fluid loss.
○ Option C: Oral fluid replacement is indicated for mild fluid deficit and is a
cost-effective method for replacement treatment. Older patients have a
decreased sense of thirst and may need ongoing reminders to drink. Being
creative in selecting fluid sources (e.g., flavored gelatin, frozen juice bars, sports
drink) can facilitate fluid replacement. Oral hydrating solutions (e.g., Rehydralyte)
can be considered as needed.
17. 9. Question
Jomarick is diagnosed with FVD; which of the following nursing diagnoses might apply to
his condition?
○ A. Altered urinary elimination
○ B. Decreased cardiac output
○ C. Increased cardiac output
○ D. Vomiting
18. Correct
Correct Answer: B. Decreased cardiac output
Decreased cardiac output is a nursing diagnosis associated with isotonic FVD. Decrease
in circulating blood volume can cause hypotension and tachycardia. Alteration in HR is a
compensatory mechanism to maintain cardiac output. Usually, the pulse is weak and
may be irregular if electrolyte imbalance also occurs. Hypotension is evident in
hypovolemia. Other appropriate nursing diagnoses include altered tissue perfusion,
potential for injury, and ineffective breathing pattern.
○ Option A: Assess color and amount of urine. Report urine output less than 30
ml/hr for 2 consecutive hours. A normal urine output is considered normal not
less than 30ml/hour. Concentrated urine denotes fluid deficit. Teach family
members how to monitor output in the home. Instruct them to monitor both intake
and output.
○ Option C: Cardiac alterations like dysrhythmias may reflect hypovolemia and/or
electrolyte imbalance, commonly hypocalcemia. Note: MI, pericarditis, and
pericardial effusion with/ without tamponade are common cardiovascular
complications.
○ Option D: Monitor active fluid loss from wound drainage, tubes, diarrhea,
bleeding, and vomiting; maintain accurate input and output record. Fluid loss
from wound drainage, diarrhea, bleeding, and vomiting causes decreased fluid
volume and can lead to dehydration.
19. 10. Question
Body fluids perform which of the following functions?
○ A. Transport nutrients
○ B. Transport electrical charges
○ C. Cushion the organs
○ D. Facilitate fat metabolism
20. Incorrect
Correct Answer: A. Transport nutrients
Body fluids facilitate the transport of nutrients, hormones, proteins, and other molecules.
Fluid moves throughout cellular environments in the body by passively crossing
semipermeable membranes. The exact chemical composition of body fluid is highly
variable. This is dependent on which portion of the body, as well as which organ of the
body, contains the fluid.
○ Option B: All body fluids should have an ionic net electrical charge close to zero,
indicating a balance of cations and anions. Ionic components will diffuse through
fluids selectively depending on the presence of permeable membranes. If a
membrane is non-permeable to an ion, this creates a gradient of relatively higher
concentration osmolarity.
○ Option C: The distribution of fluid throughout the body can be broken down into
two general categories: intracellular fluid and extracellular fluid. Intracellular fluid
is approximately 40% of the total body weight. It is the total space within cells
primarily defined as the cytoplasm of cells.
○ Option D: Extracellular fluid and interstitial fluid are similar in composition.
Extracellular spaces contain high concentrations of sodium, chloride,
bicarbonate, and proteins but are relatively lower in potassium, magnesium, and
phosphate. Interstitial fluids physiologically tend to have a low concentration of
proteins. Intracellular fluids tend to be inversed with high levels of phosphate,
magnesium, potassium, and proteins but lower sodium, chloride, and
bicarbonate.
21. 11. Question
Sodium levels are affected by the secretion of which of the following hormones?
○ A. Progesterone and aldosterone
○ B. ADH and ACTH
○ C. Antidiuretic hormone and FSH
○ D. ECF and aldosterone
22. Correct
Correct Answer: B. ADH and ACTH
The endocrine system secretes aldosterone and ADH to help regulate sodium levels.
The pituitary secretes adrenocorticotropic hormone to help regulate sodium. Vasopressin
or antidiuretic hormone (ADH) or arginine vasopressin (AVP) is a nonapeptide
synthesized in the hypothalamus. Science has known it to play essential roles in the
control of the body’s osmotic balance, blood pressure regulation, sodium homeostasis,
and kidney functioning. Adrenocorticotropic hormone (ACTH) is a tropic hormone
produced by the anterior pituitary. The hypothalamic-pituitary axis controls it. ACTH
regulates cortisol and androgen production.
○ Option A: Progesterone is a reproductive hormone. Progesterone is an
endogenous steroid hormone that is commonly produced by the adrenal cortex
as well as the gonads, which consist of the ovaries and the testes. Progesterone
is also secreted by the ovarian corpus luteum during the first ten weeks of
pregnancy, followed by the placenta in the later phase of pregnancy.
○ Option C: FSH is a reproductive hormone. Follicle-stimulating hormone (FSH) is
a hormone produced by the anterior pituitary in response to
gonadotropin-releasing hormone (GnRH) from the hypothalamus. FSH plays a
role in sexual development and reproduction in both males and females.
○ Option D: ECF is not a hormone. Extracellular fluid (ECF) denotes all body fluid
outside the cells of any multicellular organism. The main component of the
extracellular fluid is the interstitial fluid that surrounds cells.
23. 12. Question
Bicarbonate is lost during which of the following clinical conditions?
○ A. Diarrhea
○ B. Diuresis
○ C. Diaphoresis
○ D. Vomiting
24. Incorrect
Correct Answer: A. Diarrhea
Bicarbonate is lost in diarrhea because the lower intestinal tract contains fluids rich in
bicarbonate. In pathologies with profuse watery diarrhea, bicarbonate within the
intestines is lost through the stool due to increased motility of the gut. This leads to
further secretion of bicarbonate from the pancreas and intestinal mucosa, leading to net
acidification of the blood from bicarbonate loss.
○ Option B: Metabolic alkalosis occurs when a large volume of sodium-rich,
bicarbonate low fluid is lost from the body. This occurs with diuretic use, cystic
fibrosis, congenital chloride diarrhea, among others. The net concentration of
bicarbonate increases as a result.
○ Option C: The sweat duct also reabsorbs bicarbonate, either directly or through
hydrogen ion secretion, but the specific mechanism is unknown. The activity of
Na-K-ATPase is influenced by the hormonal control of aldosterone. Overall the
rate of Na, Cl, and bicarbonate reabsorption is also flow-dependent, such that
higher sweating rates are associated with proportionally lower reabsorption rates
resulting in higher final sweat electrolyte concentrations
○ Option D: Although vomiting induces gastric losses of acidic ions, there was no
difference in serum bicarbonate concentrations between patients with and
without vomiting. It is therefore likely that the reduction in serum bicarbonate
concentration represents a true metabolic acidosis.
25. 13. Question
Magnesium reabsorption is controlled by:
○ A. Loop of Henle
○ B. Glomerulus
○ C. Pituitary
○ D. Parathyroid hormone
26. Correct
Correct Answer: A. Loop of Henle
The Loop of Henle is responsible for magnesium reabsorption. The loop of Henle
appears to be the major nephron site where magnesium reabsorption is controlled. The
principal factors which alter magnesium reabsorption in the loop include parathyroid
hormone, changes in plasma magnesium and calcium concentration and loop diuretics.
○ Option B: The glomerulus, in contrast to the majority of other capillary beds, sits
between two arterioles; receiving blood supply from the upstream afferent
arteriole, and blood exiting downstream via the efferent arterioles (E for exit).
○ Option C: The pituitary endocrine gland, which is located in the bony sella turcica,
is attached to the base of the brain and has a unique connection with the
hypothalamus. The pituitary gland consists of two anatomically and functionally
distinct regions, the anterior lobe (adenohypophysis) and the posterior lobe
(neurohypophysis). Between these lobes lies a small region called the
intermediate lobe. The hypothalamus regulates the pituitary gland secretion
○ Option D: The parathyroid gland secretes parathyroid hormone (PTH), a
polypeptide, in response to low calcium levels detected in the blood. PTH
facilitates the synthesis of active vitamin D, calcitriol
(1,25-dihydroxycholecalciferol, or vitamin D3) in the kidneys. In conjunction with
calcitriol, PTH regulates calcium and phosphate.
27. 14. Question
Annaliza has a nursing diagnosis of fluid volume deficit. Which one of the following
medications could potentially exacerbate the problem?
○ A. Synthroid
○ B. Digoxin
○ C. Lasix
○ D. Insulin
28. Incorrect
Correct Answer: C. Lasix
Lasix will contribute to fluid loss through its action as a diuretic. The diuretic effect of
furosemide can cause depletion of sodium, chloride, body water, and other minerals.
Therefore, careful medical supervision is necessary during treatment. Furosemide
(Lasix) is a potent diuretic (water pill) that is used to eliminate water and salt from the
body. In the kidneys, salt (composed of sodium and chloride), water, and other small
molecules normally are filtered out of the blood and into the tubules of the kidney. The
filtered fluid ultimately becomes urine.
○ Option A: Oral levothyroxine is primarily indicated for the treatment of primary,
secondary, and tertiary hypothyroidism. Levothyroxine (T4) is a synthetic version
of one of the body’s natural thyroid hormones: thyroxine (T4). Normally, the
hypothalamus secretes thyrotropin-releasing hormone (TRH), which then
stimulates the anterior pituitary to secrete thyroid-stimulating hormone (TSH),
which subsequently stimulates the thyroid to secrete 80% thyroxine (T4) and
20% L-triiodothyronine (T3).
○ Option B: Digoxin increases the force of contraction of the heart by reversibly
inhibiting the activity of the myocardial Na-K ATPase pump, an enzyme that
controls the movement of ions into the heart. Digoxin has vagomimetic effects on
the AV node. By stimulating the parasympathetic nervous system, it slows
electrical conduction in the atrioventricular node, therefore, decreases the heart
rate.
○ Option D: Insulin is a medication used in the treatment and management of
diabetes mellitus type-1 and sometimes diabetes mellitus type-2, both of which
are a significant risk factor for coronary artery disease, stroke, peripheral
vascular disease, and a host of other vascular conditions.
29. 15. Question
Alexander has hypotonic FVE; which of the following findings would the nurse expect to
assess in the patient?
○ A. Poor skin turgor and increased thirst
○ B. Weight gain and thirst
○ C. Interstitial edema and hypertension
○ D. Hypotension and pitting edema
30. Incorrect
Correct Answer: B. Weight gain and thirst
Weight gain and thirst are symptoms of hypotonic FVE; other symptoms include the
excretion of dilute urine, non-pitting edema, dysrhythmias, and hyponatremia. Monitor for
peripheral edema, pulmonary edema, or hepatomegaly. It is important to consider
underlying cardiac dysfunction or renal failure and adjust volumes of administration
accordingly. These patients might require a lower maintenance fluid rate than expected
for their body weight.
○ Option A: Severe cases of dehydration might present with flaccid or tented skin;
Eyeballs might also appear sunken back into orbital cavities. Classically find cool
and clammy skin found in hypovolemic shock due to peripheral vasoconstriction
causing hypoperfusion of skin, especially at the extremities (i.e., hands or feet)
○ Option C: Peripheral edema can be a sign of volume overload or third spacing of
intravascular fluid. Appreciate a distended jugular vein in volume overload state;
Can also be found in patients with congestive heart failure who are euvolemic but
not pumping blood appropriately
○ Option D: Falling blood pressure is an ominous finding in the setting of
tachycardia, indicating that the cardiovascular system can no longer compensate
adequately for hypovolemia. Conversely, elevated blood pressures can be seen
in hypervolemia.
31. 16. Question
The interstitial space holds approximately how many liters?
○ A. 3 L
○ B. 6 L
○ C. 9 L
○ D. 12 L
32. Incorrect
Correct Answer: C. 9 L
The interstitial space holds 9 L. The extracellular fluid comprises approximately 20% of
total body weight and further sub categorized as plasma at approximately 5% of body
weight and interstitial space which is approximately 12% of body weight.
○ Option A: Extracellular fluid and interstitial fluid are similar in composition.
Extracellular spaces contain high concentrations of sodium, chloride,
bicarbonate, and proteins but are relatively lower in potassium, magnesium, and
phosphate. Interstitial fluids physiologically tend to have a low concentration of
proteins.
○ Option B: The distribution of fluid throughout the body can be broken down into
two general categories: intracellular fluid and extracellular fluid. Intracellular fluid
is approximately 40% of the total body weight. It is the total space within cells
primarily defined as the cytoplasm of cells. In general, intracellular fluids are
stable and do not readily adjust to rapid changes.
○ Option D: Fluid is always in flux through a variety of regulatory mechanisms to
maintain appropriate concentrations throughout the various compartments of the
body. Fluid is largely regulated through passive diffusion following the
concentration gradients of osmotically active solutes; however, hydrostatic
pressures can influence fluid movement between spaces.
33. 17. Question
Sodium balance is important for which of the following functions?
○ A. Transmitting impulses in nerve and muscle fibers via the calcium-potassium
pump.
○ B. Exchanging for magnesium and attracting chloride.
○ C. Combining hydrogen and chloride for acid-base balance.
○ D. Exchanging for potassium and attracting chloride.
34. Incorrect
Correct Answer: D. Exchanging for potassium and attracting chloride.
Sodium influences the levels of potassium and chloride by exchanging for potassium
and attracting chloride. Sodium, which is an osmotically active anion, is one of the most
important electrolytes in the extracellular fluid. It is responsible for maintaining the
extracellular fluid volume, and also for regulation of the membrane potential of cells.
Sodium is exchanged along with potassium across cell membranes as part of active
transport.
○ Option A: The sodium-potassium pump acts to transport sodium and potassium
ions across the cell membrane in a ratio of 3 sodium ions out for every 2
potassium ions brought in. In the process, the pump helps to stabilize membrane
potential, and thus is essential in creating the conditions necessary for the firing
of action potentials.
○ Option B: Magnesium is an intracellular cation. Magnesium is mainly involved in
ATP metabolism, contraction and relaxation of muscles, proper neurological
functioning, and neurotransmitter release. Most of the chloride, which is filtered
by the glomerulus, is reabsorbed by both proximal and distal tubules (majorly by
proximal tubule) by both active and passive transport.
○ Option C: The acid-base status of the blood drives bicarbonate levels. The
kidneys predominantly regulate bicarbonate concentration and are responsible
for maintaining the acid-base balance. Kidneys reabsorb the filtered bicarbonate
and also generate new bicarbonate by net acid excretion, which occurs by
excretion of both titratable acid and ammonia.
35. 18. Question
In renal regulation of water balance, the functions of angiotensin II include:
○ A. Blood clotting within the nephron.
○ B. Increasing progesterone secretion into the renal tubules.
○ C. Catalyzing calcium-rich nutrients.
○ D. Selectively constricting portions of the arteriole in the nephron.
36. Incorrect
Correct Answer: D. Selectively constricting portions of the arteriole in the nephron.
As part of the renal regulation of water balance, angiotensin II selectively constricts
portions of the arteriole in the nephron. ATII is a potent vasopressor, acting on vascular
endothelial receptors. The two types of ATII receptors present in the heart and vascular
smooth muscle that are responsible for signal transduction in mediating the
vasoconstrictive action of ATII are the AT1 and AT2 receptors.
○ Option A: The kidney regulates plasma osmolarity by modulating the amount of
water, solutes, and electrolytes in the blood. It ensures long term acid-base
balance and also produces erythropoietin which stimulates the production of red
blood cells. It also produces renin for blood pressure regulation and carries out
the conversion of vitamin D to its active form.
○ Option B: In the normal circumstance, the PCT reabsorbs all the glucose and
amino acids as well as 65% of Na and water. The PCT reabsorb sodium ions by
primary active transport via a basolateral Na-K pump. It reabsorbs glucose,
amino acids, and vitamins through secondary active transport with Na and an
electrochemical gradient drives passive paracellular diffusion.
○ Option C: The next tubular segment for reabsorption is the distal convoluted
tubule (DCT). There is a primary active sodium transport at the basolateral
membrane and secondary active transport at the apical membrane through Na-Cl
symporter and channels. This process is aldosterone regulated at the distal
portion. There is also calcium reabsorption via passive uptake controlled by the
parathyroid hormone.
37. 19. Question
Which of the following nursing diagnoses might apply to a patient with hypertonic FVE?
○ A. Ineffective airway clearance
○ B. Potential for decreased cardiac output
○ C. Ineffective breathing pattern
○ D. Potential for increased cardiac output
38. Incorrect
Correct Answer: B. Potential for decreased cardiac output
Potential for decreased cardiac output is a nursing diagnosis associated with hypertonic
FVE. Assess for bounding peripheral pulses and S3. These assessment findings are
signs of fluid overload. Check for distended neck veins and ascites. Monitor abdominal
girth to follow any ascites accurately. Distended neck veins are caused by elevated CVP.
Ascites occur when fluid accumulates in extravascular spaces.
○ Option A: Assess for crackles in the lungs, changes in respiratory pattern,
shortness of breath, and orthopnea. These signs are caused by an accumulation
of fluid in the lungs.
○ Option C: Review the patient’s history to determine the probable cause of the
fluid imbalance. Such information can assist in direct management. History may
include increased fluids or sodium intake.
○ Option D: Monitor input and output closely. Dehydration may be the result of fluid
shifting even if overall fluid intake is adequate. In some patients with heart failure,
the weight may be a poor indicator of fluid volume status. Poor nutrition and
decreased appetite over time result in a decrease in weight, which may be
accompanied by fluid retention even though the net weight remains unchanged.
39. 20. Question
The intracellular compartment holds water and:
○ A. Proteins
○ B. Glucose
○ C. Sodium
○ D. Uric acid
40. Incorrect
Correct Answer: A. Proteins
The intracellular compartment holds large amounts of water and proteins. Potassium,
lipids, and nucleic acids are also components of the intracellular compartment.
Intracellular fluids tend to be inversed with high levels of phosphate, magnesium,
potassium, and proteins but lower sodium, chloride, and bicarbonate. Interstitial fluids
physiologically tend to have a low concentration of proteins.
○ Option B: Intracellular fluid is approximately 40% of the total body weight. It is the
total space within cells primarily defined as the cytoplasm of cells. In general,
intracellular fluids are stable and do not readily adjust to rapid changes. This
space is where much of chemical reactions occur, as such, it is important to
maintain an appropriate osmolality.
○ Option C: Extracellular fluid and interstitial fluid are similar in composition.
Extracellular spaces contain high concentrations of sodium, chloride,
bicarbonate, and proteins but are relatively lower in potassium, magnesium, and
phosphate.
○ Option D: Uric acid is normally found in the body as a byproduct of the way the
body breaks down certain proteins called purines. Causes of an elevated blood
uric acid level (hyperuricemia) include genetics, obesity, certain medications such
as diuretics (water pills), and chronic decreased kidney function (kidney disease).
41. 21. Question
The majority gastrointestinal reabsorption of water occurs in:
○ A. Small intestines
○ B. The esophagus
○ C. The colon
○ D. The stomach
42. Correct
Correct Answer: A. Small intestines
Approximately 85% to 95% of water absorption takes place in the small intestine. The
colon absorbs only 500 to 100 cc. Approximately 9 liters of water travels to the
gastrointestinal tract per day, and the small intestine absorbs 7 to 8 liters, while the colon
absorbs the remaining 1 to 2 liters. Water absorption is thought to occur via osmotic
gradients and aquaporins on intestinal membranes.
○ Option B: Absorption in the esophagus is virtually nil. The mucosa does contain
mucous glands that are expressed as foodstuffs distend the esophagus, allowing
mucus to be secreted and aid in lubrication. The body of the esophagus is
bounded by physiologic sphincters known as the upper and lower esophageal
sphincters.
○ Option C: By the time indigestible materials have reached the colon, most
nutrients and up to 90% of the water have been absorbed by the small intestine.
The role of the ascending colon is to absorb the remaining water and other key
nutrients from the indigestible material, solidifying it to form a stool.
○ Option D: Contrary to popular thought, the stomach does not contribute to the
absorption of any nutrients. This organ can be in the peritoneal cavity, located in
the left upper abdominal quadrant, or in the epigastric abdominal region that acts
to relay ingested food between the nervous system and the endocrine system.
43. 22. Question
Isotonic FVD can result from:
○ A. GI fluid loss through diarrhea.
○ B. Insensible water loss during prolonged fever.
○ C. Inadequate ingestion of fluids and electrolytes.
○ D. Impaired thirst regulation.
44. Incorrect
Correct Answer: C. Inadequate ingestion of fluids and electrolytes.
Isotonic FVD may result from inadequate intake of fluids and electrolytes that can occur
secondary to an inability to ingest orally. Isotonic dehydration is a condition in which both
water and sodium are lost proportionally and the serum sodium concentration maintains
normal serum osmolality. Serum osmolality determines the movement of fluids and
electrolytes across membranes. The normal serum osmolality is 285–295 mOsm/kg.
○ Option A: GI fluid loss through diarrhea is an etiology of hypotonic FVD.
Hypotonic dehydration occurs when sodium loss is greater than water loss,
resulting in a decrease in serum osmolality. This causes a shift of water from the
extracellular space into the intracellular space. The cells swell and cerebral
edema may occur.
○ Option B: Insensible water loss during prolonged fever is a cause of hypertonic
FVD. Fever will increase the respiratory rate and therefore, water loss. Sweating
also increases to lower the body temperature. Water intake is commonly
decreased during a fever which will aggravate dehydration.
○ Option D: Impaired thirst regulation is a cause of hypertonic FVD. Hypertonic
dehydration occurs when water excretion from the body exceeds that of sodium
excretion, resulting in increased sodium concentration in the extracellular fluid
(hypernatremia). Blood osmolality is increased, causing water to shift from the
intracellular to the extracellular space.
45. 23. Question
The majority of the body’s water is contained in which of the following fluid
compartments?
○ A. Intracellular
○ B. Interstitial
○ C. Intravascular
○ D. Extracellular
46. Incorrect
Correct Answer: A. Intracellular
The intracellular compartment holds two-thirds of total body water. At a cellular level, the
distribution of the various fluid compartments in the body is paramount for the
maintenance of health, function, and survival. For the average 70 kg man, 60% of the
total body weight is water, equaling 42L. The body’s fluid separates into two main
compartments: Intracellular fluid volume (ICFV) and extracellular fluid volume (ECFV).
Of the 42L of water found in the body, two-thirds of it is within the intracellular fluid (ICF)
space, which equates to 28L.
○ Option B: The extracellular compartment is the interstitial space plus the
intravascular space. Out of the extracellular fluid volume, 75% or 10.5L of the
volume is present in the interstitial space, and 25% of that water is in the plasma,
which is equivalent to 3.5L.
○ Option C: Plasma is the only fluid compartment that exists as a real fluid
collection all in one space. It differs from the interstitial fluid by its higher protein
content and its function in transportation. Plasma is a component of blood and is
said to be the “interstitial fluid of the blood” as it bathes the suspended red and
white cells, which also reside in the blood.
○ Option D: The extracellular compartment accounts for one-third of total body
water. The ECFV consists of two spaces: The interstitial fluid volume (ISFV) and
the plasma volume (PV). One-third of the total body water is the ECFV, which is
equivalent to 14L.
47. 24. Question
Etiologies associated with hypomagnesemia include:
○ A. Decreased vitamin D intake
○ B. Constipation
○ C. Malabsorption syndrome
○ D. Renal failure
48. Incorrect
Correct Answer: C. Malabsorption syndrome
Malabsorption syndrome is associated with hypomagnesemia. Increased vitamin D
intake and diarrhea are also associated with hypomagnesemia. Magnesium deficiency
and hypomagnesemia can result from a variety of causes including gastrointestinal and
renal losses. Magnesium deficiency can cause a wide variety of features including
hypocalcemia, hypokalemia, and cardiac and neurological manifestations.
○ Option A: Magnesium deficiency entirely due to reduced dietary intake in
otherwise healthy subjects is very uncommon. Hypomagnesemia may be seen in
patients who are maintained on magnesium-free intravenous fluids or total
parenteral nutrition, especially in those patients who have a marginal or reduced
serum magnesium to start off with.
○ Option B: Magnesium deficiency is frequently observed in conditions causing
steatorrhea or severe chronic diarrhea such as Crohn’s disease, ulcerative colitis,
celiac disease, Whipple’s disease, and short bowel syndrome. In general, the
degree of magnesium depletion correlates with the severity of diarrhea, stool fat
content, and fecal magnesium concentration.
○ Option D: Hypomagnesaemia is occasionally observed in chronic renal failure
due to an obligatory renal magnesium loss. It is also seen during the diuretic
phase of acute renal failure, in post-obstructive diuresis, and after renal
transplantation.
49. 25. Question
The danger of fluid sequestered in the third space is that the fluid:
○ A. Is hypertonic and can cause hypervolemia.
○ B. Is hypotonic and can cause water intoxication.
○ C. Is not available for circulation.
○ D. Contains large amounts of acids.
50. Incorrect
Correct Answer: C. Is not available for circulation.
In third-spacing, fluid is sequestered and is unavailable to the general circulation.
Third-spacing occurs when too much fluid moves from the intravascular space (blood
vessels) into the interstitial or “third” space-the nonfunctional area between cells. This
can cause potentially serious problems such as edema, reduced cardiac output, and
hypotension.
○ Option A: Increased fluid volume can be caused by overzealous fluid
replacement or renal dysfunction. Volume overload can lead to peripheral edema,
pulmonary edema, hepatic dysfunction, cerebral edema and mental changes,
and decreased cardiac output. Other signs of fluid overload include jugular vein
distension, hypertension, and a pathologic S3.
○ Option B: Decreased sodium level, or hyponatremia, may result from sodium
loss; for example, gastrointestinal losses during diarrhea or fluid losses caused
by medications such as diuretics. Hyponatremia can also arise from volume
overload. Also called dilutional or hypervolemic hyponatremia, this can occur with
overzealous fluid replacement, heart failure, hepatic cirrhosis, renal disease,
hypothyroidism, or administration of vasopressin.
○ Option D: Albumin losses disrupt colloidal osmotic pressure. Plasma proteins are
crucial to maintaining colloidal osmotic pressure. Albumin, the major protein
constituent of the intravascular space, accounts for up to 60% of total protein.
Any condition that destroys tissue or reduces protein intake can lead to protein
losses and third-spacing.
51. 26. Question
The extracellular fluid space holds water, electrolytes, proteins and:
○ A. Red blood cells
○ B. Potassium
○ C. Lipids
○ D. Nucleic acids
52. Correct
Correct Answer: A. Red blood cells
The extracellular space contains red blood cells, white blood cells, and platelets in
addition to water, electrolytes, and proteins. Extracellular fluid (ECF) or extracellular fluid
volume (ECFV) usually denotes all body fluid outside of cells, and consists of plasma,
interstitial, and transcellular fluid. An extracellular matrix is an extracellular fluid space
containing cell-excreted molecules, and they vary in their type and function. Potassium,
lipids, and nucleic acids are intracellular components.
○ Option B: Potassium is the most abundant exchangeable cation in the body. It
exists predominantly in the intracellular fluid at concentrations of 140 to 150
meq/liter and in the extracellular fluid at concentrations of 3.5 to 5 meq/liter.
○ Option C: Cell membranes are composed of proteins and lipids. Since they are
made up of mostly lipids, only certain substances can move through.
Phospholipids are the most abundant type of lipid found in the membrane.
Phospholipids are made up of two layers, the outer and inner layers.
○ Option D: The two main types of nucleic acids are deoxyribonucleic acid (DNA)
and ribonucleic acid (RNA). DNA is the genetic material found in all living
organisms, ranging from single-celled bacteria to multicellular mammals. It is
found in the nucleus of eukaryotes and in the organelles, chloroplasts, and
mitochondria.
53. 27. Question
Magnesium performs all of the following functions except:
○ A. Contributing to vasoconstriction.
○ B. Assisting in cardiac muscle contraction.
○ C. Facilitating sodium transport.
○ D. Assisting in protein metabolism.
54. Correct
Correct Answer: A. Contributing to vasoconstriction.
Magnesium contributes to vasodilation, not vasoconstriction. Magnesium plays a vital
role in over 300 reactions involving metabolism. It is involved with hormone receptor
binding, muscle contraction, neural activity, neurotransmitter release, vasomotor tone,
and cardiac excitability.
○ Option B: Magnesium acts as a natural calcium channel blocker, and it is a
cofactor of the Na-K-ATP pump. Magnesium helps control atrioventricular node
conduction. Therefore, hypomagnesemia can cause myocardial excitability
resulting in arrhythmias such as ventricular tachycardia and torsades de pointes.
○ Option C: It is necessary for the active transport of potassium and calcium across
the cell membrane. ATP is dependent on magnesium for proper functioning.
Roughly 50% of magnesium is located within the bone, 25% is within the muscle,
and the remainder is in soft tissue, serum, and red blood cells (RBC).
○ Option D: The intestine, bone, and kidney maintain magnesium homeostasis.
Similar to calcium, magnesium is absorbed via the intestine, stored in the bone,
excreted via the kidneys. Absorption of magnesium is inversely proportional to
the concentration within the body; if there are low magnesium levels within the
body, more magnesium will be absorbed.
55. 28. Question
Which of the following clinical conditions exacerbates electrolyte excretion?
○ A. Nasogastric feedings
○ B. Use of surgical drains
○ C. Immobility from fractures
○ D. Chronic water drinking
56. Incorrect
Correct Answer: B. Use of surgical drains
Surgical drains will cause fluid loss, and electrolytes are eliminated along with the fluid.
The role of the potassium ion is ordinarily not a consideration in postoperative fluid
management. It becomes a consideration in the presence of a large amount of drainage
from wounds or abscess cavities, nasogastric suction, or intestinal fistulae. It also must
be given attention in cases in which parenteral administration of fluids is necessary for a
prolonged period.
○ Option A: Gastrointestinal losses, from diarrhea, vomiting, or nasogastric
suctioning, also are common causes of hypokalemia. Vomiting leads to
hypokalemia via a complex pathogenesis. Gastric fluid itself contains little
potassium, approximately 10 mEq/L.
○ Option C: Although the concept of “hyponatremia” is seemingly straightforward
for many clinicians, some challenges remain regarding the diagnostic thresholds,
the distinction between real and pseudohyponatremia, and its relation to human
pathologies, including bone fractures.
○ Option D: Electrolytes (sodium, potassium, magnesium, chloride and calcium)
need to be in balance in order to maintain healthy blood, heart rhythm, muscle
function, and other important functions. Drinking too much water can cause the
electrolyte levels in the body to get out of whack and cause sodium levels
plummet.
57. 29. Question
A diet containing the minimum daily sodium requirement for an adult would be:
○ A. A no-salt diet
○ B. A diet including 2 gm sodium
○ C. A diet including 4 gm sodium
○ D. A 1500 calorie weight-loss diet
58. Incorrect
Correct Answer: B. A diet including 2 gm sodium
The minimum sodium requirement for adults is 2 gm daily. Most adults consume more
than this because sodium is abundant in almost all foods. Sodium is an essential nutrient
and is needed by the body in relatively small amounts(provided that substantial sweating
does not occur) to maintain a balance of body fluids and keep muscles and nerves
running smoothly.
○ Option A: Americans eat on average about 3,400 mg of sodium per day.
However, the Dietary Guidelines for Americans recommends limiting sodium
intake to less than 2,300 mg per day—that’s equal to about 1 teaspoon of salt.
○ Option C: The American Heart Association recommends no more than 2,300
milligrams (mg) a day and moving toward an ideal limit of no more than 1,500 mg
per day for most adults. Because the average American eats so much excess
sodium, even cutting back by 1,000 milligrams a day can significantly improve
blood pressure and heart health.
○ Option D: On average, Americans eat more than 3,400 milligrams of sodium
each day — much more than the American Heart Association and other health
organizations recommend.
59. 30. Question
Which of the following electrolytes are lost as a result of vomiting?
○ A. Bicarbonate and calcium
○ B. Sodium and hydrogen
○ C. Sodium and potassium
○ D. Hydrogen and potassium
60. Incorrect
Correct Answer: D. Hydrogen and potassium
In upper gastrointestinal fluid loss, hydrogen and potassium are lost because these
electrolytes are present in abundance in the stomach. The vomiting of gastric or
intestinal contents most commonly involves the loss of fluid that contains chloride,
potassium, sodium, and bicarbonate. The sequelae of these losses include dehydration
along with hyponatremia, hypochloremia, and hypokalemia.
○ Option A: Low serum magnesium can be commonly associated with
hypocalcemia due to induced PTH resistance. It is uncommon above a serum
magnesium level of 1mg/dL. Severe hypermagnesemia, although rare, can also
cause hypocalcemia by suppressing PTH secretion through a decreased
sensitivity of calcium-sensing receptors.
○ Option B: Physiological stimuli that cause vasopressin release in adjunct with
increased fluid intake can cause hyponatremia. Hypothyroidism and adrenal
insufficiency may contribute to an increased release of vasopressin.
Physiological stimuli for vasopressin release include loss of intravascular volume
(hypovolemic hyponatremia) and the loss of effective intravascular volume
(hypervolemic hyponatremia).
○ Option C: The etiology of hyponatremia can be classified based upon the volume
status of the extracellular fluid. As mentioned earlier, sodium is the major solute
of extracellular fluid (ECF). Based upon the volume of ECF, a patient can be
classified into hypovolemic, euvolemic, or hypervolemic.

You might also like